fluid mechanics and hydraulic machinery

260
E310/1 1 FLUID MECHANICS AND HYDRAULIC MACHINERY UNIT I PREPARATION OF FLUIDS Aims: This unit aims at cursory review of properties of fluids like, viscosity, Bulk Modulus, Vapour pressure, surface tension and capillarity. Objective: 1 . To distinguish between dimension and Unit. 2. To define various properties. 3. To distinguish between Ideal fluid & Real fluid Newtonian and non Newtonian fluids 4. Practical application of the above preparation in solving problem. 1.1 Introduction: Fluid Mechanics is the science of mechanics, which deals with the behaviour of fluids at rest or in motion. Both liquids and gases are called fluids. fluid is defined as a substance which is capable of flowing and deforms continuously under a shear stress however small it may be. Before review of the various properties of fluid, dimensions and units of measurement are discussed here. 1.2. Dimensions and units: Any physical quantity can be expressed in four fundamental dimensions namely, mass(M), Length(L), Time (T) and temperature (). But in incompressible fluid flows the 3 dimensions M,L, and T are adequate to define the physical quantities. The standards to measure the above dimensions of the physical quantities, are called unit of measurement. There are 4 systems of units as given below. 1.Centimeter - Gram - second (C.G.S) 2. Meter - Kilogram - Second (M.K.S) 3. Foot - Pound - Second (F.P.S) 4. International standard system (S.I) (latest system) In this present treatment of the subject , F.P.S is not used and mostly S.I. System is followed.

Upload: courtney-ward

Post on 24-Oct-2015

355 views

Category:

Documents


14 download

DESCRIPTION

Fluid Mechanics and Hydraulic Machinery

TRANSCRIPT

Page 1: Fluid Mechanics and Hydraulic Machinery

E310/1 1

FLUID MECHANICS AND HYDRAULIC MACHINERY

UNIT I

PREPARATION OF FLUIDS

Aims: This unit aims at cursory review of properties of fluids like, viscosity, Bulk

Modulus, Vapour pressure, surface tension and capillarity.

Objective:

1 . To distinguish between dimension and Unit.

2. To define various properties.

3. To distinguish between Ideal fluid & Real fluid Newtonian and non Newtonian fluids

4. Practical application of the above preparation in solving problem.

1.1 Introduction: Fluid Mechanics is the science of mechanics, which deals with the

behaviour of fluids at rest or in motion. Both liquids and gases are called fluids. fluid is

defined as a substance which is capable of flowing and deforms continuously under a

shear stress however small it may be.

Before review of the various properties of fluid, dimensions and units of

measurement are discussed here.

1.2. Dimensions and units: Any physical quantity can be expressed in four fundamental

dimensions namely, mass(M), Length(L), Time (T) and temperature (θ) . But in

incompressible fluid flows the 3 dimensions M,L, and T are adequate to define the

physical quantities.

The standards to measure the above dimensions of the physical quantities, are

called unit of measurement. There are 4 systems of units as given below.

1.Centimeter - Gram - second (C.G.S)

2. Meter - Kilogram - Second (M.K.S)

3. Foot - Pound - Second (F.P.S)

4. International standard system (S.I)

(latest system)

In this present treatment of the subject , F.P.S is not used and mostly S.I. System is

followed.

Page 2: Fluid Mechanics and Hydraulic Machinery

E310/1 2 The dimensions and units of various quantities useful are given below.

S.No. Quantity Dimension C.C.S M.K.S S.I

1. Mass M Gram (m) m.slug Kg(m)

2. Length L C.m metre metre

3. Time T Second second Second

4. Velocity L/T = LT-1 cm/s m/s m/s

5. Acceleration LT-2 cm/s2 m/s2 m/s2

6. Force MLT-2 Dyne Kg(t) Newton

7. Pressure ML-1T-2 Dy/cm2 Kg(f)/m2 N/m2 Pascal

8. Mass density ρ= ML-3 gm(m)/cm3 m.slug/m3 kg (m)/m3

9. Specific

weight

w= ML-1T-2 Dy/cm3 kg(t)/m3

(9.81N)

N/m3

10 Viscosity ML-2 T-1

FTL-2

Dy -s/cm2

(poise) Kg( f ) − sec

m 2

(98.1 Poise)

N.Sm 2

(10 poise)

11. Kinematic

Viscosity

L2T-1 cm2 /s

(stokes)

m2/s m2/s

12 Workdone

Energy

ML2T-2 Dy-cm Kg(t) -m N-m

(Joule)

13 Power ML2T-3 Dy-cm/s kg(t) -m/s

75 kg(f) .m/s

= 1H.p

N-m/s= J/S=

Watt

1000w

=1k.w

14 Surface

Tension

MT-2 Dy/cm Kg(f)/m N/m

Self Assessment Questions

SAQ 1. Define a fluid

SAQ 2. Distinguish between Unit and Dimension

Page 3: Fluid Mechanics and Hydraulic Machinery

E310/1 3 SAQ 3 Derive the dimension of power

1.3 Fluid Properties:

Density or mass density or specific mass = ρ It is defined as the mass per unit volume of

fluid. It is denoted by ‘ρ’

Unit C.G.S gm(m) /Cm3

M.K.S. m slug / m3

S.I kg (m) /m3

For water at 40 c Unit of mass density

C.C.S M.K.S S.I

ρ 1g(m)/cm3 10009.81

= 101.94mslug m 3 1000kg(m) /m3

Specific weight is defined as the weight of the fluid per unit volume . It is

expressed as ‘w’ or ‘λ’

Unit is given as

C.C.S M.K.S S.I

Unit Dy| cm3 Kg(f)/m3 N/m3

For water at 40 c, the Unit is given by

C.C.S M.K.S S.I

Unit 1gm(f)/cm3

981 dy/cm3

1000 (f)/m3 9810N/m3

=9.81K.N/m3

Further the following relationship of units are also useful in the further study of Fluid

Mechanics.

Kg(f) = 1m slug ×1m/s2 = kg(m) ×9.81m/s2

N = (9.81) Kg (m) ×1m/s2 = kg (m) ×1m/s2

∴1m.slug = 9.81 kg (m) = 9810g(m)

1Kg(t) = 9.81N = 1000g(t)= 981×105Dy

1Kg(m) = 1000g(m)

Dyne = 1g(m) ×1cm/s2

N= 1000g(m) ×100cm/s2 = 105 Dynes

1g(f) = 1g(m) ×981cm/s2

=981 Dynes

Page 4: Fluid Mechanics and Hydraulic Machinery

E310/1 4 Thus Kg(f), g(f) are called engineering or gravitational units, where as Dyne, Newton are

called as absolute units.

Specific Volume: is defined as volume per unit mass. It is reciprocal of mass density.

It is denoted by υ = 1ρ

Unit CC.S M.K.S S.I

Cm3/g(m) m3 /m.slug m3 /kg(m)

For water at 40 c , ν = 0.001 m3 /Kg(m) in S.I System

Specific Gravity is defined as the ratio of specific weight of any fluid to the specific

weight of standard fluid which is taken as water (for liquids) at 40c it is denoted by ‘s’.

SAQ 4. What is the specific volume of a fluid whose density is 90Kg(m)/m3.

SAQ 5 10 m3 of fluid weighs 90 KN. find its specific gravity

Viscosity is defined as the property of the fluid with which it offers resistance to

shear force. It is due to both cohesion and adhesion of the fluid. It is also known as

Dynamic Viscosity,. Absolute Viscosity, Coefficient of viscosity . As force is involved in

this phenomenon, it is called as Dynamic Viscosity. It is represented by ‘µ’.

Newton’s law of Viscosity:- It states that the shear stress is proportional to the velocity

gradient normal to the direction of flow of fluid ( in rate of angular defer motion)

Thus ταdϑdy

or τ = µ dϑdy

where τ is the shear stress.

consider a fluid of viscosity in between two parallel plates at distance ‘y’ apart as

shown in figure. The lower plate is at rest and the upper plate of plane area ‘A’ is moved

by a Force “F” with a velocity ‘v’. Due to adhesion the particles at the Solid boundary

are adhered to them and so the velocity of the particles at lower plate is zero and that of

the particles at upper plate are moved by a velocity V. In between the particle move

linearly from zero velocity to v velocity, setting by a velocity gradient vy

ordϑdy

in the

direction

Page 5: Fluid Mechanics and Hydraulic Machinery

E310/1 5 normal to direction of plate movement. Suppose if the force is increased, then the velocity

gradient also is increased. by experiments it is proved that the shear force F αAvy

for

number of fluids.

ieFA

= ταvy

=dvdy

Where dv is change in velocity in dy as the force F is increased over the place A, the dv/dy

also is proportionately increased.

Thus τα dνdy =

dθdt

or τ = µ dudy

This equation is called Newton’s Equation of viscosity

Also = µ =τ

dϑ dy

So Viscosity is also defined as ;shear stress per unit velocity gradient in normal direction

to the movement of plate.

Dimension of Viscosity is obtained as

µ =FA

yv

=FL2

LL T

=FTL2 (force l-length- time)

or M.LT −2

L2 .LT

=MLT

C.C.S M.K.S S.I

UNIT Dy − scm 2 poise

Kg( f )− sm 2

98.1 poise

N.sm 2 10 poise

For Water 200C Viscosity = 0.01 poise (one centi poise)

Page 6: Fluid Mechanics and Hydraulic Machinery

E310/1 6 Based on ;the shear stress and the respective velocity gradient in normal direction,

fluids are classified as discussed below.

i).Ideal fluids and Real fluids

we know that τ = µ dϑdy

if µ= 0 τ = 0

This means that when viscosity is zero , there will be no shear stress to oppose the fluid

flow and the fluid particle ‘slip’; over the solid boundary. This leads to Uniform velocity

distribution over the cross section al flow. Such fluids are called as Ideal fluids for which

viscosity is zero and there will be no loss of energy, in the fluid flow from point to point.

On the other hand in fluids if viscosity is not zero and fluid particles satisfy ‘no-slip’

condition over solid boundary, then small fluids are called as real fluids. So for real fluids

viscosity is not zero ;and the shear stress opposes the fluids flow to make the velocity

distribution to vary from zero at solid boundary to free stream velocity to set up a velocity

gradient. In the real fluids there will be loss of energy in fluid flow from point

to point

The ideal fluid is represented in the graph shown in picture on ‘x’ axis for which shear

stress = 0.

Newtonian and non-Newtonian fluids: If fluids like air. water. kerosene. glycerine etc.

obey the law of viscosity and the shear stress in these fluid is proportional to velocity

gradient. Then such fluids are called as Newtonian fluids. If the shear stress is not

proportional to velocity gradient, then these fluids are called as Non-Newtonian fluids as

shown in figure.

Page 7: Fluid Mechanics and Hydraulic Machinery

E310/1 7 Plastic and Thixotropic fluids

Some fluids flow after certain yield stress. Thus plastic will flow with linear

relationship shear stress and velocity gradient, after certain yield stress. Thixotropic fluids

after yield stress, flow with a non linear relationship between shear stress and velocity

gradient. printers ink is called as Thixotropic fluid. These fluids are represented in the

figure.

Kinematic Viscosity is defined as the ratio of dynamic viscosity for mass density of the

fluids. It is represented by. ‘ν’.

∴υµρ

=

In this unit only Kinematic of fluid, like length, Time are involved and so it is called as

Kinematic Viscosity.

Dimension of Kinematic Viscosity

υµρ

= = × = −MLT

LM

L T3

2 1

Unit of Kinematic Viscosity

Unit C.G.S M.K.S S.I

Cm2/s stokes m2/s m2/s

Kinematic Viscosity of water at 200 c is 0.01 stokes.

SAQ 6. Define ideal fluid

SAQ 7 Why shear stress is zero for a fluid at rest

SAQ 8 Why Viscosity is called as Dynamic Viscosity

SAQ 9 Distinguish between ideal and real fluids

SAQ 10 The Viscosity of a fluids 0.09 poise Determine its Kinematic Viscosity if

its specific weight is 9Kn/m3

SAQ 11 What are the unit and dimension of velocity gradient

SAQ 12 Why the ratio of viscosity to mass density is called as ‘Kinematic

Viscosity’

SAQ 13 (a). of velocity gradient in normal direction is 50m/s/m . Determine the

rate of angular deformation

Page 8: Fluid Mechanics and Hydraulic Machinery

E310/1 8 (b) If the rate of deformation is 1 rad/s find its velocity gradient.

SAQ 14 If the Viscosity is o/.s/m2 compute the angular deformation for a shear

stress of 0.1N .S/m2.

Worked Out Examples

Worked Example (1) Two parallel plates are 2 cm part. This space is filled with glycerine

Find the force required to drag this plate of area 0.5 sqm at a speed of 0.6m/s when it is

placed at 0.75cm from the lower plate Viscosity of glycerine is 8.0 poise.

F = µAϑ1y1

+1y2

810

× 0.5 × 0.61001.25

+1000.75

=51.2 N

Worked Example (2) A piston of 496 mm dia and 150 cm long slides vertically down a

cylinder of 500 mm dia. If the clearance is filled with lubricating oil of Viscosity 5×10-2

poise find the terminal speed of the piston The weight of piston is 5N.

Sol:

p= µ vy

× A

Page 9: Fluid Mechanics and Hydraulic Machinery

E310/1 9 A = ΠDL =Π×0⋅496×0⋅15=0⋅233m2

∴5 =5 × 10−2

10

×

v0.002

× 0.233

∴v= 8.5 m/s

Worked Example (3) A 15 cm dia vertical cylinder rotates at 100 r.p.m concentrically

inside another cylinder of 15.10 cm dia. Both the cylinders are 25 cm high. If the

viscosity 8.0 poise is filled in between the space in between the cylinder, find the power

absorbed in overcoming the shear resistance.

sol

Ω =2ΠN60 −

=2 × Π ×100

60= 10.47

velocity = ϑ =ΠDN

60=

Π ×15100

×10060

= 0.785m / s

Shear force =τ×area

µ vtΠDL

=8

10×

0.7850.05100

× Π × 0.15 × 0.25 = 148N

Torque T = F×r = 148 ×0.15

2= 11.1Nm

Power absorbed = T ×Ω = or F×U =116 Watts

Worked Example (4) If the velocity distribution of a fluid over a plate is given by

u =3/4 y-y2, where u = velocity in m/s at a distance of ym above the plate, determine

shear resistance at y = 0.15 m from the plate . viscosity = 0.00833 poise

Page 10: Fluid Mechanics and Hydraulic Machinery

E310/1 10 sol

Given u =34

y − y2

∴dudy

=34

− 2y ;dudy

=34

− 2 × 0.15 = 0.45m / s / m

y = 0.15

So shear resistance = τ = µ dudy

=0.00833

10× 0.45

= 0.375 ×10-3 N/m2

Surface Tension: The surface of contact between a gas and a liquid or between two

immiscible liquids acts as a film capable of resisting small tensile forces. It is due to

cohesion between the liquid particles at surface. This phenomenon is known as surface

tension which is due to the imbalance of the intermolecular attractive forces between

liquid particles at the interface. This imbalance of forces causes the film or interface to

behave like an elastic film under tension.

So surface tension is defined as the force required so keep unit length of the film in

equilibrium condition. It is denoted by ‘σ’.

C.G.S. M.K.S S.I

Unit Dy/cm kg(f)/m N/m

For water with air at 18 0c surface tension σ = 0.074 N/m (S.I)

= 75 Dy/cm (C.G.S)

Effect of surface tension : The internal pressure inside a fluid drop is increased due to

surface tension.

Determination of excessive pressure inside a drop

Consider a drop of a diameter ‘d’ is divided into two halves. Now consider the force that

are acting on it to keep it in equilibrium condition as shown in figure

Page 11: Fluid Mechanics and Hydraulic Machinery

E310/1 11 The external force and internal force acting on the drop in horizontal direction. are given

by pressure intensity and projected area of the drop in vertical plane

ie ΠD 2

4Pa and

ΠD 2

4pi respectively

Now consider the forces in equilibrium condition i.e. the ∑F in horizontal direction = 0

∴ΠD2

4pi − ΠD2

4pa − ΠDσ = 0

(Pi-Pa)/ΠD /2

4= Π /Dσ

Pe = 4σD

is the expression for the inside excessive pressure.

SAQ 15 Find the excessive pressure inside a cylindrical jet of water of diameter ‘d’

SAQ 16 Find the expression for excessive pressure inside a soap bubble of dia d.

(hint there will be two surfaces) in contact with air.

SAQ 17 If the dia of drop is increased so 4d from d, find the excessive pressures

ratio inside the drops.

Worked Example (5) Air is introduced through a nozzle into a tank of water to produce

a stream of bubbles of 2 mm dia. calculate the excessive pressure inside the bubble over

surrounding water pressure. Surface tension of water 74 dy/cm

Pe=4σd

=4 × 74 ×10−5

10−2 ×1000

2

=2×74=148N/m2

Capillarity We know that cohesion is intermolecular attractive force between

molecules of the same liquids, where as Adhesion is the attractive force between solid and

liquid or between two different liquids.

When a liquid wets a surface, adhesion is greater than Cohesion and so the liquid rises in a

capillary tube immersed in the liquid. Similarly when liquid does not wet a solid,

Cohesion is greater than adhesion and so the liquid drops in side the capillary tube. This

rise or fall of liquid in capillary tube is known as capillarity which is due to with adhesion

and cohesion of the liquid.

Derivation of an expression for capillary rise or fall :Consider the rise or fall of liquid

in the capillary tube in water and mercury as shown in figure. Water wets the tube

surface where mercury does not.

Page 12: Fluid Mechanics and Hydraulic Machinery

E310/1 12

In case of water which wets the surface of the tube, the adhesion is more than the

cohesion and so the density of water decreases in the tube at the liquid surface. So to

make the pressure over the liquid surface equal, the water rises in the tube till the pressure

is equal in side and outside over the surface.

In case of mercury , cohesion is more than adhesion, as it does not wet the tube

There fore the density of liquid increases inside the tube near the liquid surface and so to

make the pressure in side and outside of the tube to be equal over the surface, the liquid

falls in the tube.

So consider the equilibrium condition between the forces in vertical direction on

the liquid column in the tube.

upward force = downward force

due for surface tension due to weight of liquid

ie //Π /dσ cosθ =/Π /d 2

4h × w

h =4σ cosθ

wdWhere ‘θ’ is the angle of the surface tension with vertical

For pure water ‘θ’ with glass = 00

For mercury ‘θ’ with glass = 1400

SAQ 18 What is the inclination of surface tension with glass tube with vertical for

pure water.

SAQ 19 Determine the capillary rise in two parallel plates ‘d’ apart immersed in a

liquid.

SAQ 20 Capillary rise in 5 mm dia tube is 0.6 c.m. Find the same in 10 m.m tube.

Page 13: Fluid Mechanics and Hydraulic Machinery

E310/1 13 Worked Example (6) What should be the minimum dia of a glass tube used to measure

water level if the capillary rise is to be limited to 1mm. Surface tension of water = 75

dy/cm. θ = 0

h =4σ cosθ

wd= 0.1 =

4 × 75 ×1981 × d

∴d= 3cm

Vapour Pressure When a liquid with a free surface is combined in a closed vessel, the

vapour molecules from the free surface will escape into the space above free surface in the

vessel. The escape of vapour molecules continue till the saturation of the space within

vessel with vapour molecules. This vapour exerts a pressure on the liquid surface which is

called as partial vapour pressure.

If, in any fluid flow, the fluid pressure in dropped to its vapour pressure at that

room temperature, then the fluid starts to vapourise. This vapour and the liberated

dissolved gases form as vapour bubble which grow in size. After some time these

bubbles move to higher pressure zone, where they collapse (decrease in size due to high

pressure) releasing very high pressures which are nearly equal to 20 atmospheric pressure.

This high pressure erode the solid boundaries by repeated blows. This phenomenon is

called as Cavitation. So care should be taken in design of Hydraulic machines to avoid

cavitation which causes damage of solid surface of machines.

Mercury vapour pressure is very low and so it is an excellent fluid for manometers.

S.I M.K.S

Unit N/m2 Kg(f)/m2

For water at 200c , vapour pressure = 1.23 K Pa(abs)

For pure water v.p = 18 to 24 Kpa(cabs)

SAQ 21 Which fluid is good for manometers

Bulk Modulus:

When ever a pressure is applied over a fluid in a container it will be compressed .

Compressibility of a fluid is defined as change in volume under a pressure It is the inverse

of Bulk Modulus of Elasticity ‘K’

which is defined as

K∆p−∆v

v

=stressstrain

Page 14: Fluid Mechanics and Hydraulic Machinery

E310/1 14 C -ve sign indicates the decrease in volume due to increase in pressure.

Unit N/m2 (S.I) or kg (f) /Cm2

K for water = 2.06×109 N/m2 at N.T.P

K for air = 1.03 ×105 N/m2

ie K of water = 20,000 × k of air

So water is considered to be incompressible as its K is very high. That means

the density of incompressible fluid remains constant whatever may be the pressure over it.

Air is compressible fluid.

Summary

1. Fluid is defined as a substance which is capable of flowing and deforms

continuously under a shear stress however small it may be

2. In fluid mechanics dealing with in compressible fluid, 3 fundamental

dimensions namely Mass (M), Length (L) & Time (T) are used to express

any physical quantity.

3 Units are the standards of measurement of the above dimensions.

4 3 systems of units namely C.G.S, M.K.S, and S.I. are used in present

treatment of the subject. But mostly S.I system which is the latest system is

followed

5 Kg(f) = 9.81 N ; 1M slug = 9.81 kg(m)

6. Viscosity is defined as the property of fluid with which it offers resistance to

shear force. Its unit in C.G.S is poise

7 Kinematic Viscosity is the ratio of viscosity to the fluid mass density

8 Ideal fluid is that for which viscosity and shear stress are zero.

9 Surface tension is defined as the force per unit length of the film to keep it in

equilibrium condition.

10 Vapour pressure is the partial pressure exerted by the vapour on the fluid.

11 Compressibility of fluid is the change in volume of fluid under a pressure and is

the inverse of bulk modulus of a fluid.

Answers to self assessment questions:

3. ML2T-3 4.0.0011m3/kg(m) 5. s= 0.91

7. dv/dy = 0 10. v =0.098 stokes 11. m/s/m/ -7-1

13. tan θ = 250, θ=890 .7, dθ/dt = 0.498 rad/s

Page 15: Fluid Mechanics and Hydraulic Machinery

E310/1 15 Vel gradient = 1.55 m/s/m

14. tan θ = 1, θ= 450 , 15. 2σd

16.pe = 8σd

17. p1

p 2

= 4 18. θ=0 19. h = 2σ cosθ

wd

20. h2 = 0.3cm

EXERCISE 1.(1) Of 5.27 m3 of a certain oil weights 44 KN, calculate the specific weight,

mass density and specific gravity of oil (8349N/m3 , 85109 Kg/m3 , 0.85)

1.(2) A certain liquid has a dynamic viscosity of 0.073 poise and specific gravity

of 0.87 compute the kinematic viscosity of the liquid in stakes of SI.I

system (0.08 stakes, 0.083x10-4m2)

1.(3) of the equation of a velocity distributing over a plate in given by v=2y-y2,

where v is velocity in m/s at a distance ‘y’ in from the solid surface,

determine the shear stress normality at 7.5 cm and 15cm from it.

Given m =8.6 poise.[0.175 Kg(f)/m2 , 0.162 Kg(f)/m2 , 0.149Kg(f)/m2 ]

1.4 Two large place surfaces are 20mm apart and the gap contained oil of

dyamic viscosity 0.6 poise. A this plate of 0.5m2 surface area is to be

parallel although the gap at a constant velocity of 0.6 m/s. The plate is

placed at 8mm from one of the surfaces. Find the face required to parallel

the plate. [F=3.75 N].

1.5 A flat plate of area 0.15m2 is to be tossed up an inclined plane of slope 1

vertical to 3 horizontal on a thin layer of oil m=0.75 poise of thickness

0.2mm. Of the weight of the plate in 250N , estimate the force required to

pull the place at 1.6m/s velocity [F=979N].

1.6 A piston of 7.95 cm diameter and 30 cm long works in a cylinder of 8.0 cm

diameter. the annular space of the piston is filled with an oil of viscosity 2

poise. Of an axial load of 10N is applied to the piston, calculate the speed

of the piston(v=16.68 cm/s)

1.7 A 90mm diameter shaft states at 1200rpm in a 100mm long cylinder

90.5mm internal dia mater. The annular space in the shaft and cylinder in

filled with oil of viscosity 0.12poise. Find the power absorbed to overcome

shear friction. (P=434 w)

Page 16: Fluid Mechanics and Hydraulic Machinery

E310/1 16 1.8 A tube of internal diamter 2mm is dipped vertically into a vessel containing

mercury. The lower and of the tube is 2cm blow the mercury surface.

Estimate the pressure of air inside the tube to blow a semi spherical bubble

at the lower end. Surface tension of the mercuty is 0.4N/m. [∆P=3.468

Kg/cm].

1.9 A soap bubble 51mm in diameter has an internal pressure in excess of the

external pressure of 0.00021Kg(f)/m2 . Calculate the tension in the soap

film [0.0134kg(f)/m].

1.10 Calculate the capillary rise ’h’ a glass tube of 3mm diameter when

immersed in water at 20oC. The surface tension at 20oC is 0.075kg(f)/m.

What will be the percentage increase in the value of ‘h’ if the diameter of

the tube in 2mm [10mm,50%].

1.11 By how much does the pressure in a cylindrical set of water 4mm in

diameter exceed the pressure of the sorrounding atmosphere of

σ=0.0075kg(f)/m, [3.75kg(f)/m].

Reference books 1. Fluid Mechanics and Hydraulics & Hydraulic Machinery P.N.Modi & S.M.Seth 2. Fluid Mechanics, Hydraulics & Hydraulic Machines K.R.Arora 3. Fluid Mechanics by K.Subramanyam

***

FLUID MECHANICS AND HYDRAULIC MACHINERY

UNIT II

HYDROSTATICS

Aim:

The aims of this unit are to define pressure at a point, the measuring of pressure head,

different standard practices of expressing the pressure, the principle of pressure

measurement by differential and micro manometers and fluid pressure over curved

surfaces.

Page 17: Fluid Mechanics and Hydraulic Machinery

E310/1 17 Objectives

1. To define fluid pressure and explain the meaning of a pressure head.

2. To explain atmospheric pressure gauge pressure and their relationship.

3. To explain differential and micro manometers and obtain expressions for differential

pressure.

4. To obtain expressions for hydrostatic pressure over curved surfaces which are

submerged, in horizontal and vertical directions and then to find resultant force over the

curved surface

5. To apply the above in solving the practical problems.

2.1. Introduction:

In Units (2) and (3) the fluid at rest is considered . As there is no motion of fluid

tangential forces are zero. So hydrostatic forces of fluid at rest are normal for the solid

boundaries .

In this Unit the fluid pressure is defined. The relationship between absolute atmospheric

and gauge pressures is explained. Differential and micro manometers are explained which

are useful to measure differential pressures. These manometers are useful for measuring

devices.

Knowledge hydrostatic forces on curved surface is useful in design of crest gates of

spillways etc.

2.2 Fluid Pressure is always normal to the surface. It is also called a pressure intensity.

Definition of fluid pressure ‘p’ It is defined as the weight or normal force of fluid

on unit area

p =PA

where‘p’ is normal force

P = Wt of fluid column over the area

= Volume ×Specific weight

= Ah × w

So P = w /Ah

/A= wh

∴p=wh

∴ Pressure at any depth ‘h’ is given ;by ;the product of ‘depth and specific weight of the

fluid’. It is same in all directions in a fluid at rest.

Page 18: Fluid Mechanics and Hydraulic Machinery

E310/1 18 Unit is N/m2 or Pascal (S.I)

Kg(f) /cm2 (as kg(f)/m2 is too big)

From the above expression we have

h =pw

which is called as pressure head as its dimension is only ‘L’.

2.3 Atmospheric pressure is the normal pressure excerted by atmospheric air over the

surface. It is measured by a barometer as shown in figure . In the top of tube of

barometer the pressure acting by air is zero as there is no air in that space. So this is

taken as absolute zero. This refers to complete vacuum.

At M.S.L the standard atmospheric pressure ‘p’ is 76 cm of mercury

=76

100×13.6 ×1000 Kg(f)/ m2

=76

100×

13.6 ×1000100 ×100

Kg(f)/cm2

= 1.03Kg(f)/cm2

=101KN/m2 or 101Kpa

So standard atmospheric pressure is given ;by 76 cm of mercury, 1.03 kg(f) /cm2 ,

101Kpa. The atmospheric pressure head is given by

ha =pa

w=

1.03 ×104

1000= 10.3 m of water

Local atmospheric press ure will vary from place to place, which can be measured by the

barometer.

SAQ 1. Find the magnitude of standard atmospheric pressure in meters of water.

SAQ 2 Pressure head at a point is 20 m of water . Find the pressure in Kpa.

SAQ 3 Pressure at a point is 98.1 Kpa . Find the pressure head in metres of liquid if

specific gravity 0.9.

Page 19: Fluid Mechanics and Hydraulic Machinery

E310/1 19 SAQ 4 The pressure at a point in a fluid at rest is same in all directions. True/False.

SAQ 5 The atmospheric pressure will vary with altitude. True/False.

SAQ 6 Local atmospheric pressure is constant at any place. True /False

SAQ 7 Atmospheric pressure depends on Viscosity . True /Flase.

2.4 Absolute Pressure and Gauge pressure

Absolute Pressure When a pressure is measured above absolute zero (as datum) it is

called as absolute pressure.

Gauge Pressure If the pressure is measured above or below atmospheric pressure

then it is called as gauge pressure. It is so because , when the gauge which measures the

pressure, is opened to atmosphere, reads zero.

Of the pressure is above atmospheric pressure it is called as positive pressure or gauge

pressure. If it is below atmospheric pressure it is called as negative pressure, section

pressure or vacuum pressure. of the pressure below atmospheric pressure, is expressed as

vacuum, then ‘-ve’ sign is not necessary for the value.

Relationship between absolute, atmospheric and gauge pressures

All the pressures are shown in figure. From this figure the relationships are obtained as

given below.

Absolute pressure = Atmospheric pr+ Gauge pr

” = Atmospheric Pr - Vacuum Pr

So absolute pressures are obtained by adding gauge pressure or deducting vacuum

pressure to/from atmospheric pressure respectively.

2.5 Measurement of Pressure

Introduction

We know already that pressure at a point can be measured by manometers. Simple

manometers were already known by you, which are used to measure the pressure at a point

Page 20: Fluid Mechanics and Hydraulic Machinery

E310/1 20 in a fluid. Now let us discuss about differential and micromanometers to measure the

differential pressure between two points in the same pipe line or in two different pipes.

Differential manometer This is used to measure the difference which is considerable,

between two points in same pipe or in two different pipes.

It consists of a glass ‘U’ tube. The two ends of the ‘U’ tube are connected to the two

points between which the difference of pressure is to be measured. The ‘U’ tube contains

in its lower portion a heavier manometer fluid than the fluid in the pipe and immiscible

with the fluid in the pipe

Expression for differential pressure

Consider a differential ‘U’; tube manometer as shown in figure. Let it be connected

between two pipe lines in which a liquid of sp gr sl is flowing. Now let us calculate the

pressure heads in terms of liquid flowing , at each interface in the ‘U’ tube. We know the

pressure will be more, below a column of manometric head is converted to liquid head as hsm

sl

. With this idea, let us consider the pressures from point (1) to point (2)as shown

below .

p1

wl

+ x + h

is the pressure head at A

p1

wl

+ x + h − hsm

sl

is the pressure head at B

In the same way, finally we have

Page 21: Fluid Mechanics and Hydraulic Machinery

E310/1 21 p1

wl

+ x + h − hsmsl

− y =p2

wl

This is known as gauge equation of the manometer in

terms of ‘m’ of liquid.

i.e. p1 − p2

wl

= hsm

sl

−1

− x − y( )

h(sm

sl

−1) − z

m of liquid

If z= 0 , ie the points are at the same level

Then p1 − p2

wl

= hsm

sl

−1

m of liquid.

p1 − p2

w= h(sm − sl) m of water

Qwl = sl×w

Here ‘h’ is to be measurable with accuracy, otherwise errors will be involved in pressure

measurement. In some cases ‘h’ may be too small to be measure accurately. Then

inverted ‘U’ tube manometer is t o be used.

Inverted ‘U’ tube manometer

This is used to measure small differential pressures. It is an inverted ‘U’ tube manometer

as shown in figure. The manometric fluid which is in the upper portion of ‘U’ tube, is

lighter than the fluid flowing in the pipes. An air cock is provided at the top to expel

entrapped air.

Page 22: Fluid Mechanics and Hydraulic Machinery

E310/1 22

Now the gauge equation is written as follows in terms of head of flowing fluid. p1

wl

− z − /y − h + hsm

sl

+ /y =p2

wl

p1 − p2

wl

= h 1 −sm

sl

+ z m of liquid

Now Qwl = slw, and if z = 0 , then we have

p1 − p2

w= h(sl − sm) . m of water

Micromanometer is used to measure small differential pressures very precisely .It

consists of two transparent reservoirs of enlarged section at the top of the two limbs of

‘U’ tube manometer as shown in figure. The manometer contains two manometric liquids

of different sp.gr and immiscible with each other.

Let tube CS area = a

Page 23: Fluid Mechanics and Hydraulic Machinery

E310/1 23 and reservoirs C.S area = A

When the micromanometer is connected to the points (1) and (2) , the fluid of sp gr s2 is

dropped by ∆2 in the reservoir connected to point (1) and rises by the same amount in the

another reservoir connected to point (2). Due to this drop, the fluid of sp gr s, in the ‘U’

tube is dropped by h/2 in the tube connected to point (1) and rises by the same amount in

other tube connected to point (2).

∴ The volume of fluid ∆z ×A= ah/2

or ∆z =aA

h 2

Now let us write gauge equation for the micro manometer as shown below in terms of

heads of liquid flowing in the pipes. p1

wl

+ x + ∆z + ( /y − ∆z)s2

s3

−h2

s2

s3

− hs1

s3

− ( /y −h2

+ ∆z) − ( /x − ∆z ) =p2

wl

p1

wl

+ 2∆z − 2∆zs2

s3

+h2

+h2

s1

s3

− hs1

s3

=p 2

wl

(p1 − p2 )wl

=hs1

s3

−hs2

s3

− 2∆z(1 −s2

s3

); as 2∆z =aA

h

we have (p1 − p2 )

wl

= hs1

s3

−s2

s3

aA

h(1−s2

s3

)

= hs1

s3

−s2

s3

−aA

+aA

s2

s3

h =s1

s3

−s2

s3

(1 −

aA

) −aA m of liquid

(p1 − p2 )

w= h s1 − s2 (1−

aA

) −aA

s3

m of water

If A is large aA

is too small and so can be neglected

Then (p1 − p2 )

w= h(s1 − s2 ) m of water

Here h = 2(∆z aA

)

This means by making A large ∆z is magnified to ‘h’ which is considerable.

Page 24: Fluid Mechanics and Hydraulic Machinery

E310/1 24 SAQ 8. Absolute pressure at a point is 2 m of water. Obtain vacuum pressure in k Pa.

SAQ 9. The pressure of a fluid at rest is always normal to the surface (true /false)

SAQ 10 Convert 2 m of water into mercury height

SAQ 11 Distinguish between negative pressure and vacuum pressure.

SAQ 12 Gauge pressures are measured by taking atmospheric pressure as datum.

(True/False)

SAQ 13 Convert 10 cm of mercury deflection in a differential manometer into a fluid of

sp. gr 0.9.

SAQ 14 The differential pressure head in 10 cm of water. Find the deflection of kerosine

in inverted ‘U’ tube manometer when fluid of sp. gr 1.6 is flowing in pipes.

SAQ 15 If the differential pressure is 20 k Pa determine the deflection in micro

manometer in m of water in which there is mercury in lower portion, fluid

of sp gr 1.6 in reservoirs and kerosine of sp gr 0.9 in the two pipes. The

reservoirs are too large.

SAQ 16 Very small pressure differences are measured accurately by micro manometer

(True /False)

Worked Example (1) Calculate the pressure at A in KN/m2 and m of water at the bottom

of the vessel shown in figure.

sol:

Total pressure at A

p = 0.8×9810×1.2

+0.95×9810×1.6

Page 25: Fluid Mechanics and Hydraulic Machinery

E310/1 25 +2×9810

=9417.6+14911.2+19620.0

=43948.8 N/m2 = 43.9488 KN/m2

Pressure head in meters of water = 43948.8

9810= 4.48m

Worked Example (2) Find the differential pressure between A and B in the figure

shown

Sol:

By gauge equation we have pA

w1

+ y − 25 ×0.961.2

+ (25 − /y) + 40 =pB

w1

pA − pB

w1

= 250.961.2

−1

− 40

=-45cm of liquid

Worked Example (3) Find the pressure difference of water in the pipes A and B as

shown in figure

Page 26: Fluid Mechanics and Hydraulic Machinery

E310/1 26 sol:

By gauge equation we have pA

w−15 −15 ×

13.61

− 15 =pB

wpA − pB

w= 15(13.6 + 1) + 15

` = 2.34 m of water

(pA-ps) = 2.34 ×9810=22955 N/m2

=22.955 K Pa

Worked Example (4) A micromanometer is shown in figure

Find the displacement of surface of separation when pressure over C is greater than B by 1

cm of water. Given C.S areas of tubes = 0.25 sq cm and of bulbs = 10 sq cm respectively

.n B side water if filled as C side red liquid of sp gr 0.9 is filled

sol:

By writing the gauge equation we have

pc

w+ (Zc − ∆z + h)0.9 − h − zb − ∆z =

pB

wpc − pB

w= 1cm = h + /zb − ∆z − (/zc − ∆z + h)0.9

But in the initial condition,

We have Zc ×0.9 = Zb ×1

∴pc − pB

w= 1cm = h + ∆z − (h − ∆z )0.9

but ∆z ×A = h×a, I e ∆z= h × a

A= h ×

0.2510

=h

40

Page 27: Fluid Mechanics and Hydraulic Machinery

E310/1 27

so pc − pB

w= 1 = h + h ×

140

− h −h40

0.9

=h 1 +140

−1 × 0.9 +140

× 0.9

1 cm = h0.1 +

1.940

= h × 0.1475

∴h = 6.78 cm

Pressure and curved surface

Introduction: We know already the total pressure acting on a plane vertically submerged

in liquid in give by

p= wx ×A

That means the total pressure over a vertical plane sub merged in fluid is given by the

product of pressure at centre of gravity of the plane from free surface and its area.

Also the centre of pressure in given by h =IGG

Ax+ x from the free surface. with this

understanding the expression for total pressure acting on curved surface submerged in

liquid.

Derivation of expression for total pressure.

Consider a curved surface submerged in a liquid of sp wt ‘w’ at a depth as shown in

figure. Let dt be an elemental area in the surface. The depth of liquid over this area is h.

Then the total pressure due to this depth of liquid over the elemental area is

dp = dA×wh

If this force is resolved into horizontal and vertical components as given below

Page 28: Fluid Mechanics and Hydraulic Machinery

E310/1 28 dpH = dpsinθ= dAwhsinθ = wh(dA sinθ)

Where θ is the inclination of dp with vertical. dpH is the horizontal component and the

pH vertical component dpv is given by

dpv = whdA cos θ

By integrating over the area, the above expressions we have pH = whdAcurvedarea

∫ sin θ

pH = pressure acting over vertical projection of the submerged curved area as dA sinθ

leads to vertical projection of the area dA.

This pressure acts at the C.G of the vertical projection of curved area.

Similarly , the vertical component is given by pv = whcurved area

∫ dA cos θ

As dA cosθ leads to horizontal projection of the area, wh dA cosθ leads to

weight of liquid over the area dA. So the integration leads to weight of liquid over the

curved area up to free surface. Therefore vertical component

pv = weight of liquid over the curved area upto free surface . It acts at the

C. G of the weight of liquid over the curved area.

Finally p = pv2 + pH

2

and its in clination with horizontal is given by’α’ = tan -1 (pv /pH ). The point of action of

p over the curved surface is obtained by extending the line of action of p through the point

of inter section of pv & pH to meet the surface at ‘O’ as shown in in figure..

Suppose the curve is as shown in figure. Then the vertical pressure is the imaginary

weight of liquid in the portion AOB.

Page 29: Fluid Mechanics and Hydraulic Machinery

E310/1 29

SAQ 17. The vertical pressure over a submerged curved surface is equal to the weight of

the liquid over the curved surface True /False

SAQ 18 The point of action of the vertical pressure over submerged curved surface

passes through the C.G of the weight of liquid over the surface. True/False

SAQ 19 Water is standing over a cylinder of 2m dia. Find the vertical force and its

point of action.

Worked Example (5) A 600 sector of 4 m radius is as shown in figure with one of

its edges horizontal, hinged to piers of spill way. Find the magnitude and direction of the

resultant force if the length of the gate is 3m. water is standing upto top of gate.

sol:

The horizontal force pH = ωA x

where A = BC×3

= 4 sin 60 ×3

∴pH = 9810 ×(4sin 60 × 3) ×(4 sin 60)

2

= 176.6 K N

Page 30: Fluid Mechanics and Hydraulic Machinery

E310/1 30

pV = Imaginary Wt of water on the curve A C which is in upward direction.

= (Area of gate portion in ABC) ×w×3

=( area of sector - ∆G area of BOC) w×3

=(ΠQ 2 ×60

360−

12

× 4cos 60 × 4sin 60) 9810×3

=(Π ×16

6− 3.464) × 9810 × 3

= 144.6KN

Total Pressure = p = pv2 + pH

2

= (176.6)2 + (144.6)2 K.N = 228.3KN

Its inclination with horizontal = α = tan −1(pv

pH

)

tan-1 (176.6144.6

= 0819

∴ α= 39.30

Worked Example (6) A cylindrical gate 2m dia is kept ion a floor with its

longitudinal ax is horizontal. The depth of water on one side is 2m and another side in 1

m. Calculate the resultant hydrostatic pressure on the gate and the minimum weight of it

so that it will not float away from the floor/ m length.

Sol:

Page 31: Fluid Mechanics and Hydraulic Machinery

E310/1 31 The horizontal forces are

pH1 = wA1 y1

9810 × (2 ×1) ×22

= 19.62KN

pH2 = WA2 y2

= 9810×(1×1)12

= 4.905 KN

∴The net horizontal force = pH1 - pH2 = 14.72 KN

The vertical forces are

Pv1 = ΠR2

2×1

9810 = 15.4KN

acts at x1 =4R3Π

=4 ×1

3 × Π= 0.42

Pv2 =ΠR2

4×1

9810 = 7.704KN acts at x 2 =

4R3Π

= 0.42

Total upward force = 15.4+7.704 = 23.104 KN

Resultant force = 14.722 + 23.12 = 27.4KN

Its inclination with horizontal = tan −1 (23.1

14.72)

=1.57, α = 57.50

The minimum weight of the cylinder not to be lifted by vertical forces 23.104 k.N

Summary

1. Fluid pressure is defined as weight of normal force of the fluid over unit area. It acts

normal to the surface

p = wh, pressure head = h = p/w

2. Absolute pressure = Gauge pressure + Atmospheric Pressure

Atmospheric pressure - Absolute pv = vacuum p2

3. Vacuum pr and -ve pr are same in magnitude, but vacuum pr will have no ‘-ve’ sign.

4. The differential pressure between two points as measured by differential ‘ U’ tube

manometer as ` (p1 − p2 )

w= h (sm -sl ) m of water

if it is inverted ‘U’ manometer (p1 − p2)

ω= h (sl -sm)

Page 32: Fluid Mechanics and Hydraulic Machinery

E310/1 32 m of water, which is useful to measure small pressure differences.

5. The small differential pressure is measured micromanometer very precisely as

(p1 − p2)

w= (s1 − s2 (1 −

aA

) − s3aA

) m of water

a - C.S of tubes

A- C.S of bulbs

= h(s1 -s2) m of water if A is too large

6. The horizontal pressure over a submerged curved surface is the total pressure over the

vertical projected area of the curved surface

pH=ωAx , Where is the C.G of the area A - which is the projected area in vertical

direction

7. The vertical pressure over a curved surface area which is submerged in the weight of

liquid over the surface up to free surface. If acts at the C.G of the weight of liquid over

the surface.

8. The resultant pressure over the curved surface is p = pH2 + pv

2

or its inclination with horizontal

α = tan −1( pv pH )

Answers for SAQ

1. 10.3m 2. 196.2 Kpa 3. 11.1m

4. True 5. True 6 False

7. False 8. 81.38 Kpa 9 True

10 .14.7 cm 12. True 13. 1.41 m of liquid = 1.27 m of water

14 14.28 cm 15. 17 cm of mercury 16 True

17 True 18 True 19. pv = 15.4 K N

C.G 4R3Π

= 0.42 m from centre of cylinder.

EXERCISE

2.1 For the given closed tube, the pressure at tip ‘A’ is 12cm of mercury vaccum.

Then determine the levels of liquid, in the tubes E,F & G from base

[38.96m, 27.168m, 18.48 m]

Page 33: Fluid Mechanics and Hydraulic Machinery

E310/1 33

2.2 A pipe containing water at 172 KN/m2 pressure in connected by a differential

gauge to another pipe 1.5m lower than the first pipe and contains water at high

pressure. The mercury deflection in the manometer is 7.5cm. Find the pressure in

the lower tube.[196KN/m2].

2.3 An inverted ‘U’ tube in connected between two pipes A and B. B is 60cm above

A. Water is flowing in A ∝ B. the deflection of manometer liquid specific gravity

0.9 is 50cm. Pressure in A is greater than B. Determine the pressure

difference.[0.058kg(f)/cm2.]

2.4 In the left tank shown in figure, the air pressure is -0.23m of mercury.

Determine the elevation of the gauge liquid in the right hand column at a, if the

liquid in the right and tank is water [94.62m].

Page 34: Fluid Mechanics and Hydraulic Machinery

E310/1 34 2.5 A micro-manometer consists of two cylindrical bulbs A and B each 1000sq.mm

(cross) sectional area which are connected by a ‘U’ tube with vertical limbs each of

25 sq. mm cross-sectional area. A liquid of sp.gr 1.2 is filled in A and another

liquid of sp gr 0.9 is filled in B, the surface of separation being in the tube attached

to B. Find the displacement of the surface of separation when the pr on the surface

B is greater than that in A by an amount equal to 15mm head of water.

[42.6mm].

2.6 For the container shown in figure find the resultant force on the hemispherical

bottom. [Rv=52.46KN,RH=0]

2.7 A radical gate retains 5m of water above the rest of a dam as shown in figure. Find

the resultant force on the gate per metre length.[R=124.4KN α=10.24o with

horizontal].

2.8 For a cylindrical gate 4m long shown in figure calculate the resultant force due to

fluid pressure. [R=219.64KN α=40.340 with horizontal].

Page 35: Fluid Mechanics and Hydraulic Machinery

E310/1 35

2.9 A sector gate of radius 4m and 5m long controls the flow of water in a horizontal

channel . Determine the total thrust on the gate. [ 205.7KN, θ=170-30’ with

horizontal]

2.10 A cylinder of 1m diameter supporting water is shown in figure. Find the reactions

at A and B if the cylinder is 5m long and has a weight of 6000kg(f).

2.11 A quadrant of a cylinder 3m long is shown in figure. Find the horizontal and vertical

forces FX=18000kg(f). F1=21424.2 kg(f)]

Page 36: Fluid Mechanics and Hydraulic Machinery

E310/1 36

2.12 Find the horizontal and vertical components of the force an the gate per metre

length and the resultant force and its direction. Shown that the resultant force

passes through the hinge.

References

Same as unit(1)

****

FLUID MECHANICS AND HYDRAULIC MACHINERY

UNIT-III

BUOYANCY AND FLOATATION

Aims: To know about the buoyant force, centre of Buoyancy, meta centre, meta centric

height and stability of floating bodies. Finally to gain ability to design a floating

body to support a given load.

Objectives:

Page 37: Fluid Mechanics and Hydraulic Machinery

E310/1 37 (1) To determine the buoyant force and the centre of buoyancy.

(2) To determine the conditions of stability

(3) To define metacentre, meta centric height and to determine the metacentric height

theoritically

(4) To determine the meta centric height experimentally

(5) To apply the above, in solving the practical problems.

3.1 Introduction

We know already that Fluid static deals with fluids at rest and so there is no motion

nor any shear forces between any two layers of fluid. So only forces involved are pressure

forces and body forces.

In the previous units it was shown that the pressure at a point in fluid at rest is

p=wh, where

h=the depth of fluid above that point.

This concept is useful in this unit .

Using Archimede’s principle the force of buoyancy is determined.

3.2 Buoyancy

When a body is immersed in fluid either wholly or partially, it is subjected to an

upward force which tends to lift the body up. This forces is in opposite direction to the

gravity force which is the weight of the body. The tendancy of the liquid to cause the

buoyant force over a floating or submerged body is called as buoyancy. The point of

application of the buoyant force on the body is called as centre of buoyancy.

Determination of buoyant force

The buoyant force over a submerged or floating body can be determined by

Archimede’s principle. This states that when a body is immersed in a fluid either wholly

or partially, it is buoyed or lifted up by a force which is equal to the weight of fluid

displaced by the body.

To obtain the buoyant force consider a wholly submerged body as shown in figure.

Page 38: Fluid Mechanics and Hydraulic Machinery

E310/1 38

The horizontal force that are acting on it are equal in magnitude and opposite and so the

net The horizontal force that is acting on the body is zero, as they are acting on the vertical

projection of the body. Now to find the resultant vertical force over the body, consider a

vertical prisum of ‘dA’ cross sectional area, out of the body.

The downward force over the area=p1dA

The upward force over the area= p2dA

So the net upward force = p2dA - p1dA

=(wh2-wh1)dA

=wydA=wd A

Where elemental volume of prisum=dv =ydA and wdv is the weight of the prisum of fluid

which is acting upward over the body.

∴ dFB =Elementar buoyant force = wydA=wdv

By integrating this over the body, we have

FB=Buoyant force ∫wdv =w v

This shows that the weight of the fluid displaced by the submerged body which is

equal to Buoyant force which acts vertically upward through the centroid of the volume of

fluid displaced. this centroid is equal to centre of Gravity of the body.

Page 39: Fluid Mechanics and Hydraulic Machinery

E310/1 39 If the body is immersed in two immiscible fluids of specific gravity ω1 and ω2 as shown

in figure, the buoyant force

=FB=w v1 + w2 v2

The centre of buoyancy coincides with the respective Centre of Gravity of the areas of the

body in the respective fluids.

When a body is floating as shown in figure the buoyant force FB=w v , whre v is

the volume of fluid displaced by the body. As the body is in equilibrium condition,

Upward force = down ward force

is Buoyant force = weight of the body

FB=W.

The line of action of these forces must lie along the same vertical line to make the

net moments about any axis zero.

Principle of floation: It states that the weight of a floating body in a fluid is equal

to the buyoyant force which is equal to the weight of the fluid displaced by the body.

When a body is immersed in a fluid if the buoyant force is more than the weight of

the body, then the body will rise up till the weight of the body is equal to the buoyant

force. If the weight of the body is more than the buoyant force, then it will sink.

SAQ (1) Buoyant force is equal to weight of body . True/False

SAQ (2) If the volume of water displaced by a body is 10m3. find the buoyant force.

SAQ (3) If the buoyant force more than the weight of body, then the body will rise

till its weight is equal to buoyant force. True/False

SAQ (4) Buoyant force in sea water is more. Why?

SAQ (5) 0.2 m3 body weighs 1.5KN in water. Determine its weight in air

Page 40: Fluid Mechanics and Hydraulic Machinery

E310/1 40 Worked Example(1) A boat when moved from sea water to fresh water sinks by 5cm. A

man weighing 686.7N gets out of the boat to keep it to its original

level. Find the weight of boat. Sea water w=10.06KN/m3.

Sol:

Let ‘W’ be the weight of boat

W=FB=v1w1

We know

W-686.7=∀1w2

is v1w1-686.7=∀1w2

v1(w1-w2)=686.7

∴ v1=686.7

(10.06 − 9.81)1000= 2.8m 3

So weight of boat and man

W=w1v1=2.8x10.06KN=28.15KN

∴weight of boat

=W-686.7

=27.485 KN

Worked Example (2)A rectangular wooden barge without a deck is 6m long, 3m wide

and 1m deep weighs 26KN. find the load that it can carry when water level is 0.6m above

bottom.

Sol:

Buoyant force FB

=6x3x0.6x9810=105.95 KN

∴Total weight = FB

W of boat +load = FB

Page 41: Fluid Mechanics and Hydraulic Machinery

E310/1 41 load = FB - weight of boat

=(105.95-26)KN=79.95KN

3.3 Metacentre and Metacentric height

When a floating body is in equilibrium condition the two forces that are acting

over it are (1) weight of body ‘W’ acting through its centre of gravity and (2)

Buoyant force FB equal to weight of fluid displaced by the body, which acts

vertically upward through the centre of buoyancy. Therefore these two forces must

be equal is

W=FB. Since there is no moment about any axis of the forces acting over the

body, which is in equilibrium condition, the two forces must lie on the same

vertical line as shown in figure(1)

Figure(1) Figure(2)

Now if the body be titled by an external force through a small angle ‘θ‘(angle of heel), the

buoyant force is changed and its line of action shifted to B2 from B1 as the value of water

displaced is more towards B2 while the G is assumed to be the same, even though a small

shift may be there.

The new buoyant force acts vertically upwards through B2 which meets the

extended line GB, at M. This point of inter section is called Metacentre and the distance

from Metacentre to centre of Gravity of the floating body, GM is called Metacentric height

as

θ→LO.

SAQ(6) Define metacentre and Metacentric height.

Page 42: Fluid Mechanics and Hydraulic Machinery

E310/1 42 3.4 Stability of floating bodies(or submerged bodies)

Consider a floating body as shown in figure(1) be tilted by a small heel angle ‘θ‘

by an overturning moment in clockwise direction. Now the buoyant force is shifted to new

Figure(1) Figure(2)

position B2 which is acting upward. This force forms a couple weight = w GMSinθ

anticlockwise direction. This couple restore the body to its original position. So this

condition is called stable equilibrium condition.

Stable Equilibrium condition: A floating body is said to be in stable equilibrium

when a small angular displacement of the body sets up a couple with its weight and

buoyuancy in tilted position, that tends to oppose the displacement and brings back to its

original position. Thus it may be stated that a floating body, if the metacentre lies above

its centre of Gravity G, is BM > BG will be in stable equilibrium condition. In this

condition the couple formed by the weight of body and buoyancy in new position will be

in opposite direction to the angular displacement and tends to bring back to its original

position. Here it may be noted that more portion of the body is to be submerged.

Unstable equilibrium condition:

Consider a floating body as shown in figure, be tilted through a small heel angle

‘θ‘ by a overturning moment. Here the couple formed by the weight of the body and

buoyant force in new position will be in the same direction of the overturning moment.

Page 43: Fluid Mechanics and Hydraulic Machinery

E310/1 43

Therefore the body will not be brought back to original position. This condition is called

as unstable equilibrium condition. Thus it may be stated that a floating body, if the

metacentre lies below the centre of Gravity G, ie BM < BG , will be in unstable

equilibrium condition.

Neutral equilibrium condition:

However if the metacentre coincides with the Centre of Gravity is BM = BG ,

then the floating body is said to be in Neutral equilibrium condition and it will occupy new

position, because the restoring couple formed by weight and buoyancy in new position, is

zero(GM = 0) .

Examples of forces causing overturining moments are wind forces, wave action, pressure

due to tidal or river current, shifting α(oa) in ship from are place to another.

So the ship is be to designed such that the metacentre is always well above the

Centre of Gravity, by lowering the centre of gravity by adding weight(by loading the ship

with ballast)

3.5 Determination of metacentric height

(a)Experimental determination:

Page 44: Fluid Mechanics and Hydraulic Machinery

E310/1 44

Consider a ship floating in the water as shown in figure. A plump bob is suspended from

P on the centre line of ship, ‘l’ above scale.

Let the weight of ship =W and weight of moving load = w(Magnified)

Now the movable load is shifted to a new position by a distance ‘x’ from the centre

of ship. This causes an over turning moment about ‘Μ’ = wx, in clockwise direction

through a heel angle ‘θ’. The plumb bob also shifted on the scale by ‘d’ giving tan θ =l/d,

where ‘θ’ is equal to heel angle.

In the new position, the buoyancy is shifted to B2 and the new centre of gravity is

G2 . The tilt of ship causes a restoring couple in anticlockwise direction =W GM tan θ.

∴ we have

WGM tanθ = wx or

GM =wx

w tanθ=

wxlwd

Page 45: Fluid Mechanics and Hydraulic Machinery

E310/1 45 Analytical method to determine metacentric height:

Consider the ship floating in the water as shown in figure be tilted by a small heel

angle ‘θ’. Due to this tilt, AOB wedge emerges from water causing -ve buoyant force

while COD wedge sinks into water causing +ve buoyant force which is equal to AOB

wedge. This causes an overturning moment as out ‘Μ’ due to the shift to Buoyancy B1 to

new position B2. To determine the overturning moment consider an elemental volume of

dx width at ‘x’ from the centre of ship.

Weight this elemental volume = wdx × xθ × L

Where L is the length of ship

= wθLxdx

The overturning couple of the elemental volume due to Buoyancy

= (wθLxdx )2x

Now by integrating this, the overturning couple is obtained as = ∫ wθLxdx 2x

= wθ∫ 2Lx2 dx = wθ ∫2(Ldx)x2

The integral value ∫2(Ldx)x2 is the double moment of the area(Ldx) which is equal to

moment of Intertia Ixx of the ship about its centre of gravity.

∴ overturning couple = wθIxx

Due to the tilt the restoring couple due to shift of buoyancy from B1 of B2 is

W BΜ tanθ (about M) = W × B1 B2. Note the weight of ship is equal to Buoyant force

and further as there is no change in volume of water displaced, the buoyant force at B1 and

Page 46: Fluid Mechanics and Hydraulic Machinery

E310/1 46 B2 are equal. This moment is the moment of the upward force F B1at B1 about Μ. This is

same as the moment of Buoyant force F B2 about B1 which oppose the overturning

moment.

∴ WBM tanθ = wθIxx as θ is small tanθ = θ

∴ BΜ = wIxx/W

But we know W = FB = ωv , Where v is volume of water displaced.

∴ BΜ = /wIxx

/wv=

Ixx

V

Metacentric height = GΜ= BΜ - BG

SAQ(7) What is the condition for stable equilibrium

SAQ(8) Loading the ship with ballast leads to what type of condition and why?

SAQ(9) A rectangular block 2m long 1m wide and 1m deep sink, into water by

0.6m deep. Find the weight of block.

SAQ10) Determine the distance between centre of Buoyancy to Meta Centre of a

wooden block 12 m long, 1m wide and 1m deep if it sinks by 0.6m.

Worked Example(3) A cylinder of dia 30cm having specific gravidy 0.8 floats in water.

What is the maximum permissible height of cylinder so that it can float in stable

equilibrium with its axis vertical.

Sol:

Volume of water displaced = V = πd 2

4× h1

we know

Page 47: Fluid Mechanics and Hydraulic Machinery

E310/1 47

BM = I / V; Ixx =πd 4

64

=πd 4

64×

1πd 2

4× h1

=d 2

16h1

But weight of body = Buoyant force ie

πd 2

4h × sw = πd 2

4h1 × w

= sh = h1

substituting this in above equation we have

BM =d2

16sh

From figure we know that OB = h1 / 2 = hs / 2

and OM − OB = BM

is OM = BM + OB

=d 2

16sh+

s2

for stable equilibrium, M should be at higher level than G,

so OM>OG = h/2 d 2

16sh+

s2

> h/2

or h2−

hs2

<d 2

16sh

h2(1-s) < 2d2/16s

∴ h < 2d2

16sx 1

(1 − s)

< 2 × 302

16 × 0.8−

1(1− 0..8)

= 26.52 cm.

Worked Example(4) A cone floating in water with its apex downwards has a diameter (d)

and vertical height ‘h’. If the specific gravity of the cone is ‘s’ prove that for stable

equilibrium.

h 2 <14

(d 2s

13

(1 − s13

) and Sec2θ > h/y

Page 48: Fluid Mechanics and Hydraulic Machinery

E310/1 48 where y is the depth of submergence

Solution:-

Let the depth of immersion = y

W weight of cone = ws(13Πd2

4h)

FB = Buoyant force = w(13Πd 2

4y)

we know that W = FB

is ws(13Πd2

4h) = w(

13Πd 2

4y)

d2 hs = d12y (1)

also tanθ/2 = d1

21y

=d2

1h

∴ d1 /d = y/h (2)

By (1) and (2) we have

(d1

d)2 = (

yh

)2 hsy

is y3 = sh3

y = hs1/3

OG = C.G of cone from apex = (3/4)h

OB = Centre of Buoyuancy from apex = (3/4)y

BG = OG − OB =34

(h − y) =34

(h − hs13 ) =

34

h(1 − s13 )

Now BM =I

VI =

Πd14

64; V=

13Πd2

4y

∴ BM =Πd1

4

641

13

Πd12

4y

=3d1

2

16y=

3d12 y

16h 2

∴ BM =3d 2hs

13

16h2 =3d2s

13

16h

For stable equilibrium BM > BG

3d2 (s13 )

16h>

34

h((1− s13 )

Page 49: Fluid Mechanics and Hydraulic Machinery

E310/1 49

h 2 <14

d 2s13

(1 − s13 )

To find Sec2θ > h/y

d = 2h tanθ

d1 = 2y tanθ

BM =3

16d1

2

y=

316

z2 y2 tan2θ =34

y tan2θ

BG =34

(h − y)

For stable equilibrium

BM > BG

3/4 ytan2θ > 3/4(h-y)

y sec2 θ > h

∴ Sec2 θ > h/y

Worked Example (5) A rectangular pontoon 10m long 7m broad and 2.5 m deck weighs

70 tonnes. It carries on its upper deep a boiler of 5m diameter weighing 50 Tonnes. The

centre of gravity of the boiler and pontoon may be assumed to be at their centres of figure

and lie in the same vertical line. Find the meta centre height.

Solution:-

W = Total Weight = 70 + 50 = 120 Tonnes

Let the height of immersion = y m

FB = Then buoyant force = 7 x 10 x y x 1 = 70 y Tonnes

But W = FB

Page 50: Fluid Mechanics and Hydraulic Machinery

E310/1 50 120 = 70 y

y = 120/70 = 1.171 m

To find Centre Gravity of the combined figure

Let Centre Gravity of the combined figure about base be = OG

Then tasking momenty of the weight about base

120 x OG = 70 x (25/2)+50(2.5+2.5)

∴ OG = 2.81m

Centre of buoyancy above base = OB = y/2 = 1.171/2 = 0.855 m

∴ BG = OG − OB = 2.81− 0.855 = 1.955m

But BM =IV

=

112

×10 × 73

120= 2.38

QFB = V .w

120 = V ×1

V = 120/1 = 120m3

Meta Centric Height = MG = BM − BG = 2.38 - 1.955 = 0.427

Worked Example(6) A rectangular pontoon weighting 240 Tonnes has a length of 20m .

The centre of Gravity in 30cm above the centre of the cross section. The meta centric

height is 1.33m when the angle of heel is 10o. The free board is not to be more than 0.67m

when the pontoon is vertical. find the breadth and height of the pontoon if it is floating in

fresh water.

Solution:-

Volume of water displaced = W/w = 240/1 = 240m3

OG = Centre of Gravity above base = H/2 +0.3

OB = Centre of buoyancy above base = (H - 0.67)/2

Page 51: Fluid Mechanics and Hydraulic Machinery

E310/1 51 ∴ BG = OG − OB = (

H2

× 0.3) −(H − 0.67)

2= 0.635 m

BM =IV

=

112

× xB3 × 20

240=

B2

144

MG = BM − BG1.33 = ((B2 /144) - 0.635)

∴ B = 6.5 m

Volume of water displaced =

240 = B x (H - 0.67)20

240 = 6.5(H-0.67)20

H = 2.51 m

Worked Example (7) A battle ship weighs 130 x 103 KN. On filling the ship’s boats on

are side at a distance 10m, from centre, with water weighing 600KN the angle of

displacement of the plumb line is obtained as 2o16’. Determine the meta centric height.

Solution:-

Given w= 600 KN x = 10m

W = 130 x 103 KN

tanθ = d/l = tan 2.27o = 0.04

GM =wx

W tanθ=

600130 × 103 ×

10.04

= 1.15m

Worked Example (8) A cone of wood floats in a fluid of specific gravity 0.9 with its apex

downwards. If the specific gravity of wood is 0.6 and weight of it is 300N. find the

weight of steel .If specific gravity 7.6 which should be suspended with the help of a chain ,

so that it can be submerged.

Solution:-

Page 52: Fluid Mechanics and Hydraulic Machinery

E310/1 52 V = Volume of cone = W/w

= 300/(0.6x9810) = 0.0051m3

FB = Buoyant force

= Volume of water displaced = w x V

=0.9 x 9.810 x 0.051 = 0.45KN

Downward force = upward force

W+ T = FB

∴ Tension in volume

= FB - W

=0.45KN - 0.3KN

=0.15KN

Again (T + Buoyant force over steel ) = weight of steel

But Buoyant force of steel

= Ws

ωsteel

w liquid

Volume of steel x wliquid

(Ws/7.6x9.81) x 0.9 x 9.81 = (Ws x 0.9)/7.6KN

∴ T +( Ws x 0.9)7.6 = Ws

Ws(1-(0.9/7.6)) = T = 0.15KN

∴ Ws = 0.17 KN

Summary

(1) Buoyant force is equal to weight of fluid displaced by the floating body and acts at

the centroid of the volume of fluid displaced.

FB = w V ; Where V = volume of fluid displaced

(2) Buoyant force is equal to the weight of floating body ie FB = W. The line of action

of these two forces will lie on the same to vertical line because these two forces

keep the body in equilibrium condition and so their moment about any axis is zero.

(3) The Metacentre :- When a floating body is tilted through a small heel angle θ, the

line of action of buoyant force through new buoyancy will meet the line external

through the centre of gravity and original buoyancy, at a point. This point is called

Metacentre and the distance of it from the centre of gravity of the body, is called

as metacentric height. is MG as θ →0.

Page 53: Fluid Mechanics and Hydraulic Machinery

E310/1 53 (4) The floating body will be in(a)stable equilibrium condition if M is above centre of

gravity G.

(b) unstable equilibrium condition ] if M is below G

and (c) Neutral equilibrium condition ] if M = G(M coincides with G)

(5) Metacentric height GM is determined by a keeping a wt ‘w’ at a distance u from

the centre of slip and knowing the heel angle ‘ ‘ by the plumb bob as

Gr = ωx/W tanθ ; where W = weight of ship

(6) Analytical method to determine the metacentric height is as follows .

BM (Distance of buoyancy to Metacentre) = I/V

Where

I = moment of intertia of the body about liquid surface

V = Volume of liquid displaced

Then Metacentric height = GM = BM − BG

BG is the distance between Buoyancy and centre of gravity of the body.

Answers to SAQ (1) True

(2) 98.1KN

(3) True

(4) Because specific weight is more for sea water

(5) 3.46 KN

(6) Stable condition

(7) 1200 Kg

(8) 0.14m

EXERCISE

3.1 A cylinder has a diameter of 0.3m and a sp.gr of 0.75. What is the maximum

permissible length in order that it may float in water with its axis vertical. Obtain

Page 54: Fluid Mechanics and Hydraulic Machinery

E310/1 54 an expression for the length in terms of diameter ‘d’, sp.gr of cylinder s and sp.gr

of liquid

[0.245m,

dso

8s(so − s) ].

3.2 A conical buoy floating with its apex downward in 3.5m high and 2m diameter.

Calculate its weight if it is just stable when floating in sea water weighing

10055N/m3.[29.127KN].

3.3 A solid concial float of wood weighing 7.5537N/m3 is to float in a liquid weighing

9417.6N/m3. Find the least apex angle in order that it may float with apex down

and its axis vertical [30050’ ]

3.4 A buoy carrying a beacon light has the upper portion cylindrical 2.5m diameter

and 2m deep. The lower portion which is curved displaces a volume of 0.4m3 and

its centre of buoyancy is 2.5m below the top of the cylinder. The C.G of the whole

buoy and beacon in situated 1.5m blow the top of the cylinder and the total

displacement is 19.62KN. Find the matacentric height of the sp. weight 9 sea

water is 10055N/m3.[0.505m]

3.5 A rectangular pantoon weighing 1716.75KN has a length of 20m. The Centre of

Gravity is 0.3m above the centre of cross section and the metacentric height is to

be 1.25m when the angle of heel is 90 .The free board must not be less than o.6m.

when the portion is vertical. Find the breadth & the height of the portion if it is

floating in fresh water .[5.792m, 2.11m].

3.6 A ship displace s 4150 Tonnes when floating in a sea water with its axis vertical

and a weight of 410 tonnes is placed on the centre line. Moving this weight by 1m

towards the hole of the deck causes a plumb hoh suspended from a 2.75m long

string, to move by 30cm. Find the metacentric height of the unloaded, ship.

3.7 A floating buoy in the sea water is floating upright by a submerged weight of

concrete attached to the bottom of the buoy. How many m3 of concrete weighing

22759 N/m3 must be provided to get a net downward pull of 2217N from the

weight., [0.1745m3].

3.8 A hollow cylindrical vessel of diameter 2m and 3m high weighs 29.43KN and its

C.C1 is at the mid point of the longitudinal axis. Shown that it will not float in sea

water with its longitudinal axis vertical. Find to what depth the inside of the buoy

is to be filled by concrete of sp.wt 22.563KN/m3 to give a depth of immersion of

Page 55: Fluid Mechanics and Hydraulic Machinery

E310/1 55 b2.5m and what is then the meta-centric height sea water weighs 10.055 KN/m3.

[0.6984m, 0.572m].

3.9 A rectangular pontoon 6mm by 3m plan floating in water has a depth of

immersion of 0.9m and is subjected to a torque of 7848N.m about longitudinal

axis. If the centre of gravity is 0.7m up from the bottom, find the angle of heel

[4050’]

References

Same as unit(1).

***

FLUID MECHANICS AND HYDRAULIC MACHINERY

UNIT-IV

KINEMATICS

Aim: To Know about the flow concepts like velocity, accelerations both tangential and

normal, potential theory of fluid flow in which the velocity potential and stream

functions. Finally to know about flow net and practical applications of the above

theory in solving the problems.

Objectives:

(1) Velocity of fluid in explained

(2) To explain convective & local acceleration and to arrive expressions for the same

(3) Then to obtain expressions for normal and tangential accelerations

(4) To define velocity potential and stream functions and to explain their uses in fluid

Mechanics

(5) To explain equipotential lines and stream lines, then flow net and its uses in fluid

mechanics and irrigation.

4.1 Introduction:

Types of flow, stream line, path line and streak lines are known already.

Now the acceleration is defined and the expressions for total acceleration is

arrived. This is necessary in further studies in deriving Euler’s and energy

equations.

Page 56: Fluid Mechanics and Hydraulic Machinery

E310/1 56 Velocity potential and stream functions are introduced which are useful in

describing the kinematics of fluid flow.

Based on these functions flow net is explained which is vary useful in solving

many problems in irrigation and potential theory.

4.2 Velocity of fluid

Consider the fluid flow occupied by the space (x,y,z) at a point as shown in figure.

Let the particle moves in time ‘dt’ sec through a distance ‘ds’ in the space. Then

the velocity of the fluid in the flow field is

V= L

t dsdt

as dt → 0= V(x,y,z,t)

If it is resolved in x,y,α,z co-ordinates,

Then velocity in x direction =ϑ=Lt

dxdt

as dt → 0= u(x,y,z,t)

Velocity in y direction = v = Lt as dt → 0dydt

= v(x,y,z,t)

and velocity in z direction = w = Lt as dt → 0dzdt

= w(x,y,z,t)

Page 57: Fluid Mechanics and Hydraulic Machinery

E310/1 57 Thus V = iu+jv+kw

Where i,j,K are unit vectors parallel to x,y,z axes respectively.

4.3 Acceleration in fluid flow

Acceleration in fluid flow is defined as the rate of change of velocity of the fluid.

is a = Lt

dvdt

as dt → 0This acceleration can also be resolved into the 3 components

ax,ayaz in x,y,z directions respectively as given below.

ax = Lt as dt → 0du(x, y, z,t)

dt= Total derivative of u

= ∂u∂x

dxdt

+∂x∂y

dydt

+∂u∂z

dzdt

+∂y∂t

u∂u∂x

+ v∂u∂y

+ w∂u∂z

+∂y∂t

(Connective accl.) (local acceleration)

In the above expression the sum of 3 terms (u∂u∂x

+ v∂u∂y

+ w∂u∂z

)indicates the

change in velocity taking place in space and so it is called as connective

acceleration. And ∂u∂t

is called as local acceleration as the change occurs at a

particular point in ‘dt’ seconds.

Similarly the accelerations in y and z directions are given by

ay = Lt as dt → 0dvdt

=u∂u∂x

+ v∂u∂y

+ w∂u∂z

+∂u∂t

and az = Lt as dt → 0dwdt

= u∂w∂x

+ v∂w∂y

+ w∂w∂z

+∂w∂t

The above expressions are arrived for unsteady non uniform flows.

∴ Total acceleration = connective acceleration + local acceleration

4.3 Tangential and normal acceleration

Consider an unsteady and non uniform flow of fluid along a curved stream lines of

radius ‘r’ as shown in figure.

Page 58: Fluid Mechanics and Hydraulic Machinery

E310/1 58

At A let the velocity be V and in

‘dt’ seconds after travelling ‘ds’ distance the fluid velocity is V+dv. As shown in figure at

point B, the magnitude and direction of ‘dv’ is obtained graphically by deducting of ‘V’’

from V+dv vertically. It is than resolved along tangential and normal directions to the

stream line, as dvs and dvm respectively.

Further we know that the tangential and normal velocities can be represented as vs(s,n,t)

and vn (s,n,t,) and then the total accelerations along tangential and normal directions are

represented by

Tangential acceleration = as =Lt

dvs

dtdt → 0

=∂vs

∂sdsdt

+∂vs

∂ndndt

+∂vs

dt

as =Vs∂vs

∂s+ vn

∂vs

∂n+

∂vs

∂tNote Vs =V along the stream line.

Similarly , the normal acceleration an is represented by

an =Lt

dvn

dt=

∂vn

∂sdsdt

+∂vn

∂ndndt

+∂vn

∂tdt → 0

=vs

∂vn

∂s+ vn

∂vn

∂n+

∂vn

∂tAccording to the definition of stream line, the flow will take place along stream

line only, but perpendicular to stream line, the velocity is zero. as there is no flow in that

direction.

Page 59: Fluid Mechanics and Hydraulic Machinery

E310/1 59 ∴υn = 0

Thus as = ϑ s

∂ϑ s

∂s+

∂ϑ s

dt

and an =ϑ s

∂ϑ n

∂s+

∂ϑn

∂t

It may be noted here that though ϑn =0, dϑn is not equal to zero as it is the

component of ‘dv’ in normal direction.

If the flow is stready then

as = ϑ s

∂ϑ s

∂sand an = ϑ s

∂ϑ n

∂s

Now from the figure we have, from the triangle BCD, tanθ =θdϑ n

v(dvs is

neglected as it is small)

For he curve with radius of curvature ‘υ’ . ds = υθ is θ= ds/υ

∴ equating both expressions

dϑ n

ϑ s

=dsυ

dϑ n

∂s=

ϑ s

rSubstituting this expression in normal acceleration An, we have

an =ϑ s

∂ϑ n

∂s= ϑ s

ϑ s

r=

Vs2

r=

V 2

r

and as = V dvds

Q Vs = V

SAQ (1) In steady flow the velocity along a curved stream line of radius of curvature 3m,

is 3m/s and it changes over 1m distance of the stream line into 3.2 m/s. Find the

tangential and normal accelerations.

SAQ(2) In a converging cone a fluid is flowing with a velocity of 1m/s at inlet section and

3m/s at out let section under steady condition, over a distance of 2m. Determine

the tangential velocity.

SAQ(3) For steady uniform flow, the acceleration is how much?

SAQ(4) There is no flow perpendicular to stream line . True/False.

Worked Example (1).

Page 60: Fluid Mechanics and Hydraulic Machinery

E310/1 60 A sheet of water is flowing over a curved bucket with a radius of curvature of 10 m with a

velocity of 30m/s

Find the normal acceleration

an =v 2

r=

30 × 3010

= 90m / s 2

Worked Example (2) A converging cone is changing in diameter from 2m to 1m over a

distance of 2m. The discharge changes from 15m3/s at t = θ sec., to 75 m3/s at t= 3

seconds. Find the total acceleration at t = 0 at a section where the diameter is1.5 m.

Solution

We know that dia at a distance x is

dx = 2-(2-1)x/2

= 2-0.5 x

Ax =Π(2 − 0.5x)2

4

Vx =QAx

=15

Π4

(2 − 0.5x)2=

60Π(2 − 0.5x)2

dVx

dx=

60Π

(−2)(2 − 0.5x)−3 (−0.5)

VX dVx

dx=

60Π(2 − 0.5x)3 ×

60Π(2 − 0.5x)

=364.7

(2 − 0.5x)5

Local acceleration = dVx

dt=

1A

dQdt

=60

Π(2 − 0.5x)2 ×(75 −15)

3=

25.5(2 − 0.5x)2

when t = 0, total acceleration as

as =Vx dvx

dx+

dvx

dt

Page 61: Fluid Mechanics and Hydraulic Machinery

E310/1 61 =

364.7(2 − 0.5x)5 +

25.5(2 − 0.5x)2

Given dx =1.5m

∴1.5 = (2-0.5x)

x=1m

at t=0

as =364.7

(2 − 0.5 ×1)5 +25.5

(2 − 0.5 ×1)2

=47.99+11.33 = 59.32 m/s2

Worked Example (3)

In two dimensional flow the velocity components are as follows

u = 2xy+3t2 +6

ϑ=-2xy2-15t

Find the acceleration at a point (1,2) when t = 2 seconds

Solution:

ax = u∂u∂x

+ϑ∂u∂y

+∂u∂t

ax =(2xy+3t2+6)(2y)-(2xy2 +15t)(2x)+6t

Given x = 1, y = 2, t = 2

ax = (4+12+6)(4)-(8+30)(2)+12=-20m/s2

ay =u∂u∂x

+ϑ∂u∂y

+∂ϑ∂t

=(2xy+3t2 +6)(-2y)2 -(2xy2+15t)(-4xy)-15

=22(-8)-38(-8)-15

=-495m/s2

a= ax2 + ay

2

202 + (495)2

=495.4 m/s2

Page 62: Fluid Mechanics and Hydraulic Machinery

E310/1 62

θ = tan −1 ayax

= tan −1 1 − 495

20

θ = tan −1 (−24.75)= −870.68

4.4 Stream function and velocity potential function

For 2 dimensional flow a stream function is defined as a continuous function of the

space (x,y) such that the partial derivative of it with respective to any direction gives

velocity component 900 to that direction in anticlockwise direction.

ie u = −∂ψ∂y

ϑ = +∂ψ∂υ

Its value is constant along a stream line. ie when stream function is equated to a series of

constants, it yields equations of series of stream lines.

Along a stream line

ψ = constant

Qϑ = 0 =

+∂ψ∂x

= 0

∂ψ = 0

ie ψ = constant along stream line)

ie dψ =∂ψ∂x

dx +∂ψ∂y

− dy = 0 (1)

along a stream line

Page 63: Fluid Mechanics and Hydraulic Machinery

E310/1 63 But equation of stream line, we know as

ϑ/u = dy/dx ie ϑdx-υdy=0 (2)

Compare (1) and (2) , we have

ϑ=∂ψ∂x

and u =-∂ψ∂y

shows that ψ is constant along a stream line.

Further consider the discharge flowing through a stream tube with ψ , and ψ2 as boundary

stream lines, as shown in figure. Now consider the discharge flowing across the curve

ACB is given by

dQ = -udy+ϑdx = dψ (here flow is taken +ve in anticlockwise.)

∴ charge in the stream function is equal to the discharge flowing between the two stream

lines for which ψ 1, and ψ2 are the stream functions passing through the points A&B

respectively.

Consider continuity equation as given below for stready incompressible and continuous

fluid

∂u∂x

+∂ϑ∂y

= 0

Substituting the expressions for uαϑ in the above equation,

−∂ 2ψ∂x∂y

+∂ 2ψdy∂x

= 0

ie −∂ 2ψ∂x∂y

+∂ 2ψ∂y∂x

Page 64: Fluid Mechanics and Hydraulic Machinery

E310/1 64 This is true if ψ is continuous . So it can be stated that when fluid flow is continuous, ψ

also continuous which may be irrotational or rotational. So if it is be irrotational, we have

rotation about ‘2’ axis w2

w2 =12

∂ϑ∂x

−∂u∂y

= 0

ie ∂ϑ∂u

−∂u∂y

= 0

Substituting the expression for u ∂ ϑ we have

∂ 2ψ∂x2 +

∂ 2ψ∂y2 = 0

This is known a Laplace equation in ψ. So it replace equation in ψ is satisfied than the

fluid flow is irrotational, otherwise it is rotational only. So ψ exists both for rotational

and irrotational fluid flows.

Velocity potential ‘φ’ is defined in 2D flow as a continuous function of space (x,y) such

that its -ve a have partial derivative gives velocity in that direction.

ie −∂φ∂x

= u;-∂φ∂y

= ϑ

Substituting these in continuity equation for steady incompressible fluid flow we

have

∂u∂x

+dϑ∂y

= 0

∂ 2φ∂x 2 +

∂ 2φ∂y2 = 0

Which is Laplace equation in φ. So if Laplace equation in φ is satisfied the fluid is flow is

continuous and so ‘φ is continuous function. Further substituting the above expressions

for u & ϑ in condition for rationality we have

∂υ∂x

−∂υ∂y

= 0

ie -∂ 2φ∂x∂y

+∂ 2φ∂y∂x

= 0

Page 65: Fluid Mechanics and Hydraulic Machinery

E310/1 65

ie ∂ 2φ∂x∂y

=∂ 2φ∂y∂x

is true only and is continuous . That means, if φ exists it is

continuous, then it should represent only irrotational fluid flow. So φ exists only for

irrotational fluid flow, ie for ideal fluid flow. So ideal fluid flow theory is also called as

potential flow theory.

If the equation for velocity potential is equated to series of constant we have series

of equipotential lines, for which the velocity potential function is constant.

SAQ (5) Define ψ and φ functions.

SAQ(6) φ exists only for irrotational flow? True / False

SAQ(7) Irrotational flow is called potential fluid flow. True / False

SAQ (8) ψ = 2xy, find the discharge through the points (1,1) and (1,2)

SAQ(9) ψ = 3x2 -y3 , find u at (1,2)

SAQ(10) φ = x2-y2 find u at (1,2)

SAQ(11) If ψ exists it may be either rotational or irrotational. True / False

Worked Example (4)

Given φ = x2 -y2 , find the velocity at a point (1,2) and its direction.

u =−∂φ∂x

= -(2x)=-2×1=-2 units

ϑ = −∂φ∂y

=-(-2y)=2y=2×2=4 units

V = u 2 +ϑ 2 − 4 +16 = 4.47m / s

Direction with horizontal = θ =tan −1 ϑu

= tan −1 y

−z

= tan −1 −z( )

Worked Example 5

Given φ = x2 -y2 , find ψ and discharge through (1,1) and (1,2) point

Sol:

Page 66: Fluid Mechanics and Hydraulic Machinery

E310/1 66

−∂φ∂x

= u =−∂ψ∂y

ie ∂φ∂x

=∂ψ∂y

−∂φ∂y

= ϑ =∂ψ∂x

∂φ∂y

=−∂ψ∂x

φ = x2 -y2

∴∂φ∂x

= 2x =∂ψ∂y

Integrating w.r.to y, we have

ψ = 2x dy∫ = 2xy+const

=2xy+f(x)

∴ψ = 2xy+f(x)

Now differentiating ψ w.r.to x,

∂ψ∂x

= 2y+ f x( )( )

dx= −

∂φ∂y

)=-(2y)=2y

∴d(t(x)

dx= 0

f(x) = const

∴ψ = 2xy+constant

Now ψ 1 = 2 ×1 × 1 = 4

(1,1)

ψ 2(1, 2)= 2 ×1 × 2 = 4

∴dψ =dφ=ψ2 -ψ1 =4-2=2units

Worked Example (6)

Verify whether ψ = 2x2y2 is a possible irrotational flow . If so find velocity at (1,2).

Solution:

For flow to be continuous

∂ 2ψ =∂ 2ψ∂x ∂y

=∂ 2ϑ∂y∂x

∂ψ∂x

= 4xy 2 ∂ψ∂y∂x

= 8xy

∂ψ∂y

= 4x 2 y∂ 2ψ∂x∂y

=8xy

∴It is continuous and so ψ represents a possible flow.

Page 67: Fluid Mechanics and Hydraulic Machinery

E310/1 67 Now for irrotational flow Laplace.

equation in ψ to be satisfied.

ie ∂ 2ψ∂x2 +

∂ 2ψ∂y2 = 0

∂ 2ψ∂x2 = 4y2 ∂ 2ψ

∂y2 = 4x 2

∴∠p is ψ ± 0

So it is rotational flow only.

Vol at (1,2)

u =−∂ψ∂y

= −(4x 2 y)

=-(4×1×2)=-8 units

ϑ =∂ψ∂x

= 4xy2 = 4 × 1× 22 =16

V = u2 + ϑ 2 = (−8)2 + 162 =16.5 units.

Flow net

we know that

∂ψ∂s

= ϑ n = 0

Qψ is constant along ‘s’ direction

that means, there is no flow normal to stream line.

Also ∂ψ∂u

= −Vs

Similarly −∂ψ∂u

= ϑ n = 0

as φ is constant along u direction

−∂φ∂s

= ϑ s

Thus Ψ = different constants, gives a series of stream lines

and φ = different constants gives a series of equipotential lines, as already explained

Page 68: Fluid Mechanics and Hydraulic Machinery

E310/1 68

. Now let us consider the slope of these lines. The slope of φ lines is

= dydx

=∂φ∂φ

×dydx

=∂φ∂s

oφ oy =

−u−ϑ

=uϑ

Similarly the slope ofψ lines

dydx

=∂ψ∂ψ

×dydx

=∂ψ∂x

∂ψ ∂y =

ϑ−u

∴(slope of φ lines) ×(slope of ψ lines)

=uϑ

×−ϑu

= −1

This shows that the φ lines and ψ lines meet each other orthoganally at any point, giving

rise to square mesh as shown in figure.

This grid of stream lines and equipotential lines is called as flownet. Thus a

flownet is defined as a grid of stream lines and equipotential lines which are orthogonal

to each other at every point of intersection. It is applicable to ideal fluid flow. But except

in Boundary layer, it can be applied even to real fluid flows.

Uses of flownet

1. If a flow net is known for a flow field, then the velocity distribution over the entire

flow fields can be determined as follows.

Page 69: Fluid Mechanics and Hydraulic Machinery

E310/1 69

Between any two stream lines the discharge flowing perunit thickness of flow is known

as dψ=ψ2 -ψ1 etc. The mesh size at any two points are known as ‘du’ and du2 . Then

ϑ 1 =dψ

du1 ×1( )&ϑ 2 =dψ

du2 ×1( ) are known

In this away at any point the velocity is known in the entire fluid field for which the flow

net is known.

2 . Then by application of Bernoulli’s equation, the pressure at any point can be calculated

in the flow field.

2. Using flow net theory, the seepage through earthen dams and foundations can be

calculated as shown in figure. This is useful in the design of earthen dams.

3. The uplift pressures and exit pressure gradient downstream of irrigation structures like

weirs, barrages etch can be known. These are very useful in the design of the irrigation

structures, as shown in figure.

SAQ(12) Define flow net.

SAQ(13) The mesh size at a point in flow net is 1m where the velocity is 3m/s. Find

the velocity at another point in the flow net where the mesh size in 1/2 m.

SAQ (14) Mesh size is more, the velocity is less. Is it true? True/False.

Page 70: Fluid Mechanics and Hydraulic Machinery

E310/1 70 Summary

1. Expression for total acceleration in direction is written as

Lt

DυDt

=u∂u∂x

+ϑ∂u∂y

+ w∂u∂z

a1 24444 34444

curvature acceleration

+∂u∂tb

local acceleration

dsdt → 0

Similarly in other directions, y& Z the expression for

DvDt

&DwDt

can be written.

2. For steady flow, the tangential and normal accelerations are given by

as =vdvds

(tangential acceleration)

an = v2/r (normal acceleration)

where ‘r’ is the radius of curvature of the stream line and V is the Velocity of

flow .along the stream line.

3. Stream function ‘ψ’ is defined as function of space (x,y for 2D flow, such that

its partial derivative gives the velocity in 900 to its direction in anticlockwise

direction.

∂ψ∂x

= ϑ -∂ψ∂y

= u

4. Velocity potential φ is defined as a function of space such that its -ve partial

derivative in any direction given velocity in that direction.

ie −∂φ∂x

= u -∂φ∂y

= ϑ

5. Velocity potential exists for irrotational flow only

6. Flow net is a grid of stream lines and equipotential lines such that they meet

orthogonally at any point of intersection giving rise so square mesh.

ie slope of φ lines × slope of φ ψ lines = -1

7. Using flow net velocity distribution, pressure over a flow fluid can be determined.

It can also be used to determine seepage flow through earthen dam through

foundations, uplift pressures below irrigation structures and exit pressure

gradient.

Answers for SAQ

Page 71: Fluid Mechanics and Hydraulic Machinery

E310/1 71 1.as =0.6m/s2 an =3m/s2

2.as = 1m/s2 an=3m/s2

3.ax =0 4. True 6. True 7.True

8.2 units 9.6m/s 10.-2m/s

11.True 13.12m/s 14.True

EXERCISE

4.1 The velocity along the centre line of nozzle of length L s given by v=4 t(1−x

2L)2

Where ‘v’ is the velocity in m/s , t = time in seconds, x = distance from inlet. Find

total acceleration when t=3s,x=0.5m,L=0.8m. [233.023m/s2].

4.2 A 90cm diameter pipe is reduced to 30cm diameter in a length of 1.5m. Water flows

through this pipe at a rate of 280 lps. If at an instant the discharge is found

to decrease at the rate of 60 lps estimate the total acceleration at a distance of

75cm from the 90cm diameter inlet.. [1.0953 m/s2].

4.3 In a steady flow two points A and B are 0.5m apart on a straight stream line of the

velocity of flow varies usually between A & B, what is the acceleration at each

point if the velocity at A is 2m/s and the velocity bat B is 6m/s.

[at A=16m/s2 , at B=48m/s2]

4.4 Check whether the following sets of velocity components satisfy the continuity

equation of steady incompressible flow.

(a) u=4x+2y-3, v=2x+4y+3

(b) u=4xy+y2, v=6xy+3x+2\

(c) u=2x2+y2, v=-4xy

(d) u=x3+y3, v=x-3x2y

(e) u=c(y2 − x 2 )(x 2 + y 2 )2 , , v=ϑ =

−2cxy(x 2 + y2 )2

(f) u=(3x-y), v=(2x+3y)

(g) u=A Sin xy v=-A Sin xy

[not satisfied -a,b,c,f,d ; satisfies -c,d]

4.5 Calculate the missing utilising velocity component, so that they satisfy the

continuity equation

Page 72: Fluid Mechanics and Hydraulic Machinery

E310/1 72 (a) u=2/3xy3-x2y ; (b)u=Ayex

v=? v=?

(c) u=2x2,v=xy2, w=1

(d) u=(2x2+2xy), v=? , w=(x2 -4xz-2yz)?

(e) u = 3x2 , ϑ=-x2y-yz-xy, w-?

(f) u=x3+y2+2z2, ϑ=-x2y-yz-xy, w=?

[(a) =v-1/6y4+xy2, (b) ϑ=-−Ay 2

2ex + f (x)]

(c) w=-4xz-xz2+f(x,y)

(d) -3yz2+f(x,z) (e) w=-(6xz+2xz2)+f(x,y)

(f) w=2x2z-z22-xz+f(x,y)

4.6 Determine which of the following velocity fields represent possible example of

irrotational flow.

(a) u=cx, v=-cy

(b) u=-cx/y, v=c logxy

(c) u=(Ax2-Bxy), v=(-2Axy+1/2By2)

[(a) irrotational,(b)(c) rotational]

4.7 Verify whether the following flow fields are rotational. If so determine the

components of rotation about verious axes

(a) u=xy2, v=zx, w=1/2yz2-xy

(b) u=xy, v=1/2(x2-y2)

[(a) Rotational w2 = 1/2 z(1-x)

wx=1/2(z2/2 -2x)

wy=1/2y(x+1)

(b) irrotational wx=wy=0

4.8 For the following sets of velocity components obtain the relevant stream

functions.

(a) u=6y, v=6x, (b) u=-Alnx, v=A(y/x)+x2

(c) u=y3/3+2x-x2y, v=xy2-2y-(x2/3)

[ (a) Ψ = 3(y2 − x 2 ) + c

(b) Ψ =y 2

12−

x 2 y 2

2+ 2xy +

x 3

9

Page 73: Fluid Mechanics and Hydraulic Machinery

E310/1 73

(c) Ψ =−Ay | nx

mx−

x 3

3+ c ]

4.9 Which of the following stream functions ψ are possible irrotiational flow fields.

(a) ψ=Ax2+By2 (b) ψ=Ax2y2

(c) ψ=A sinxy (d) ψ=Alog(x/y)

(e) ψ= Ay(1 −c

x2 y2 ) (f) ψ=(y2-x2)

[(a) & (d) - None, (e) & (f) are possible irrotational flows]

4.10 Verify whether the follwing functions are valid potential functions

(a) φ=y3-3x2y (b) φ=y4-6x2y2

(c) φ=x2-3x2y (d) φ=x3-y3

[ (a) yes., (b) (c) (d) - No ]

4.11 Determine the velocity potential functions for the following . Also determine the

discharge flowing between the stream lines passing through the points (1,3) and

(3,3)

(a) ψ=3xy (b) 3/2(y2-x2) (c) ψ= x2-y2

[ (a) φ=3/2(y2-x2) 24 units (b) φ=-3xy Zero (c) 8 units ]

4.12 Calculate the velocity at the point(3.3) for the following stream functions

(a) ψ=-xln x y + x (b) ψ=1/2(y2-x2)+xy+6

[ (a) v=2.42units (b) v=6 units ]

4.13 Determine the stream functions, ‘ψ‘ for the following velocity potential functions

φ. Calculate the velocity at (2,2).

(a) φ=x + y (b) tan-1(y/x) (c) φ=A x/ (x2+y2)

[ (a) ψ=y-x + constant (b) ψ=-1/2 log(x2+y2)+constant

v=√2 units v=1/√13 units

(c) ψ=-A y/((x2+y2)+constant)

v =A/√169 ]

***

Page 74: Fluid Mechanics and Hydraulic Machinery

E310/1 74 FLUID MECHANICS & HYDRAULIC MACHINERY

UNIT-5

DYNAMICS OF IDEAL FLUIDS

AIM:

To list the various forces causing the fluid flow, to identity the important fluid

forces viz. body forces and surfaces in deriving Euler’s equation of motion and to derive

Bernoulli’s equation of motion by integrating Euler’s equation. Finally to apply

Bernoulli’s equation in solving fluid flow problems.

OBJECTIVE:

1) The various forces causing the fluid flow are listed and name the two forces

namely, body forces and surface forces

2) To formulate the Euler’s equation

3) To derive the Euler’s equation along a stream line considering ideal flow.

4) To derive the Bernoulli’s equation by integrating the Euler’s equation along stream

line. Here the assumptions necessary to derive this equation, the modifications and

limitations are discussed .

5) Kinetic energy correction factor is derived.

6) To derive momentum equations based on Newton’s 2nd law of motion.

7) To apply the above equations in solving fluid problems.

5.1 INTRODUCTION

In dynamics of ideal fluid flow the viscosity is neglected and so the loss of energy

due to friction is zero. The various fluid forces are listed to derive Euler’s equation and

hence. Bernoulli’s equation based on Euler’s equation using Newton’s 2nd law of motion

The importance of Bernoulli’s equation is discussed by applying it to solve many fluid

problems. Along this equation continuity equation is also useful in solving the problems.

The third important equation in fluid mechanics is momentum equation. This is

also very useful in solving the fluid problems, When Bernoulli’s equation along with

continuity

Page 75: Fluid Mechanics and Hydraulic Machinery

E310/1 75 equation can alone not solve the fluid problems. This equation is very useful in Hydraulic

machinery.

5.2 DIFFERENT FORCES CAUSING THE FLOW:

The different forces causing the motion are listed below.

Fp Pressure force which is a function of fluid motion.

Fg Gravity or body force is due to weight of fluid flowing

Fo Viscous force is due to viscous shear action of fluid. It depends on

viscosity of fluid. So for ideal fluid it is zero.

Ft Turbulent force is due to the turbulence of fluid. In turbulent flows due to

intermixing of fluid particles, these forces are developed. It exists only for

real fluids and so are zero for ideal fluid flows.

Fe Elastic force is due to elastic compression of the fluid. This force is

involved only in compressible fluid flows.

Fσ Surface tension force is due to surface tension of the fluid, Which is

extremely fluid film at interfaces between two different fluids.

We know that by Newton’s 2nd law that the sum of all external forces acting as the

fluid is equal to the rate of change of momentum in the direction of forces. That means

∑F = Mass × acceleration.

In majority of flows surface tension forces, elastic forces are negligible and so neglected.

Thus Fp + Fg + Fϑ + Ft = ma , is known as Reynolds equation because turbulance of fluid

is involved in this equation. If this is neglected then.

Fp + Fg + Fϑ = ma is called as Navier-Stoke’s equation because viscous forces are

involved in this equation. The above two equations will refer to real fluid flows. If

viscous forces are neglected then we have

Fp + Fg = ma is known as Euler’s equation for ideal fluid flows where velocity is

zero.

Page 76: Fluid Mechanics and Hydraulic Machinery

E310/1 76 5.3 EULER’S EQUATION OF MOTION:

Now let us know about the two forces causing the fluid flow in derivation of

Euler’s equation for ideal flow.

Surface forces are due to (stress x surface area). Further these forces are

subdivided into normal forces called pressure forces and tangential forces, which are due

to viscous shear force. As viscosity is zero for ideal fluid, the tangential forces are zero

and so surface forces in ideal fluid flows, are only pressure forces which are normal forces

obtained by (pressure x C.S area of flow)

Body force is proportional to volume of body, which is due gravity i.e. weight of

fluid is considered as body force.

Now consider a small elemental fluid prisum of C.S area dA and length ‘d s’ in

fluid flow along a streamline as shown in figure for steady incompressible, ideal fluid

flow. As the flow

Fig 5.1

considered is ideal, mean velocity is considered over the cross section of fluid flow. The

particles move ‘ds’ distance in time dt seconds.

We have by Newton’s 2nd law that

∑ Fs = m × as along streamline

Fp + Fg = m × as

The sum of pressure force and body force along streamline = mass × acceleration along

streamline.

Fp = pdA - p +dbds

ds

dA =

−dpds

dsdA

Page 77: Fluid Mechanics and Hydraulic Machinery

E310/1 77 Fg = sPdAds

Where ‘S’ the component of body force along streamline

∴−dpds

dsdA - sρdAds = ρdAds × as

Now dividing the above equation throughout by mass of the fluid, we have −1ρ

dpds

- S = as

which is known as Euler’s equation along a streamline

Body force along ‘S’ direction is the weight component of weight of fluid along

streamline.

i.e. Fg = ρgdAds cos θ

but dzds

= cosθ

∴Fg = ρgd A.ds. dzds

= sρd A.ds

i.e. S = gdzds

Further for steady flow acceleration along stream line is

as =Vdvds

+Avdt

; Where dvdt

= 0 for steady flow ∴ as =Vdvds

where ‘v’ is the

velocity of fluid along steam line over the cross section, which is uniform so

as =d v 2 2( )

dssubstituting the above in the Euler’s equation we have

−1ρ

dpds

−gdzds

=d V 2 2( )

ds

i.e. 1ρ

dpds

+gdzds

+d V 2 2( )

ds= 0

Derivation of Bernoulli’s equation

Now integrate the above equation with respect to s, we have energy equation,

because Force × distance = work done

Thus 1ρ∫

dpds

ds +gdzds

ds +d V 2 2( )

ds∫∫ ds= constant

Page 78: Fluid Mechanics and Hydraulic Machinery

E310/1 78 i.e.

1ρ∫ dp + gdZ + d V 2 2( )∫∫ = C

As the flow is incompressible, ρ is constant along the flow, so it is taken out of integration

as

dp + gdZ +∫∫ d V 2 2( )∫ = C

i.e. pρ

+ gz + V2 2 = C

or pg

ZV

gC

ρ+ +

21

2

i.e. pw

ZV

g+ + =

2

2constant

This is called as Total head at a point on the stream line or total energy per unit weight of

fluid.

Total head = pressure head + datum head + velocity head

The above equation is called as Bernoulli’s equation which states that for steady

incompressible continuous fluid flow the total head is constant at any point along its flow.

This equation in the conservation of energy of fluid, as discussed below.

Consider pw

= pressure head

We know the work done by pressure is given by

p × a× v =pw

wav = pw

( weight / sec )

∴pw

is (W.D / sec ) per unit weight of fluid = pressure energy / unit weight = pressure

head = mkg/kg work done by elevation of the particles above a datum is given by potential

energy = Mass × g × Z.

Where Z is the elevation of the fluid particles above the datum W.D by elevation of fluid

= (ρAV) gZ

ω AVZ = W/ secxZ

∴Datum energy = W/sec xZ

ρPotential energy or Datum head = Datum energy / unit weight = Z m-kg/kg

Page 79: Fluid Mechanics and Hydraulic Machinery

E310/1 79 work done by movement of fluid is

Kinetic energy

= 12

mass × V2

=12

× ρAV × V2

=12

×wg

AV× V2=V 2

2g( w/sec) = m.kg/sec

or kinetic energy / unit weight = V 2

2gm.kg/kg

∴ + +

W

pw

ZV

g

W

2

2= constant

or Total energy / unit weight = constant

i.e. Total head = constant

i.e. pw

ZV

g+ +

2

2= constant which is called as Bernoulli’s equation.

ASSUMPTIONS

In the derivation of Bernoulli’s equation.

(1) Fluid is assumed to be ideal and friction less is µ = 0 and hL = 0.

(2) Fluid flow is steady flow , so that local acceleration is dVdt

= 0.

(3) Fluid is incompressible ρ is constant along its flow.

(4) Fluid flow is continuous, i.e mass rate flow is constant over the cross

section of flow.

(5) Fluid flow is considered as ideal and so mean velocity is considered over

the cross sectional flow. This mean velocity is constant over entire C.S. of

flow.

LIMITATIONS

(1) It is applicable to only ideal fluid only .

Page 80: Fluid Mechanics and Hydraulic Machinery

E310/1 80 (2) It is considered as ideal and so velocity over the C.S is considered as

constant which is not true in the actual fluid flow.

(3) It is applicable to steady flow only

(4) It is applicable to particles moving along a stream line, but total head is

varying from stream line to stream line.

(5) Energy added or subtracted from the control volume, is taken as zero.

MODIFICATIONS

(1) For real fluids there will be loss of energy due to fiction, which is to be

considered in the equation as

Z1 + + = + + +−

pw

Vg

Zpw

Vg

hL1 1

2

22 2

2

2 2 1 2where hL1−2

is the constant

of the energy from (1) to (2)

(2) Between two streamlines the centrifugal head added is to be considered.

(3) The energy added or subtracted is to be considered in the equation as

Z1 +pw

Vg

1 12

2+ + Hadded / unit weight = Z2+

pw

Vg

2 22

2+ (Pump)

Z1 +pw

Vg

1 12

2+ - Hadded / unit weight = Z2+

pw

Vg

2 22

2+ (Turbing)

(4) The velocity distribution over the c-s is not uniform. ∴ Kinetic energy

correction factors are to be considered in the equation.

Z1+pw

Vg

Zpw

Vg

hL1

112

22

22

2

2 2 1 2+ = + + +

−α α

where ‘α1’& ‘α2 ’ are called as energy correction factors

KINETIC ENERGY CORRECTION FACTOR

Page 81: Fluid Mechanics and Hydraulic Machinery

E310/1 81 In the derivation of Bernoulli’s equation mean velocity which is uniform over

entire c-s flow is considered. But in actual fluid flow , it is non uniform as shown in the

figure.

Fig 5.2

Thus kinetic energy calculated on the basis of mean velocity is less than the actual kinetic

energy. So a correction factor is introduced to get actual kinetic energy as αV 2

2gwhere ‘α‘

is the kinetic energy correction factor which is derived as shown below.

Kinetic energy based on

uniform velocity distribution = 12

( mass ) V2

=12

2wg

Avv

=12

3wA

Vg

The actual kinetic energy is calculated based on the actual velocity distribution over c-s

flow, is calculated as shown below

consider an elimental are dA on which the velocity of flow is v

∴dr = mass flowing through this elemental area = wg

dAv

K.E of this mass = 12

2wg

dAv v

×

And by integrating this over the complete c-s of flow, we have

Actual K.E = 12

12

2 3

A A

wg

dAv vg

v dA∫ ∫× =ω

.

Page 82: Fluid Mechanics and Hydraulic Machinery

E310/1 82 Thus the cube of mean velocity over the area is less than the cubes of instantaneous

velocity over the area. ∴ The kinetic energy of the flow based on mean velocity is to be

multiplied by a factor ‘α‘ to correct the K.E. This factor is known as kinetic energy

correction factor is given by

α = =∫ ∫

12

12

3

3

3

3

wg

v dA

wAv

S

v dA

AvA A

Here α >1.0

For turbulent flow α = 1.03 −1.06 and for laminar flow α = 2

In turbulent flow α can be taken as 1.0

SAQ(1) In Euler’s equation only pressure forces and body forces are considered in

steady flow condition.

True/False

SAQ(2) In I-D flow mean velocity over the C.S is considered

True/False

SAQ(3) In Euler’s equation the terms are per unit mass

True/False

SAQ(4) Bernoulli’s theorem studies the conservation of energy

True/False

SAQ(5) In Bernoulli’s theorem the terms are energises per unit weight of flow per

second

True/False

SAQ(6) Bernoulli’s theorem is applicable to ideal steady incompressible continuous

flow

True/False

SAQ(7) The unit of term is Bernoulli’s theorem is, m, N - m/N, Jouls/N

True/False.

SAQ(8) Bernoulli’s heads are also called as energies/unit weight

True/False.

SAQ(9) Is Bernoulli’s theorem applicable to rational flow along a stream line

Yes/No.

Page 83: Fluid Mechanics and Hydraulic Machinery

E310/1 83 SAQ(10) Is Bernoulli’s theorem applicable to real fluid flows

Yes/No.

SAQ(11) Why the flow in the derivation of Bernoulli’s theorem is to be steady ?

SAQ(12) Why flow is to be incompressible in derivation of Bernoulli’s theorem?

SAQ(13) Why the kinetic energy correction factor is greater than 1?

SAQ(14) Given the total head at a point is 10m of water and at another point the

piezometer head is 8m. Find the velocity at second point?

SAQ(15) If the total head at (1) is 10m and at (2) is 8m, Find the head loss and

direction of flow?

SAQ(16) If the total head at (1) is 10m of water. Find the pressure at another point in

kpa, Which is 2m above point (1) if the velocity head at this point is 1m?

SAQ(17) A point in a pipeline which is in the form of a syphon , is 2m above the

water level in a tank . The velocity head in the pipe is 1m of water. Find the

pressure at that point?

SAQ(18) Find the maximum elevation of the summit of a pipe line which is in the

form of a syphon, above water level in the tank, if the velocity head in the

pipe is 1m of water. Take vapour pressure as -8m of water?

WE(1) A Pipe line carrying oil of specific gravity 0.9 changes in diameter from 20

cm at A to 50cm diameter at B which is 4m above A. If the pressure at A and B are 100k

Page 84: Fluid Mechanics and Hydraulic Machinery

E310/1 84 pa and 60kpa respectively and the discharge flowing is 200 lps, determine direction of

flow and loss of energy.

SOLUTION

Fig 5.3

Velocity at A = ( )

2001000

40 2 2×

π .

6.34m/s ;VA

2

2g2.066m

Velocity at B = ( )

2001000

40 5 2×

π .= 1.0185m/s ;

VB2

2g=0.052

Now applying Bernoulli’s theorem between A and B taking datum through A, we have

Total head at A = ZA +Pw

Vg

A A+ = +××

+2

20

100 10000 9 9810.

2.066 = 13.4m of oil.

Total head at B = ZB +Pw

Vg

A A+ = +××

+2

24

60 10000 9 9810.

0.052 = 10.85m of oil.

Energy at A > Energy at B ∴Flow is from A to B and the loss of head.

= HA - HB

= 13.4 - 10.85

= 2.55m of oil.

WE(2) A conical tube 1.5m long is fixed vertically with upper end which is smaller in dia

in a pipe line. When a liquid of sp gr 0.9 flows down the tube at a of 294 l.p.s the velocity

at upper and lower ends are 4.5m/s and 1.5m/s respectively. The pressure head at upper

Page 85: Fluid Mechanics and Hydraulic Machinery

E310/1 85

end is 3m of liquid. The loss of head in the tube is 0.3 ( )ϑ ϑ1 2

2

2−

g, where ϑ1 and ϑ2 are

velocities in upper and lower ends respectively. Calculate the pressure head in Kpa at

lower end and diameters of the tube.

SOLUTION

Fig 5.4

Discharge = Q = 295

1000m/s

a1 =Qϑ1

=2951000

×1

4.5=0.0655m2

d1 = 28.9cm

a2 =Qϑ2

=295

1000×

11.5

= 0.1967m2

d2 = 50cm

Now apply Bernoulli’s theorem between A & B taking datum through B,

ZA +VA

2

2g+

PA

ω= ZB +

VB2

2g+

PB

ω+ hAB

1.5 + 4.5( )2

2 × 9.81+ 3 = 0 +

1.5( )2

2 × 9.81+

PB

ω+

0.3 4.5 −1.5( )2

2 × 9.81

1.5 + 1.03 + 3 = 0+0.115 + PB

ω+ 0.138

PB

ω= 5.53 - 0.253 = 5.28m of liquid

PB = 5.28 × 0.9 × 9810 = 46.617 kpa

Page 86: Fluid Mechanics and Hydraulic Machinery

E310/1 86 WE(3) A 5m long vertical conical pipe with upper and lower diameters as 0.3m and 0.6m

respectively is submerged in water in a sump, such that the depth of water above the lower

end is 1.75m. If the pressure at upper end is 30cm of mercury vacuum and the loss of head

in the pipe is 1

10of K.E at the upper end, calculate the discharge and pressure head at

lower end. The water in the sump is exposed to atmosphere.

SOLUTION

Apply Bernoulli’s theorem between A and B taking datum through A

Fig 5.5

PwA + ZA +

Vg

Pw

A B2

2= + ZB +

VB2

2g+ hf A. B

Pw

Vg

Vg

Vg

A A B B+ + =−

××

+ + +02

30100

136 98109810

52

012

2 2 2..

= - 4.08 + 5 + 1.1 VB2

2g

a = aA vA = aB vB

VA =aBvB

aA=

0.30.6

2

× vB =vB

4

To find PwA , apply Bernoulli’s theorem between (A) and free surface, take datum through

‘A’

Page 87: Fluid Mechanics and Hydraulic Machinery

E310/1 87

0 + Pw

Vg

A A+2

2= 1.75 +0 + 0

Pw

Vg

A A+2

2= 1.75 , substituting this in above equation

1.75 = - 4.08 + 5 + 1.1 VB

2

2g

VB2

2g=

0.831.1

= 0.7545

VB = 3.87m/s

Discharge = Π 0.3( )2

4× 3.87 = 0.272m3/s = 272lps

PwA = pressure at A = 1.75 -

VA2

2g= 1.75 −

VB4

12g

= 1.75 −3.87

4

119.62

= 1.708

m

WE(4) A 2.5cm dia pipe is fixed to the end of a 10cm dia pipe discharging water from a

tank to atmosphere. The depth of water in the tank over the centre line of the nozzle is 4m.

The loss of head in the pipe is given as 20v2

2g, where v is the velocity in the pipe. Also

determine the pressure at the base of the nozzle.

SOLUTION

Fig 5.6

Apply Bernoulli’s. theorem between 1 and 3 datum through 3

Page 88: Fluid Mechanics and Hydraulic Machinery

E310/1 88

Pw

ZV

gPw

Zg

ghL

11

12

33

32

2 2 1 3+ + = + + +

0 + 4 + 0 = 0 + 0 +V3

2

2g+

20v22

2g

But by continuity equation, we have

A2 V2 = As V3

V2=A3

A3

V3 =2.510

2

× v3 =1

16v3

Substitute this in the above equation, then

4 = V3

2

2g+ 20 ×

116( )2

V32

2g= 1.07813

V32

2g

V3= 8.53m/s ; V2 =1

16× V3 = 0.5332m/s

∴Discharge Q = Π4

0.025( )2 × 8.53 = 4.19lps

Loss of energy = hL1−3 = 20 ×V2

2

2g= 20 ×

0.5332( )2

19.62= 0.29m

Now apply B. Theorem between 1 and 2

4 02

202

2 22

22

= + + +Pw

Vg

Vg

4 =( )P

wVg

Pw

Pw

2 22

22

2212

210 5332

19 620 3+ = + × = +

..

.

∴Pw2 4 03 3 7= − = .

i.e. P2 = 3.7 × 9810 = 36.3 KN/a2 (kpa)

WE(5) A siphon consisting of a pipe of 2cm dia is used to empty water from a tank which

discharges into atmosphere at a point 3m below the water level in the tank. The highest

point in the siphon is 2m above the water level. Determine the discharge and pressure at

highest point the siphon. The losses unto the highest pt is 0.5m and unto the end of pipe is

1.5m

Page 89: Fluid Mechanics and Hydraulic Machinery

E310/1 89

Fig 5.7

SOLUTION

Apply Bernoulli’s Theorem between 1 and 3 take datum through 3

Z1+Pw

Vg

ZPw

Vg

hL1 1

2

33 3

2

2 2 1 3+ = + + +

3 + 0 + 0 = 0 + 0 +V3

2

2g+1.5

∴V2

2

2g= 1.5;V3 = 2 × 9.81× 1.5 = 5.425m/s

Discharge = ( )π πdV lps

2

32

4 40 02 5425 17× = × =. . .

Apply Bernoulli’s Theorem between 1 and 2 datum through 1 V2 = V3 = 5.425m/s

0 + 0 + 0 = Pw

Vg

Z hL2 2

2

22 1 2+ + +

.

=( )P

wP2

225425

19 622 05 15 2 5+ + + = + +

..

. . .ω

Pw2 = - 4m of water

P2 = - (4 × 9810) = 39.24 kpa vacuum

Page 90: Fluid Mechanics and Hydraulic Machinery

E310/1 90 WE(6) A closed tank in which water is filled partly and the space above it is under

pressure. A hose pipe of dia 5cm connected to a tank discharges water to the roof of a

building of 6m high above water level in the tank. The frictional losses are taken as 3m of

water. Determine the pressure of air in the tank be maintained to deliver 15l/s to the roof.

SOLUTION

Apply Bernoulli’s Theorem between 1&2 datum through 1

Fig 5.8

Pw

Vg

ZPw

Vg

Z hL1 1

2

12 2

2

22 2 1 2+ + = + +

Pw

Vg

1 22

0 0 02

6 3+ + = + + +

but V2 =( )Q

a2

2151000

10 05

47 64= × =π . . m/s

∴Substituting V2 is above equation, we have

( )P

wPw

m12

17 6419 62

9 9 2 98 1198= + = + =.

.; . .

So P1= 11.98 × 9.81 × 1000 = 117kpa

Page 91: Fluid Mechanics and Hydraulic Machinery

E310/1 91 WE(7) Water flows from a tank into a pipe at the rate of 60lps as shown in figure. At ‘P’

a head of 20m is added by external device, to the fluid. The pipe is 10cm in dia. Determine

the pressure at B if the loss of head is 2V 2

2g, Where V is the velocity in the pipe.

SOLUTION Velocity in the pipe

Fig 5.9

=Qa

=60

1000×

1Π4

0.01( )2

= 7.64 m/s ; V 2

2g=

7.64( )2

19.62= 2.975m

Now apply Bernoulli’s Theorem between A and B. Take datum through ‘P’

Pw

Vg

Z HPw

Vg

Z HA AA ad

B BB LAB

+ + + = + + +2 2

2 2

Where Had is the energy added.

0 + 0 + 3 + 20 = PwB + 2.975 + 5 + 2 × 2.975

23 = PwB + 3 × 2.975 + 5 =

PwB + 13.925

∴ =PwB 9 075.

PB = 9.075 × 9810 = 89.025 kpa

5.4 MOMENTUM EQUATION (impulse momentum equation)

Page 92: Fluid Mechanics and Hydraulic Machinery

E310/1 92 INTRODUCTION

Momentum equation is derived based on Newton’s 2nd law. It is another

important equation in fluid mechanics. Along with continuity equation and energy

equation, it is used to solve many fluid problems which are not solved only continuity and

energy equation.

DERIVATION OF MOMENTUM EQUATION

Momentum equation states that the external forces acting on a fluid are equal to

rate of change of momentum in that direction

( )

a Fd mv

dtΣ =

i.e. Σ F × dt = d( mv )

Where Σ F × dt is called as impulse.

Now consider a steady I-D flow through a bend as shown in the figure.

Fig 5.10

between 1 -2. Let the particles move in ‘dt’ time through the distances of ds1 and ds2 from

1 and 2 respectively and occupy 3 and 4 positions.

Here the particles in the portion 3 -2 is common before and after ‘dt’ sec and so the

momentum of the mass in this common portion gets cancelled in the calculations of

change in momentum of the mass before and after ‘dt’ seconds. So to calculate the

momentum of the mass before ‘dt’ sec, it is enough to consider the mass in 1 & 3 and 2 &

4 as the mass in the portion 3 & 2 is common.

Thus the momentum of the mass before ‘dt’ sec is called as ‘Initial momentum’ given by (

in x direction )

Page 93: Fluid Mechanics and Hydraulic Machinery

E310/1 93

= × ×wg

A d Vs a1 11

Where A1 is the cross section area of flow at (1) and V1 is the mean velocity over the C.S

at (1) After ‘dt’ sec, the final momentum in X derivation is

= = × × ×wg

A d Vs x2 22and

Where A2 is the C.S of flow at 2 and V2 is the mean velocity at 2. So change in

momentum in dt sec is.

= Final momentum - Initial momentum

= wg

A ds Vwg

A ds Vx x2 2 2 1 1 1−

i.e. Rate of change of momentum is

= −wg

Adsdt

Vwg

Adsdt

Vx x22

2 11

1

=wg

A V Vwg

A V Vx x2 2 2 1 1 1−

where A2 V2 = A1 V1= Q = Discharge ∴ change in momentum per second is

( )wQg

V Vx x2 1−

This is equal to external force in the direction

i.e. ( )ΣFwQg

V Vx y x= −2 1

Similarly in y direction

( )ΣFwQg

V Vy y y= −2 1

and the resultant force on the bend is

ΣF = ΣFx2 + ΣFy

2 which is inclined to horizontal

byθ = tan −1 ΣFy

ΣFx

ADVANTAGE OF MOMENTUM EQUATION :

Page 94: Fluid Mechanics and Hydraulic Machinery

E310/1 94 When the details of fluid flow with in a control volume are not known and the

external forces acting over the fluid with in the control volume are known, then

momentum can be applied to this flow and solve the problem along with the other two

equations called 1 continuity equation and 2 energy equation. this equation is very useful

in hydraulic machinery

MOMENTUM CORRECTION FACTOR ' β' .

The same analysis which is followed to calculate the kinetic energy correction

factor, is now also considered to calculate momentum correction factor. Momentum over

the cross sectional area of fluid based on mean velocity is

Mass × velocity

= (ρ A× V ) × V = ρ AV2

But based on the actual velocity distribution over the cross sectional area of flow is

calculate d as follows over ‘dA’ area,

= elemental mass × V

= ( ρ dA v ) × v

Now if this is integrated over the area of cross section of fluid flow, the momentum of the

mass flowing over the cross sectional area of flow is

= ρdAV2

A∫

= ρV 2

A∫ dA

In these calculations, it is clear that the square of mean velocity over the area is

less than the square of instantaneous velocity over the area. So to get the actual

momentum based on mean velocity it is to be multiplied by a factor ' β' called as

momentum correction factor.

i.e. β × ρAV2 = ρV 2

A∫ dA

β =V 2

A∫ dA

AV 2

Q AV2 less than V 2

A∫ dA

Page 95: Fluid Mechanics and Hydraulic Machinery

E310/1 95 β is always greater than ‘1’. For laminar flow through pipe β = 1.33 and Turbulent flow

through pipe β = 1.01. So in turbulent flow through pipes β is almost equal to ‘1’ and so

this factor is not considered in the momentum calculations in turbulent flow i.e. β =1 is

taken.

SAQ(19) In momentum equation change in momentum takes place in the direction of

forces. True/False.

SAQ(20) There is no need of estimation of forces with in the fluid in control volume

in application of momentum equation.

True/False.

SAQ(21) For steady uniform flow, calculate the external forces acting on the fluid.

SAQ(22) 1kg(m) /s changes its velocity form 3m/s to 33m/s in a nozzle. Determine

the force acting on the nozzle.

SAQ(23) The momentum of a jet 30N.s is brought to zero by a plate perpendicular to

the jet, then calculate the force of jet on the plate.

SAQ(24) Given the mean velocity over 1m2 cross sectional area of flow is 2m/s and

the integral volume of squares of velocities over the area is 4.4, calculate

the momentum correction factor.

WE(8) In a 450 bend of cross sectional area 1m2 is gradually reduced to 0.5m2 area. Find

the force required to keep the bend in position if the velocity and pressure at 1m2 section

are 10m/s and 300 kpa respectively

Page 96: Fluid Mechanics and Hydraulic Machinery

E310/1 96 SOLUTION Velocity at 2 is

Fig 5.11

V2 =a1v1

a2

=1 ×10

0.5= 20m / s,

V22

2g= 20.387m

V12

2g=

10( )2

2 × 9.81= 5.097m

Pw

m1 300 10009 810 1000

30=××

=.

Now apply Bernoulli’s Theorem 1 and 2

Pw

Vg

Pw

Vg

1 12

2 22

2 2+ = +

30+5.097 = Pw2 + 20.387

∴P2

ω= 10.2 ; P2 = 10.2 × 9810 = 100 kpa

Now apply momentum equation, we have in ‘x’ deviation,

P1a1 - Rx - P2 a2 cos 45 = ( )wag

V Vx x2 1−

Here R is the resultant force required to keep the bend in position. This is resolved into

horizontal and vertical components as Rx × Ry

So 300 × 1 - Rx - 100 × 0.5 × cos 45 = 9.819.81

× 1 ×10( ) 20 cos45 −10( )

Rx = 223.22 KN

in y direction, P1a1 sin θ + Ry - P1a1 sin45 = ωas

V2 sin 45 − V1 sin 0( )

Page 97: Fluid Mechanics and Hydraulic Machinery

E310/1 97 0 + Ry - 100 × 0.5 × sin45 = 10 (20 × sin45-0)

Ry =176.78 KN

∴ The resultant force

R = RX2 + Ry

2

= 223.22( )2 + 176.78( )2

= 284.7 KN

Its inelination = tan-1 Ry

Rx

α = 38o.37

WE(9) A 2.5 cm dia nozzle is fitted to a 4.0 cm dia pipe. If the discharge is 15 lps.

Calculate the force exerted by fluid on nozzle tends to tear it of the pipe .

SOLUTION

a1 =Π4

0.04( )2 = 0.00125m 2

The velocity in pipe = 0.015

Π4

0.04( )2= 11.94m / s

The velocity through the nozzle = 4

2.5

2

×11.94 = 30.57m / s

V12

2g=

11.94( )2

2 × 9.81= 7.27m;

V22

2g=

30.57( )2

2 × 9.81= 47.63m

Now apply Bernoulli’s Theorem to fluid pressure at base of nozzle as

Pw

Vg

Vg

1 12

22

20

2+ = +

Fig 5.12

Pw

Vg

Vg

m1 22

12

2 247 63 7 27 40 36= − = − =. . .

Page 98: Fluid Mechanics and Hydraulic Machinery

E310/1 98 P1 = 40.36 × 9.81 = 395.9kpa

Now apply momentum to find the force an nozzle as

P1a1 - Rx = ( )wag

V V2 1−

395.9 × 0.00125 - Rx =9.8109.81

×15

100030.57 −11.94( )

Rx = 0.7765 KN = 776.5N

WE(10) A vertical jet of water issuing from a 5cm dia nozzle with a velocity of

10m/s supports a horizontal flat plate at a height of 3m.

Find the weight of plate Neglect losses

SOLUTION

Fig 5.13

Q = a1× v1=Π4

0.05( )2 × 10 = 1.96 ×10−2 m / s

To find velocity at 2 apply Bernoulli’s Theorem between 1 and 2

ZPw

Vg

ZPw

Vg1

1 12

22 2

2

2 2+ + = + +

0 + 0 +10 2

2 × 9.81= 3 + 0 +

V22

2g;V2

2

2g= 5.0968 − 3( )

V2 = 5.0968 − 3( )× 2 × 9.81 = 6.414m / s

To find W, apply momentum equation

W = ( )wQg

V0 2−

Page 99: Fluid Mechanics and Hydraulic Machinery

E310/1 99 =

98109.81

×1.96 ×10−2 −6.414( )

weight of plate = 125.94N

SUMMARY

(1) Forces causing the flow of fluid are listed as Fp,Fg,Fv,Ft,Fe and F

(2) Out of these, FP and Fg are the two forces causing flow in ideal fluid. Fg are called

as body forces which are due to weight of fluid. FP pressure forces are due to

surface area. The pressure forces are further divided into tangential forces and

normal forces. But in ideal fluid tangential forces are zero. So pressure forces are

given by = (pressure × C-S of flow) = normal force.

(3) In Euler’s equation. Body forces and pressure forces are considered as it is applied

to steady incompressible ideal fluid flow.

(4) 1ρ

dpdx

− S = a, is the Euler’s equation along stream line.

(5) Then Bernoulli’s equation is derived by integrating Euler’s equation along the

stream line as

H = ZPw

Vg

+ +

2

2= constant at any point.

Where Z = datum head = datum energy / unit ωt of fluid

Pw

= Pressure head = pressure energy/ unit ωt of fluid

V 2

2g= Velocity head = K.Energy/ unit ωt of fluid

The sum ZPw

Vg

+ +2

2is also called Total head ‘H’ or Total energy / unit weight of

fluid flowing where w= weight of fluid flowing = wQ. This Bernoulli’s Theorem

states that for ideal incompressible steady and continuous fluid flow the total head

and velocity head is constant at any point along its path

(6) Pw

Z+

is called as piezometer head

Page 100: Fluid Mechanics and Hydraulic Machinery

E310/1 100 (7) In real fluid losses are to be considered in the application of Bernoulli’s equation

as

H1 = H2+HL, where HL is the loss of head between 1 and 2

(8) Momentum equation states that the external forces acting an a fluid flow are equal

to rate of change of momentum in that direction , i.e.

( )ΣFwQS

V VS S S= −2 1

(9) The advantage of momentum equation is that when the information about the flow

process with in a control volume is not known then such problems can be solved

by momentum equation by knowing the external forces acting on the fluid.

ANSWER FROM SAQ

(1) to (8) True (9) Yes (10) No (14) v = 6.26m/s

(15) 2m, 1 and 2 (16) 68.67 kpa (17) -3m of water (18) 7m

(19) True (20) True (21) Zero (22) 30N

(23) 30N (24) β 1.1

UNIT 5

EXERCISE

(5.1) A pipe line carrying oil of sp gr 0.877 changes in size from 0.15m dia at A to

0.45m to B. Which is 3.6m above A. If the pressure at A and B are respectively

90.252 KN/m2& 59.841 KN/m2. The discharge is 145 lps. Then determine the

cross head and direction of flow.

[ HL= 3.32m of oil A →B]

(5.2) A pipe line carrying water changes in area from 0.031m2 at A to 0.124m2 at B

which is 3m above A’. If the velocity and pressure at A are 5m/s and 100KN/m2,

find the velocity and pressure at B. Neglect losses

[ 1.25m/s & 82kpa]

(5.3) A 60cm dia pipe is supplying water under a pressure head of 40m to a Hydraulic

Machine (T). Then water flows out of it through a 45cm dia pipe at a rate of 0.8m3/s

Page 101: Fluid Mechanics and Hydraulic Machinery

E310/1 101 into atmosphere. Neglecting the losses determine the energy consumed by the

machine.

Fig 5.14

[ 38.71m ]

(5.4) Water enters a Hydraulic machine at A where the dia of pipeis 40cm, under a

pressure head of 30m. Then water flows out of the machine through tapering tube as

shown in figure at a rate of 0,5m3/s. At a point B which is 2m below the point A, the

dia is 60cm and pressure head is -4m. Determine the power developed by the machine

if the efficiency of the machine is 90%

Fig 5.15

[ 161.45 kw]

(5.5) A conical pipe has diameters 0.40m and 0.80m at its two ends. The smaller end is

2m above the larger end. For a flow of 0.5m3/s of water the pressure at the lower

end is 10kpa. Assuming a head loss of 2m and kinetic energy correction factor

α=1.1 and 1.5 at smaller and larger ends respectively estimate the pressure at

smaller end

[P17.144 kpa]

Page 102: Fluid Mechanics and Hydraulic Machinery

E310/1 102

(5.6) A pipe line has the following data at its two section’s A and B

A B

Diameter 30cm 45cm

Elevation(m) 10.0 16.0

Pressure kpa 40.0 30.0

Kinetic energy correction

factor

1.08 1.25

Assume a loss of head equal to 20 times the velocity head at A. Calculate the

discharge of water flowing from B to A

[Qa = 147.5 l/s]

(5.7) Water enters a hydraulic machine ‘P’ at 70 lps through A and delivered by it through

B to higher level. A mercury manometer connected between A and B shows a

deflection of 40cm. Determine the head added by the Hydraulic machine.

Fig 5.16

[Hp = 6.193m]

(5.8) A 20cm dia pipe leading water from a reservoir ends in a nozzle of dia 10cm at

elevation of 900m. The water level in the reservoir in 100m. The loss of head in

the pipe is 12 times velocity head in the pipe. Calculate the discharge.

Page 103: Fluid Mechanics and Hydraulic Machinery

E310/1 103

Fig 5.17

(5.9) A pipe line delivering water from a reservoir is shown in figure. A hydraulic

machine P is adding energy to the flow. The rate of discharge into atmosphere at

out let is 4.5lps. Assuming loss of head as 2 times the velocity head in the 20cm

pipe and 10 times the velocity head in 15cm dia determine power delivered by the

machine.

Fig 5.18

(5.10) Calculate the discharge through the pipe line shown in figure.Also find the

pressure at A and B. The depth of water in the reservoir in 5m. Neglect losses.

Fig 5.19

[ 0.0194m3/s, 0.769 kg./m2, 0.231kg/cm2]

(5.11) Velocity distribution in a pipe is given by V = Vmax 1 −r 2

R2

at any radius ‘r’

from the centre of the pipe. Vmax= Max velocity at centre of pipe R= Radius of the

Page 104: Fluid Mechanics and Hydraulic Machinery

E310/1 104 pipe. Determine kinetic energy correction factor and momentum correction factor

β

α = 2,β =43

(5.12) A 20cm dia pipe has a 90o bend in the horizontal plane. Oil of sp gr 0.8 is flowing

at 150lps in the pipe. At in let the pressure is 0.5m of Oil . Find the resultant force

exerted by the oil on the bend

[F = 1612.5 N inclined at 4.190to horizontal]

(5.13) A 60o reducer bend at the end of a pipe 30cm dia, discharges freely through 20cm

dia into atmosphere at 10m/s. The bend is in vertical plane and the centre of exit is

60cm above the centre line of pipe. The weight in side the bend in 700N.

Determine the force required to keep the bend in position

[ R= 4593 N, θ = 131.95o with horizontal]

(5.14) Water flows through 180o vertical reducing bend shown in figure. The pressure at

in let is 20kpa and the discharge is 0.4m3/s If the bend volume 50.8m3. Calculate

the force required to hold the bend in place.

Fig 5.20

[ F = 10854N at 136.19o to

horizontal]

Page 105: Fluid Mechanics and Hydraulic Machinery

E310/1 105 (5.15) A sluice gate in a rectangular channel carrying water is so opened to create depths

of flow of 1.70m and 0.25m on u/s and d/s of the gate respectively. The discharge

intensity is 1.3m3/s per meter width. Estimate the force per meter on the gate

[ F = 8.09 KN]

(5.16) A tank and a trough are placed on a trolley as shown in figure. Water issues from

the tank through a 4cm dia nozzle at a velocity at 5m/s and strikes the trough

which turns it up by 45o. Determine the compression of the spring if its stiffness is

2kg /cm

[1.129cm]

Fig 5.21

References same as unit 1

***

FLUID MECHANICS & HYDRAULIC MACHINERY

UNIT-6

AIMS

The aim of this unit is to know about some more fluid flow measuring devices like

pitot tube, elbowmeter, Trapezoidal notch, their principle of measurement and uses using

Bernoulli’s Theorem.

OBJECTIVES

(1) To know about the meaning of pitot tube, static pressure, stagnation pressure and

dynamic pressure heads.

Page 106: Fluid Mechanics and Hydraulic Machinery

E310/1 106 (2) To derive an equation for velocity of flow by pitot tube by applying energy

equation and to know the application of pitot tube in measuring discharges

flowing in open channels and through pipes.

(3) To understand the meaning of elbow (bend) meter and to obtain an expression

for discharge flowing in a pipe of elbow meter.

(4) To review wiers and notches .

(5) To explain about end contractions velocity of approach and ventilation of wiers.

(6) To derive an expression for discharge flowing in open channels for Trapezoidal

notch.

(7) To explain about cippoletti weir. which is a special type of Trapezoidal weir.

(8) To know how to solve practical problems using pitot tube, elbowmeter and

Trapezoidal notch in measuring the rate of flow.

6.1 INTRODUCTION

The rate of fluid flow in open channels or through pipes is very much useful in

solving many practical fluid flow problems like in irrigation and water supply etc. The

discharge in open channels or through pipes can be measured by various measuring

devices. In previous courses some of such devices like venturimeter, orificemeter. flow

nozzle meter, orifices, mouth pieces rectangular notch and triangular notch are known.

Now in this unit still some more useful measuring devices like pitot tube, elbow meter and

trapezoidal notch are to be studied.

6.2 PITOT TUBE

Before deriving an equation for velocity of flow in a pipe, let us consider flow of

fluid with a velocity over a cylinder as shown in figure

Fig 6.1

Page 107: Fluid Mechanics and Hydraulic Machinery

E310/1 107 When the flow approaches the body the velocity tends to be zero at S1andS2, Which are

known as stagnation points. At this stagnation point all the kinetic energy is converted to

pressure energy. If Bernoulli’s equation is applied between 1 and S1, we have

Pw

Vg

Pw

s+ = +2

20

or P + ρv2

2= Ps

Here P is called as static pressure

Ps is called as stagnation point

and ρv2

2is called as dynamic pressure

At stagnation point, the dynamic pressure is converted into equivalent pressure.

With this understanding let us study about pitot tube. Derivation of equation for velocity

of flow by pitot tube

Pitot tube is a bent tube used first in 1730 by French scientist Henri Pitot to

measure velocity of flow in a river. So it is named after him. Let the pitot tube be kept in

the fluid flow as shown in figure in open channel and pipe

Open Channel Flow Pipe Flow

Fig 6.2

Let at point 1 the undisturbed mean velocity of flow is Vm/s and the static pressure head

isPw

Now applying Bernoulli’s Theorem between 1 and 2 taking datum through them.

Page 108: Fluid Mechanics and Hydraulic Machinery

E310/1 108

Pw

Vg

Pw

s+ =2

2, neglecting losses

V

gPw

Pw

Hs2

2= − =

V = 2gH is the ideal velocity

or Actual velocity = Va = C 2gH = 2g H − hf( )Which is less than ideal velocity

where hf = loss of head

so the coefficient C < 1.0

i.e. C = 0.98 nearly in actual case to account for loss of energy. The actual value of ‘C’ can

be determined by calibration.

If the differential pressure head is more, then a differential momentum may be

used to measure the differential pressure as shown in figure

Fig 6.3

By gauge equation we have

Pωl

+hsm

sl− h− =

Psωl

Ps − Pω l

= H = hsm

sl

− 1

H = 12.6h (m of water) if

sm = 13.6 (mercury)

sl = 1.0 (water)

Pitot - static tube

It measures both static pressure and stagnation pressure as shown in figure

Page 109: Fluid Mechanics and Hydraulic Machinery

E310/1 109

Fig 6.4

The central tube measures the stagnation pressure and the outer tube with holes

around the periphery as shown in figure, measures static pressure.

Prandtl pitot tube

Is a pitot - static tube as shown in figure with standard proportions which gives

accurate results. The blunt nose is so designed that the loss of head due to turbulence is

negligible and so the coefficient ‘C’ is almost equal to 1.0

Fig 6.5

Pitometer

Is a combination of both static pressure tube and stagnation pressure tube as shown

in figure which gives more differential pressure head than the pitot - static tube. It consists

of two pitot tube, one facing upstream and another facing down stream.

Page 110: Fluid Mechanics and Hydraulic Machinery

E310/1 110

Fig 6.6

V = C 2gh1

Here the value of ‘C’ is less than that of pitot static tube. i.e. C is about 0.8 to 0.85.

IDENTIFY THE FOLLOWING WHETHER “TRUE” OR “FALSE”

SAQ(1) Pitot tube measures total head.

SAQ(2) Pitot static tube measures both static and stagnation pressures.

SAQ(3) In pitot tube dynamic pressure is equal to kinetic head.

SAQ(4) The difference of static and stagnation pressure heads measured by pitot

static tube gives the dynamic head which is equal to velocity head.

SAQ(5) Pitot tube gives velocity at a point or discharge flowing in the pipe.SAQ(6)

What do you understand by C=1?

SAQ(7) The difference between static and stagnation pressure heads is 100cm of

water. what is the velocity C=1.

SAQ(8) The mercury deflection connected to pitot static tube gives 10cm of

mercury. Find the velocity of water C=1

SAQ(9) If velocity of oil of sp gr 0.9 at a point is 5m/s, find the deflection in

mercury differential manometer connected to a pitot static tube C=1.

WE (1) A pitot tube is inserted into an air stream where the static pressure is

100kpa. If the water deflection in the differential manometer is 30cm, determine the

velocity of air. Given C=1 and density of air = 1.22 kg/m3

SOLUTION

Differential head

Page 111: Fluid Mechanics and Hydraulic Machinery

E310/1 111

H = 0.3 10001.22

−1

= 0.3 ( 818.67 )

= 245.6 m of air

V = 0.98 2 × 9.8 × 245.6

= 68m/s

WE (2) Carbon tetrachloride of sp gr 1.6 flows in a pipe. A differential manometer

attached to a pitot static tube gives a deflection of 100mm of mercury. Determine the

velocity of flow C=1.0

H = hSm

Sl

−1

=

1001000

13.61.6

−1

= 0.75m

V = 2g × 4

= 19.62 × 0.75 = 3.84m/s

WE (3) In a 12cm dia pipe line water is flowing. Given the diameter of the pipe the

pitot static tube gives a deflection of 5cm and 15m of water at centre point and at 1.5cm

from the wall respectively. Determine the discharge C=1

Fig 6.7

a1 =Πd1

2

4=

Π 0.06( )2

4= 2.83 × 10−3 m 2

V1 = 2g × 5 = 9.9m / s

∆Q1 = a1V1= 0.028 m3/s

Page 112: Fluid Mechanics and Hydraulic Machinery

E310/1 112 a2 =

Π4

( 0.122- 0.062) = 8.48×10-3m2

V2 = 2g × 1.5 = 5.43 m/s

∆Q2 = 0.046m3/s = a2v2

Total discharge Q = ∆Q1 +∆Q2

= 0.074m3/s

6.3 ELBOMETER OR BEND METER

We know already that by creating a pressure difference between two points in a

pipe flow, by reducing the cross sectional area of the pipe, the rate of flow through it can

be measured, viz. venturimeter, orificemeter and flow nozzle meter by using the equation

Q = cd a1a2

a1 − a22

2gH or cd a1

a1

a2

2

− 1

2gH

Where a1a2 are the cross sectional area of pipe throat/ orifice/ nozzle. respectively

H=Pw

ZPw

Z11

22+

− +

; the piezometric head difference. Similarly based on

the above principle rate of flow through a pipe can also be measured by a bend or elbow as

derived below. And it is to be noted that bend meter is very simple and no construction of

meter at all, except pressure tappings at inner and outer walls of the bend or elbow as

shown in figure. In the bend the pressure difference is due to centrifugal force on the flow.

The flow in the bend is free vertex flow.

Derivation of equation for discharge

The particles in a bend move on the curved stream lines in a free vortex. Therefore

the velocities at 1 and 2 are

Page 113: Fluid Mechanics and Hydraulic Machinery

E310/1 113

Fig 6.8

V1 =C

R − r( )V2 =C

R + r( )Q We know in free vertex Vr = C at any radius ‘r’. Where C = a constant

Now applying Bernoulli’s Theorem between 1 and 2, We have

Pw

ZV

gPw

ZV

g1

112

22

22

2 2+ + = + +

H =Pw

ZPw

ZV V

g2

21

112

22

2+

− +

=

But V1=c

R − r( ) V2=c

R − r( )

H =c2

2g1

R− r( )2−

1R + r( )2

=c2

2g4Rr

R − r( )2 R − r( )2

=c 2

2g4Rr

R2 − r 2( )2

i.e. C = 2gHR2 − r 2( )2 Rr

To obtain an expression for discharge through the bend consider a stream tube of thickness

dx at a radicus x .

Page 114: Fluid Mechanics and Hydraulic Machinery

E310/1 114

Fig 6.9

The mean velocity over the C.S stream tube is

Vm =c2

R − x( ) R + x( ) =C

R2 − x2

Then the discharge through this stream tube is

dQ = 2ΠxdxVm

= 2Πxdx ×C 2

R − x( ) R + x( )

= 2Πxdx ×C

R2 − x 2

Now by integrating this equation the discharge through the bend is obtained as

Q = dQ xdxC

R xo

Q

o

r

∫ ∫= ×−

22 2

Π

= 2ΠCx

R xdx

o

r

2 2−∫

Q = 2ΠC R R xr

− −

2 20

= 2ΠC R − R2 − r 2[ ]

Page 115: Fluid Mechanics and Hydraulic Machinery

E310/1 115 Substituting the value of C, from the previous equation we have

Q = Π 2gHR2 − r2( )

R2R − R2 − r2

= Πr 2 2gH Rr

2

R− R 2 − r 2( )R2

The actual discharge is

Qa = cd A 2gHRr

2

−1

R − R2 − r 2

R2

Qa = cd k1 A 2gH where k1=Rr

2

−1

R − R2 − r 2

Rr

= KA 2gH

Where K = Cdk1= Bond meter constant . But by experiments it is shown that

K1 =R

2a

and K = 0.55 - 0.88

SAQ(10) In bend meter, the flow is subjected to centinfugal force. Is it True/False?

SAQ(11) In bend meter the flow is free vertex flow. Is it True/False

WE (4) Oil of sp gr 0.9 flows in a horizontal bend meter of 10cm dia of the

pressure difference between inner and outer walls is 10 KN/m2, find the discharge. Given

bend meter constant as 0.6.

SOLUTION

Q = KA 2gH

H =10 ×10000.9 × 9810

= 1.132 m of oil

A = Π4

0.1( )2

Q = 0.6 ×Π4

0.1( )2 2 × 9.81×1.132

Page 116: Fluid Mechanics and Hydraulic Machinery

E310/1 116 = 0.0222 m3/D

6.4 TRAPEZOIDAL NOTCH

We know already that the discharge flowing in open channel can be measured by

weirs and notches viz. Rectangular notch and triangular notch.

The equation for discharge for rectangular notch is

Q = Cd23

L 2gH 3 / 2

and for triangular notch

Q = Cd8

152g tanθ / 2H5 / 2

Where L = Sill length of rectangular notch.

H = Head over sill of notch.

θ = The vertex angle of triangular notch.

End contractions

When the sill length of rectangular notch is less than the width of channel, then the

nappe will contract giving rise to two end contractions as shown in figure 6.10

Fig 6.10

The effective sill length Le is

Le = ( L - 2 × 0.1 H )

where and contraction = 0.1 H If there are ‘n’ end contractions then the effective sill

length is.

Le = [ L - ( n× 0.1 ) H ]Velocity of approach

Page 117: Fluid Mechanics and Hydraulic Machinery

E310/1 117

Fig 6.11

Let Q be the discharge flowing in the open channel, then the velocity of approach with

which the particles are approaching the notch is.

Va =Q

B Z + H( )

where B = With of channel

Z = Height of notch sill over the bed of channel.

H = Head over sill

ha = Velocity head = Va2/ 2g

Then the equation for rectangular notch considering both end contractions and velocity of

approach is

Qa = Cd23

L - h × 0.1 × (H +ha)( H+ha)3/2- ha3/2

and for triangular notch considering velocity of approach is

Qa = Cd8

152g tanθ/2 ( H+ha)5/2-ha5/2

Steps Involved in finding discharge considering velocity of approach

1) First neglect the velocity of approach and determine the discharge to 1st

approximation.

2) Now in the 2nd trial, find velocity of approach as indicated already using the 1st

approximate discharge. Then using this velocity of approach 2nd approximate discharges

is determined.

Page 118: Fluid Mechanics and Hydraulic Machinery

E310/1 118 3) Again is 3rd trial, the above procedure is repeated to find the refined discharge. These

trials are repeated till the difference in the final discharge and the previous discharge is

negligible.

Ventilation of weirs

In case of suppressed rectangular weir where the width of channel is equal to sill

length of rectangular notch, the space below the nappe is hollow containing air as shown

in figure.

Fig 6.12

As the flow takes place the air inside this hollow space is carried away by the

flowing water, making the pressure inside the space is vacuum, because both ends of this

space, are closed by the walls of channel. Thus there is no way for the air to enter in for

this space below the nappe. So as the pressure below the nappe is vacuum, the nappe will

cling as shown in figure due to atmospheric pressure over the water. This clinging nappe

leads to error in the discharge measuring about 6 to 7% more than that of force nappe. To

avoid this error, air is supplied through end walls as shown in figure by providing two

numbers of holes of 25mm diameter in the end walls.

Page 119: Fluid Mechanics and Hydraulic Machinery

E310/1 119 Fig 6.13

Discharge of Trapezoidal notch

The Trapezoidal notch is a combination of rectangular and triangular notches. The

equation for discharge for this notch is

Fig 6.14

Q = Discharges through( rectangular notch + Triangular notch )

Q = Cd1

23

L 2gH 3 / 2 + Cd2

815

2g tanV2

H 5/ 2 of Cd1= Cd2

Q = Cd 2gH 3/ 2 23

L +8

15tan

θ2

H

Cippoletti weir

It is a special trapezoidal notch in which the loss of discharge due to end

contraction, of rectangular notch is made up by adding half triangular notch to each side of

rectangular

notch. That means the discharge lost by end contractions is equal to extra discharge

through triangular notch portions at the two ends. Finally the discharge for this cippoletti

weir is equal to the discharge of suppressed rectangular notch. Therefore the discharge

through cippoletti weir is

Q = Cd23

L 2gH3 /2 = Cd23

(L − 0.2 × H ) 2gH3/ 2 + Cd815

2g tanθ2

H5/ 2

i.e. Discharge lost through end contractions = Discharge added by triangular notch

∴ Cd23

0.2H( )H3/ 2 2g = Cd815

2g tanθ2

H5 / 2 14

= tanθ / 2

So the side slopes of this notch is 1 Horizontal to 4 vertical.

This trapezoidal notch with side slopes 1 in 4 is called as cippoletti weir whose discharge

is.

Page 120: Fluid Mechanics and Hydraulic Machinery

E310/1 120 Q = Cd

23

L 2gH 3 / 2 . Which is same as that for rectangular notch without end

contractions.

SAQ(12) Trapezoidal notch discharge is sum of both rectangular notch and triangular

notch discharges. Is it True/False.

SAQ(13) A trapezoidal notch with side slopes 1 in 4 is called as cippoletti weir. Is it

True/False.

SAQ(14) Given Q = 0.1m3/s in a channel of 0.5m wide. The height of sill of a notch

is 0.1m and head over the notch is 0.1m. Then find the velocity of

approach.

WE(5) The discharge flowing over a rectangular weir under a head of 0.4m is 5m3/s

without end contractions. Find the side slopes in order to increase the discharge by 5%.

Take Cd = 0.6. Find the increase in discharge for side slope of 45o with vertical.

SOLUTION

Q2 = Q - Q1= 0.05 × 5 = 0.25 m3/D

0.25 = 0.6 8

152g tanθ / 2 × H5/ 2 = 0.1434

tanθ / 2 = 1.743

θ/2 = 6.16o or slope is 1 to 0.5737

Increase in discharge for sides slope of 45o with vertical is

Q2 = 0.1434 tan θ/2 = 0.143 × 1m3/s

∴ increase in discharge = 0.1434

5= 2.868%

SUMMARY

1. The equation for velocity by a pilot tube is V = C 2gH , where H is the

differential head between stagnation pressure and static pressure.

2. The equation for discharge by an elbow meter is

Q = KA 2gH

where K = meter constant = 0.55 to 0.88 and K = Cd K1

Page 121: Fluid Mechanics and Hydraulic Machinery

E310/1 121 where K1= R / 2d , R = radius of curvature of bend centre line and d = diameter

of pipe.

3. The equation for discharge for trapezoidal notch is sum of discharges rectangular

and triangular notches.

i.e. Q = Cd1L3

L 2gH3 / 2 + Cd28

152g tan

θ2

H5/ 2

4. Cippoletti weir is a special trapezoidal notch with side slopes 1 H to 4V and the

discharge is equal to that of rectangular notch without end contractions

i.e. Q = Cd23

L 2gH 3 / 2

5. The discharge for rectangular notch considering end contractions as velocity of

approach is

Q = Cd23

L − 0.1h H + ha( )[ ] 2g H + ha( )3 / 2 − ha 3/ 2 where ha = Va2/2g

6. Ventilation of nappe is necessary in case of suppressed rectangular notch to avoid

cligging of nappe. Further cligging of nappe leads to 6 to 7% more in discharge

estimation meter the same head.

Answer to SAQ

1-4 True 5. velocity

7. V = 4.43m/s 8. V= 5.58m/s 9. h = 9cm 10-13 True

14. Va = 1m/s

Exercise

(6.1) The velocity of an oil of sp gr 0.9 was measured by a pitot-static tube. The tube’s

coefficient ‘C’ is 0.98. Calculate the velocity if the deflection in the mercury

manometer is 6cm

[ Vo= 4.0 m/s]

(6.2) A pitot tube was arranged in centre line as shown in figure. Calculate the velocity

if the coefficient is 1.0

Page 122: Fluid Mechanics and Hydraulic Machinery

E310/1 122

Fig 6.15

[ Vo = 3.851m/s]

(6.3) Kerosene of sp gr 0.81 is flowing in a pipe and the static pressure at a point is 3kpa

of the stagnation pressure of a pitot tube ( c=0.99) inserted in the pipe at centre line

is 4kpa, find the velocity.

[ Vo = 1.56m/s]

(6.4) At the summit of a siphon in a water pipe the centre line velocity is 2.5m/s and the

pressure as 3m of water (vacuum) If a pitot tube (c=0.99) is inserted into the pipe

centreline at this section, what will be the stagnation pressure in kpa (abs).

Atmospheric pressure = 101kpa

[ Pst = 74.81kpa(abs)]

(6.5) A trapezoidal sharp-crested weir has a base width of 1.2m and side slopes of 1.5

horizontal-1vertical. Calculate the discharge over the weir for a head of 35cm.

Cd=0.62

[ Q = 0.614m3/s]

(6.6) Estimate the head over a cippoletti weir of base width 0.9m required to pass a

discharge of 600l/s. Cd = 0.63

[ H = 50.45 cm]

Page 123: Fluid Mechanics and Hydraulic Machinery

E310/1 123 (6.7) A trapezoidal notch is to be designed to pass a discharge of 1.0m3/s at a head of

0.8m over the crest of 0.5m3/s at a head of 0.5/m. If Cd = 0.7, then Calculate the

base width and the side slope of the notch.

[L = 0.643m θ =2.96o]

(6.8) Find the discharge through a trapezoidal weir with inward sloping sides as shown

in fig if Cd = 0.6

Fig 6.16

[ 210l/s ]

***

FLUID MECHANICS & HYDRAULIC MACHINERY

UNIT VII

FLOW THROUGH PIPES - TURBULENT FLOW

Aims: In this unit the aims are (1) to distinguish between Laminar flow and turbulent flow

by Reynold’s experiments, (2) to derive an equation for loss of energy in turbulent flow

through pipes, (3) to distinguish between flow in series pipes and parallel pipes and (4) to

study the flow in branch pipes.

Objectives:

1) To explain the Reynlod’s experiments to distinguish between laminar and turbulent

flows by flow patterns and a graph between loss of head and velocity on logarithmic scale.

(2) To define and derive Reynold’s number and its use in flow through pipes.

Page 124: Fluid Mechanics and Hydraulic Machinery

E310/1 124 (3) To explain fluid friction laws for laminar and turbulent flows.

(4) To derive an expression for loss of head in Turbulent flow through pipes.

(5) To analysis flow through series pipes.

(6) To explain about hydraulic gradient and energy live and

(7)To study flow through parallel pipes and branch pipes and to apply the above in solving

practical problems.

7.1 Introduction

When fluid flows under pressure in a closed conduct, then this flow is called as

pipe flow in which fluid run full. Due to viscosity of fluid in rear fluids there will be

opposition to fluid motion, which leads to loss of energy in fluid flow. Its determination is

very important in design of water supply system, carriers of water in irrigation etc. So in

this unit we will study how to determine the loss of energy in laminar and turbulent flows

and how to make use of this loss of energy in solving fluid problems.

7.2 Reynold’s experiments

Osborne Reynold’s was the first man to demonstrate an experiment in l883 to

distinguish between Laminar and Turbulent flows. He maintained a constant water level

in a glass tank as shown in figure.

He allowed a dye of same specific gravity as that of water, from a small tank into a bell

mouthed glass tube as shown in figure. A control valve is provided to the glass tube to

control the fluid flow in the glass tube.

Page 125: Fluid Mechanics and Hydraulic Machinery

E310/1 125 At low velocities, the dye appeared as parallel straight lines, indicating that the

fluid particles move in parallel layers. This flow is called as’ Laminar flow’ because the

fluid flow is taking place in layers, one layer sliding above and the layer, as shown in

figure.

When the velocity of flow is increased the particles slightly deviate from the

parallel paths but the viscous force makes the particles to move back to their original path,

leading to wavy paths of motion. This flow is called as transition flow.

When further the velocity is increased the colour diffuses into the water , making

water completely colourful.

This indicates the zig zag motion of fluid particles and this flow is called as turbulent flow.

In this turbulent flow, some particles move fast, some slow, some dash against another

leading to mixing of fluid particles. This mixing leads to loss of energy.

The velocity at which the region of flow changes, is called as critical velocity.

In the above experiments it was observed by Reynolds that the motion of water

was governed by relative magnitudes of Inertia force and viscous force. The ratio of these

two forces is called Reynold’s number, which is non dimensional.

Thus Reynolds’ no= Rn =Inertia force

Viscous force

Page 126: Fluid Mechanics and Hydraulic Machinery

E310/1 126 Inertia force = Mass × acceleration

= lL3 × LT2 = lL2 L2

T2 =lL2 V2

Where l-is mass density,

L-Characteristic length

V- Velocity of fluid

viscous force = τ × Area

= µ VL

× L2 = µVL

Where τ = shear stress = µ VY

µ = Viscosity of fluid

∴RN =ρL2V2

µVL=

ρVLµ

=VLυ

Where υ =µρ

For pipe flow let L=D-Dia of pipe

So RN =ρVD

µor

VDυ

At low velocities, the viscous force is predominant and so in laminar flow, this

force controls the particles. So move on parallel paths, where as in turbulent flow the

inertia force is predominant and so the particles deviate from the parallel paths leading to

mixing of the particles.

Thus in laminar flow the Viscous force is more so Reynolds’ number is less, where as in

turbulent flow the Viscous force is less and so Reynolds’ number is more.

To obtain the range of Reynolds’ number to distinguish between laminar and turbulent

flows, experiments on flow through pipe were conducted in which the loss of head over a

Page 127: Fluid Mechanics and Hydraulic Machinery

E310/1 127 length ‘L’ is measured for different velocities from zero to maximum possible. as

indicated in the diagram. The loss of energy per unit length is taken on ‘Y’ axis and

velocity on ‘X’ axis in logarithmic scale and a graphs drawn between loss of energy and

velocity of flow as shown in figure.

Again the loss of energy is measured from maximum possible velocity to zero

velocity by decreasing the flow rate. The trends shown in the figure.

From the graph it is clear that

(1) At low velocities the head loss hL V (velocity)

Upper A or even B, in which the regime of flow is laminar flow. At ‘A’ the flow is

definitely laminar even there is some disturbances in the flow. Because these disturbances

are damped off by viscose force which is more in the case of laminar flow. So ‘A’ is

called as a stable critical point at which regime of flow is changed.

The flow can be maintained laminar without any least disturbance in the flow upto

B at which the regime is changed. This point is unstable as the least disturbance will

cause the regime of flow changed. So this point is called unstable critical point.

Page 128: Fluid Mechanics and Hydraulic Machinery

E310/1 128 From the graph it is clear that when the velocity is going on increased gradually,

the regime of flow changes from laminar to transmission at B, and into turbulent at C. In

turbulent flow the trend of loss of energy Os non linear and so hL ϑυ Where υ = 1.72 to 2

when the flow is gradually decreased from maximum to minimum, the trend of loss of

energy is not the same as in the previous case, but deviates as shown in fig. 1 in which the

flow changes from turbulent to transition at ‘C’ and transmitted to laminar at ‘A’.

At point A the RN= 2000, which is known as lower critical point, while at point B,

which is unstable the RN can be maintained even upto 12000- 14000 without any

disturbances. This point is called as upper critical point. The practical value of Rnat

which the negative of flow is changed from transition to turbulent is 2700-4000. At these

point, the velocities are called as critical velocities. In between 2000-4000 the flow is

transition flow.

Thus from the graph it is clear that when RN ≤2000, the flow is always laminar and

when RN ≥ 4000, the flow is turbulent in a pipe and for RN from 2000 to 4000 the flow is

transition.

SAQ(1) for laminar flow in a pipe, the RN is≤ 2000. Is it true or false ?.

SAQ(2) The lower critical point is stable point Is it true or false?

SAQ(3) Determine the lower critical velocity in a pipe of 2cm dia given the velocity of

water=0.01 stokes.

7.3 Laws of fluid friction.

From the experimental observations the laws of fluid friction in laminar and

turbulent flows are given below.

(i) Laws of fluid friction in laminar. flow

The fluid frictional resistance in laminar flow is

(a) proportional to velocity of flow

(b) proportional to surface area which is in contact with fluid.

(c) Independent of the nature of the surface of contact, since a thin fluid layer adjacent to

the surface, known as laminar sub layer, submerges all the projections of the surface.

(d) Independent of pressure

(e) is affected by temperature,

since it depends on viscosity, which in turn varies with temperature.

(II) Laws of fluid friction in Turbulent flow.

Page 129: Fluid Mechanics and Hydraulic Machinery

E310/1 129 In turbulent flow the fluid frictional resistance is

(a) Proportional to Vn where =1.72to2.0

(b) Independent of pressure.

(c) Proportional to density of fluid

(d) Proportional to surface area in contact

(e) Dependent on nature of surface in contact

(f) Slightly affected by temperature.

III Froude’s experiments

From the experiments conducted by Froude, to investigate, the frictional resistance

for turbulent flow on different surfaces, the following conclusions are drawn.

(a) Frictional resistance varies approximately as square of velocity.

(b) The frictional resistance varies with the nature of surface.

(c) The frictional resistance per unit area of the surface decreases as the length of board

increases, but is constant for long lengths.

So the frictional resistance ‘F’ in turbulent flow is given by

F=f1 A Vn where n =2.0

Where f1 is the frictional resistance per unit area at unit velocity.

7.4 Equation for loss of head in Pipes in which the flow is turbulent

Consider a steady and uniform turbulent flow in a pipe of cross sectional area ‘A’

m2 and ‘L’m long in which fluid flows with a mean velocity Um/A

Let p2 and p2 be the pressures at (1) and (2) respectively.

Now applying Bernoulli’s theorem between (1) and (2), we have

p V L

gZ

p V Lg

Z hf1 1

12 2

22 2ω ω+ + = + + +

But Z1= Z2 , v v1 2=

∴(p1 − p2 )

ω= hf =loss of head due to friction

Page 130: Fluid Mechanics and Hydraulic Machinery

E310/1 130 Further for steady flow

∑ =F 0 because acceleration =0

i.e., (p1A-p2A) = Frictional resistance

=f1xSurface area xϑ n

(By Froude’s experiments)

=f1 p x L x vn

Where p= Wetted perimeter

(p1-p2) =f1 pLA

vn =f1MLv n

Where m= Hydraulic mean depth

=pA

πD

D2 4=

4D

Take n=2.0

Then (p1-p2) = f1 X4D

Lϑ2

i.e.,(p1 − p2 )

ω= hf =

4 f 1

ωLϑ 2

D=

8 f 1Lϑ 2

p2gD

ie hf=fL

gDf

fϑρ

2 1

28

; ( )=

Where ‘f’ is called as friction factor or coefficient of friction

The above equation is called as Darcy- Weis bach equation

This loss is more in magnitude and so it is called as Major loss of head.

7.5 Minor Losses

Minor losses are those which are due to sudden change either in magnitude or in

direction. The magnitude of these losses is less than the major loss due to friction and so

there losses are called as minor losses. These are to be considered when the lengths of

pipe are short. These losses are due to

(a) Sudden expansion(ϑ1 −ϑ2 )2

2g

(b) Sudden contraction=0.5v2

2g

(c) entrance = 0.5v2

2g

Page 131: Fluid Mechanics and Hydraulic Machinery

E310/1 131

(d) exit = v2

2g

(e) graded expansion or contraction = k( )V V

g1 2

2

2−

(f) bend kv

g

2

2

(g) Obstruction like value ,pipe fittings = kv 2

2g

Derivation of loss of energy due to sudden expansion.

Consider a sudden expansion as shown in figure from A1to A2.

Let p1 and p2 be the pressures and ϑ1andϑ2 be the velocities at (1) and (2) respectively

Now applying Bernoulli’s theorem between (1) and (2) we have

p V

g

L1 2

2ω+ + Z1=

p V Lg

2 2

2ω+ + Z2+ hLSE

Here when the cross sectional area of the pipe is suddenly enlarged from A1 to A2,

the fluid separates from the boundary at sudden expansion leading to deceleration of fluid.

This leads to folders formation which consume certain energy of fluid. After some

distance of travel those eddies die away giving rise to certain heat energy which is not

important in incompressible fluid. So the energy consumed by the eddies is a loss of

energy.

So hLSE=P1 − p2

ω+

V12 − V

2

2

2g

Further applying momentum equation over fluid in between (1) and (2) ,we have

p1A1 + p0 (A2- A1) - p2 A2 =ωQg

(V2-V1)

Page 132: Fluid Mechanics and Hydraulic Machinery

E310/1 132 Where Q= a1v1=a2v2

But by experiments it is shown that p0≈ p,

So (p1-p2) A2=ωA v

g2 2 (v2-v1)

i.e., (p1 − p2 )

ω=

v v vg

2 2 1( )−, substituting this in the above equation, we have

hLSE =v v v

g2 2 1( )−

+v v

g1

22

2−

=2 2 2

22 1 2 1

22

2v v v v vg

− + −=

( )v vg

1 22

2−

Loss of energy due to sudden contraction

Here loss of energy due to sudden contraction is also due to sudden expansion as shown in

figure

As the flow takes place from A1 to A2 the fluid contracts to a minimum cross sectional

area Ac and Ac

A2

=Cc called as coefficient of contraction Cc=0.62.

So loss of head due to sudden contraction is

HLSC=( )v v

g2 2

2

2−

But v a a vC C = 2 2

vaa

vCC

=2

2

=vC

C

C

HLSe = (vC

v XgC

22

2 12

− )

Page 133: Fluid Mechanics and Hydraulic Machinery

E310/1 133

=v

g CC

22

2

21

1( )−

=v

g ov

g2

22 2

2

2162

1 0 3752

(.

) .− =

But the actual value of this loss is

HLSe =0.5v

g2

2

2( as the loss of energy due to contraction is not considered here)

Loss of head at entrance

It is also similar to sudden contraction and so HLen=0.5vg

2

2

Loss of head at exit

It is similar to sudden expansion

ie Hlex =( )v v

g1 2

2

2−

But v2≈0

∴ Hlex =v

g1

2

2

7.6 Flow through series pipe line

(a). Between two tanks

Consider the flow of water from upper tank to lower tank through a series

(compound) pipe line as shown in figure.

Page 134: Fluid Mechanics and Hydraulic Machinery

E310/1 134

The total energy line is drawn taking into consideration of entrance loss hLen=0.5v12/y, loss

of head due to friction in pipe(1) =hf1=f1L1v1

2

2gd

loss of energy due to sudden contraction hLSe=0.5v2

2

2g, loss of head due to friction in pipe

(2) =f2 L2V2

2

2gd2

=hf2, loss of head due to sudden expansion =hLSE =(V2 − V3 )2

2g, loss of head

due to friction in pipe (3) = f2 L3V3

2

2gd3

= hF3 and loss of head at exit =hLex=V3

2

2g, respectively

from A to B, as shown in figure. From this total energy line the respective velocity heads

viz, V1

2

2g,V

g2

2

2,V

g3

2

2

in pipe (1),(2),(3) are deducted, the Hydraulic grade line is obtained which is parallel to

Total energy line which is a line joining piezo meter levels along the pipe line.

The flow through series pipe line can be solved by applying Bernoull,s’ theorem

between (A) and (B) above the datum.

Let

V1,V2,V3 - Velocitiesd1,d2,d3 - diametersL1,L2,, L3- LengthsL1,L2,, L3- Lengths

of pipes (1), (2) , (3), respectively

Then ZPw

Vg

pw

BV

ghA

A A B BL+ + = + + +

2 2

22

2

Page 135: Fluid Mechanics and Hydraulic Machinery

E310/1 135 Z Z h h h h h h hA B Len f Lse f LsE f Lex+ + = + + + + + + + + +0 0 0 0 1 2 3

( )Z Z HVg

f L Vgd

rVg

f L vgd

V VgA B− = = + + + +

−0 52 2

02 2 2

12

1 1 12

1

22

2 2 22

2

2 32. . ( )

+ +f L V

gdv

g3 3 3

2

3

32

2 2

and by continuity equation

Q= a1v1=a2v2= a 3 v 3

Using the above equations the problems of flow through series pipe line can be solved ,V13

(a) of H is given, and if it is required to find Q, then the unknowns V1,,V2, ,V3 can be

determined using the above 3 equations.

(b) of Q is given, the unknowns, V1,V2 V3 and H can be determined from above 4

equations.

(c) of the pipe lines are large the major loss due to friction is more when compared to

minors losses and so the minor losses in that case can be neglected , without any serious

error. Then the solution of the problem is simple.

(4) Flow through series pipes discharging into atmosphere

Consider the flow through series pipes discharging into atmosphere as shown in

figure.

The energy line and Hydraulic grade lines are also indicated.

This problem can be solved by applying Bernolli’s theorem and continuity equation as

explained below.

Applying Bernaulis Theorem between (A) and (B) we have

Z o o o oV

gh h h h h hA

BLen f Lsc f LsE f+ + = + + + + + + + +

2

1 2 32

Page 136: Fluid Mechanics and Hydraulic Machinery

E310/1 136

( )V

gVg

f L Vgd

Vg

f L Vgd

v Vg

f L Vgd

B B B

B

212

1 1 12

2

22

2 2 22

2

22

3 32

20 5

2 20 5

2 2 2 2+ + + + +

−+

. .

and by continuity equation we have

Q = a1v1 = a2v2 = a3v3

By using the above equation the flow through series pipe line discharging into atmosphere

can be solved as explained in series pipe line between two tanks.

7.7 Flow through parallel pipes

Let there be ‘n’ no of parallel pipelines in between two tanks as shown in figure.

Now applying Bernoulli’s theorem, between A and B over the datum, we have

ZPw

Vg

ZPw

Vg

hAA A

Bw B

L+ + = + + +2 2

2 2

ZA + 0 + 0 + = ZB + 0 + 0 + hL

ZA - ZB = H = HL = hf1 = hf2 = hf3 =--------=hfn

and Q = Q1 +Q2 + Q3+ --------------------------Qn

Using the above equations the flow through parallel pipe lines can be solved.

Here it can be noted that,

(a) in case if series pipe line loss of head is additive and discharge is constant, where as

(b) in case of parallel pipe lines, discharge is additive and Loss of head is same for all

parallel pipe lines.

7.8 Flow through branch pipes

Consider flow of fluid from tank (A) to tank (B)and tank (C) which are at lower

levels than tank (A), as shown in figure.

Page 137: Fluid Mechanics and Hydraulic Machinery

E310/1 137

Now applying Bernoull’s theorem between (A) &(D), (D)&(B), (D)&(C), We have

Z Upw

Z hAd

d f+ + = + +0 1

pw Z Z hd

d B f+ = + 2

p w Z Z hd c fθ + = + 3

Q1 =Q2+Q3 if pw

Z Z or Zd b cφ +

>

(Q1+Q2)=Q3 if (Zb)> pw

Zdφ +

The un knows Pa/w v1, v2, and v3 can be obtained by solving the above equations.

Further if fluid flows from A to B and C,. we have

ZA= ZB+hf1+hf2 - along route A to B

ie (ZA-ZB)=hf1+hf2

and ZA= Zc+hf1+hf3 along route Ato C

ie (ZA-Zc)=hf1+hf3

and φ 1 = φ 2+φ 3

Using these equations, without knowing Pd/w, the pressure at junction the flow through

branch pipes can be solved if routes of flow are known.

Identify the following as true or false.

SAQ (4) Loss of head in Laminar flow in proportional to velocity.

SAQ(5) Loss of head in Turbulent flow is proportional to v4

SAQ(6) Loss of head in Laminar flow depends on surface roughness

Page 138: Fluid Mechanics and Hydraulic Machinery

E310/1 138 SAQ(7) Loss of head in Turbulent flow is dependant on surface roughness

SAQ(8) Loss if head in pipes depends on surface area

SAQ(9) Minor losses are due to change in velocity and direction of flow

SAQ(10) Velocity head in a pipe line is 0.5m, then find the loss of head at entrance.

SAQ(11) In problem (10), find loss of head at exit.

SAQ(12) Hydraulic grade line is always falling in the direction of flow

SAQ(13) Total energy line is always falling in the direction of flow

SAQ(14) The level difference in two tanks is equal to total loss of head

SAQ(15) In series pipe line discharges are additive

SAQ(16) In case of parallel pipe line the loss of head is same for all pipes.

SAQ(17) Tank A and B are above tank C then QA+QB=Qc

SAQ(18) Friction factor is non dimensional

WE (1) Two reservoirs are connected by a pipe line which is 15 cm in dia for the first 6cm

and 25 cm in dia for the remaining length of l5 cm. There is a valve in the 25 cm dia pipe

line. When the valve is completely opened a discharge of 0.112 m3/s is flowing in the pipe

line. Determine the level difference considering minor losses also. The energy loss at

valve is equal to entrance loss of head.

Applying Bernaulic Theorem between (A) and (B) taking datum through (B) , we have

0 + 0 + H = 0 + 0 + 0 + Total Flows

H = ( )05

2 2 2 205

2 212

1 1 12

1

1 2 2 22

2

22

22. _ .V

gf L V

gdv v

gf Lv

gdVg

vg

+ + + + +

Q = a1 v1 = a2v2 ; v1 =2515

2 782

2 2

=v v.

So, H = 0.5 ×(2.78)2 vg22

2+

( )0 04 6 2 78015 2

2 78 12

0 04 150 25 2

152

222 2

22

22

22. .

.( . . ) .

..

× ×+

−+

×+

vg

vg

vg

vg

Page 139: Fluid Mechanics and Hydraulic Machinery

E310/1 139

H = 23.264 v

g22

2; a2 = ( )Π

40 25 0 0492 2. .= m

vQa

m s22

4 1120 049

2 28= = =..

. /

vg

22

2 = 0.265m

∴H = 23.264 ×0.265 =6.17m

_____

WE (2) Water flows through a 60 cm dia pipe line between two tanks whose level

difference is 20 cm of another parallel pipe of same length and dia ”d” is laid to reduce the

level difference to 10m. Find the dia of 2 nd pipe. Friction factor is same for both the

pipes. The discharge flowing is same in both cases.

Case(1) H1 =hf=

202 3 026 0 6

2 2

5= =fLV

gdfLQ

, . ( . ) (1)

Case (2) H2 = hf1 = hf2

102 3026 0 61

12

12

5=

=

fLd

vg

fL Q. ( . )

(2)

102 30262

22 2

5=

=

fLd

vg

fL Qd. ( )

(3)

Dividing (2) by (1)

1020

05 1

2

2

= =

.

QQ

ie Q1 = 0.707Q

But Q=Q1 +Q2 =Q-0.707Q= 0.273Q

Dividing (2) by (3)

1010

10 6

12

5

5

22= = ×

Q dQ( . )

QQ

d2

1

2 5

0 6

=

.

0 2730 707 0 6

0 68342 5.

. ..

QQ

d

=

=

Page 140: Fluid Mechanics and Hydraulic Machinery

E310/1 140 d = 0.41 m

WE (3)Two pipes of dia 30 cm and 60 cm are connected in parallel between two

reservoirs. The length and friction factor are same for both the pipes. Determine the level

difference if the total discharge flowing is 3 m3/s. The length of pipe is 100m and f=0.02

Solution:

hf1=hf2

fLQQ

fLQQ

12

15

22

253026 3 026. .

=

QQ

dd

2

1

22

1

5

5 22 3

=

= =

Q2/Q1 = 5.657

Q2 = 5.657Q1

But Q=Q1 +Q2 = 3.0

Q1 + 5.657 Q1 = 3.0

6.657Q1 = 3.0

Q1 = 0.451 m3/s

and Q2 = 3.0 - 0.451 = 2.549 m3 /s

so, level difference = hf1 = hf2

=0 02 100

3 026 0 312

5

.. ( . )

× × Q

= 271.99(0.451)2

= 55.32 m

WE(4) Two reservoirs have a difference of level of 10m are connected by a pipe line

which consists of 50 cm dia pipe 3000m long and then 2 parallel pipes of 25 cm dia are

laid to another length of 3000m. Find the total discharge if f=0.04

Solution:

Page 141: Fluid Mechanics and Hydraulic Machinery

E310/1 141

Route (1) & (2)

fLV

gdfL v

gd12

1

2 22

22 210+ =

0 04 30002 9 81 05

0 04 30002 9 81 0 25

1012

22.

. ..

. .× ×

× ×+

× ×× ×

=V v

v v12

222 0 8125+ = .

But Q1 = Q2+Q3 =

since hf2= hf3

Q2 =Q3

So Q1 =2Q2

Q Q2 1

12

=

( ) ( )Π Π4 4 2

5025 22

2

2 1

2 12

12d v d

v v=

=; ϑ

v2 =2v1

∴v12 +2(2v1)2 = 0.8175

9v12 = 0.8175; v1

2 = 0.0908

v m s1 0 0908 0 3= =. . /

So ,discharge Qd

v112

14=

Π( )

=Π4

(0.5)2 ×0.3

0.1963× 0.3

=0.0589 m3/s

Page 142: Fluid Mechanics and Hydraulic Machinery

E310/1 142 WE(5) A reservoir A is feeding two tanks, B and C. The level difference between A and

B is 20m and A and C is 30 m. The pipe line from A is 1500m long and 30 cm in dia upto

junction. From junction two pipe lines of 30 cm in dia feed the tanks B and C . Find the

discharge to each tank. Take f=0.04 in all pipes

Take f=0.04 for all pipes

Solution:

Route (1) and (2)

hf1+hf2= 20

Route (1) and (3)

hf1 +hf3 = 30

So 0 04 1500

2 9 81 0 30 04 15002 9 81 0 3

2012

22.

. ..

. ..× ×

× ×+

×× ×

=v v

( )1019 2012

22. h v v+ =

v v12

22+ = 1.962 (1)

and 10.19h (ϑ12 +ϑ2

2)=30

v12+v3

2 = 2.943 (2)

But by continuity equation we have

Q1 = Q2 +Q3

Π Π Π4

0 34

0 34

0 321

22

23( . ) ( . ) ( . )× = +v v v

i.e., v1 = v2 +v3 (3)

v1 = 1962 2 94312

12. . .v v+ −

Solving by trial and error

v1 = 1.37, v2 = 0.292 m/s v3 = 1.032 m/s

Q12

40 3 137 0 0707 137= × = ×

Π( . ) . . .

= 0.0968 m3 /s

Page 143: Fluid Mechanics and Hydraulic Machinery

E310/1 143 Q2 = 0.0707 × 0.292 = 0.02 m3 /s

Q3 = 0.0707 × 1.032 = 0.073 m3 /s Summary

(1) Reynolds number. is represented by RN =ρ

µvD

where, ρ is mass density

µ viscosity

υ Kinematic viscosity

(2) Initial Reynold’s number = 2000

below which the flow is laminar

(3) loss of head in turbulent flow = hfLV

gdf =2

2is known as Darcy = Weis Bach

equation where ‘f’ is called as coefficient of friction or friction factor.

(4) Pipes in series.

Energy difference H = total losses

Q1 = Q2 = ----------------- = Qn

(5) Parallel pipes

hf1 = hf2 ------------------ = hfn = H

Q = Q1 +Q2 --------------+Qn

(6) Branch pipes for tanks A,B,C

ZA = 2d+pd/w + hf1 where pw

d is pr head at junter

pw

Z Z hdd B f+ = + 2

pw

d +2d= Zc+hf3

QA= QB +Qc

Answers to SAQ

1 & 2 = True 3. 10cm/3 4. & 5 True 6. False

7. - 9. True 10. 0.25 m 11. 0.5 m 12. False

13 & 14. True 15. False 16. - 18 True

Exercise

Page 144: Fluid Mechanics and Hydraulic Machinery

E310/1 144 7.1 A compound pipe of total length 690 m connects two reservoirs. Water surface in

one reservoir is at elevation 601.2 and in the other at elevation 610.2. There is 300m of 30

cm pipe l50 m of 20 cm pipes, and then 240m of 25 cm pipe, 25 cm pipe is connected to

the lower tank. The changes are sudden (Cc =0.62 ) and friction factor of 30 cm, 20 cm,

and 25 cm are 0.02,0.022 and 0.0213 respectively. Determine (a) Discharge, (b) Various

losses and (c) draw the hydraulic gradient.

[Q= 0.076 m3/s, hLen = 0.0224,

hf1 = 1.19m, hLgc = 0.113m, hf2 = 5.0m hLse = 0.039m

hf3 = 2.52 m, Lhex = 0.12m]

7.2 A pipe 20 cm dia is 20 cm long and the velocity of the water in the pipe is 8m/s.

What loss of head would be saved if the central 6m length of pipe is replaced by30cm dia

pipe, the change of section being sudden (f = 0.04, Cc = 0-62)

[ 1.17m]

7.3 Two reservoirs are connected by a pipe 20 cm dia and 3000m long, the difference

in surface being 15m. Calculate the discharge in lps.

If a loop line 30 cm dia and 1200m long is connected to last 1200m of pipe,

calculate the increase in discharge in Lp on due to addition of loop line. Neglect all the

losses other than due to fraction.

F = 0.03

[1476 lpm, 386 lpm]

7.4 Two pipes with dia 2D and D are first connected in parallel and when a discharge Q

passes the loss of head is H1. When the same pipes are connected in series for the same

discharge, the loss of head is H2.. Find the relationship between H1 and H2. Neglect minor

losses. Both pipes are equal in length and have the same friction factor.

[A1 = 45.7 H2 ]

7.5 Two reservoirs are connected by three pipes laid in parallel, then dia are respectively

d, 2d, and 3d and they are all of the same length L. Assuming to be the same for all the

pipes, what will be the discharge through the larger pipes if that through the smallest is

one m3/s

[5.66 & 15.58 m3/s]

7.6 Three tanks A,B,C are connected by a pipe system. The elevation of A is 33.00 and B

is 39.88m. The discharge from A is 0.07 m3/s Determine the rate of flow into or from

Page 145: Fluid Mechanics and Hydraulic Machinery

E310/1 145 the reservoirs B and C, Find also the R.L of water level in the reservoir C, Given f =

0.030

Pipe length dia

A to junction 306 cm 30 cm

junction to B 250 cm 25cm

Junction to C 300 m 20 cm

[ Discharge from B = 0.03 m3/s , Q w to c = 0.1 m3 /s

RL of water level is C = 24.75 m]

***

FLUID MECHANICS AND HYDRAULIC MACHINERY

UNIT 8

Laminar Flow and Turbulent Flow

AIM: In this unit the aims are to derive expressions for velocity distribution and loss of

energy in laminar flow through pipes and parallel plate and to distribute between smooth

and rough pipes.

Objectives:

1. To derive expressions for velocity distribution over the cross sectional area of flow, loss of

head and shown stress at boundary for laminar flow through parallel plates, both plates at

rest.

2. To derive an expression for velocity distribution for laminar flow through parallel plates,

one plate at rest and other moving. This flow is called as couette flow.

3. To derive expressions for velocity distribution over the cross sectional area of flow , loss

of head and shear stress at boundary for laminar flow through pipe.

4. To distinguish between smooth and rough pipes in case of turbulent flow through pipes.

5. To derive a condition for the pipes to be hydraulically smooth and rough.

6. To derive expressions for velocity distribution over the cross-sectional area of flow,

friction factor for smooth and rough pipes.

7. To explain the useful moody’s diagram.

Page 146: Fluid Mechanics and Hydraulic Machinery

E310/1 146 To use the above in solving practical problems.

Laminar Flow:

8.1 Introduction:

We know in the Reynold’s experiments that in case of laminar flow the viscous forces

are predominant over inertia forces and so the layers slide one over the other. Due to

viscosity of fluid, there will be opposition to motion which leads to loss of head over a

length of flow. The movement of layers leads to shear stress in between layers, whose

magnitude varies from point to point. Based on these theories the laminar flow

in studied, between parallel plates and horizontal pipes. This analysis is useful in solving

the problems of laminar flow through parallel plates and pipes.

8.2 Laminar flow between parallel plates at rest:

Consider a steady and uniform laminar flow of fluid between two parallel plates at rest

‘B’ distance apart as shown in Figure. In this flow consider a small elemental volume of

thickness ‘dy’ at a distance ‘y’ from the lower plate, of ‘dx’ m long. Let the width of flow

be 1 unit length. The forces acting on this volume are pressure and shear forces as shown

in figure.

Fig.

As the flow is steady and uniform total forces acting on the volume

∑ F = 0

ie., pdy − p +∂p∂x

dx

dy − τdx − τ ×

∂τ∂y

dy

dy

= 0

Page 147: Fluid Mechanics and Hydraulic Machinery

E310/1 147

∂p∂x

dx dy =∂τ∂y

dy dx

or ∂p∂x

=∂τ∂y

But according to Newton’s law of viscosity

τ = µ dvdy

Substituting this in the above equation we have

dpdx

= µ d2vdy2

Integrating this equation w.r.t y we have

dvdy

=1µ

dpdx

y + C1

v =1

2µdpdx

y2 + C1y + C2

Using the boundary conditions

v=0 at y=0

v=0 at y=B , we have

0 =1

2µdpdx

× 0 + 0 + C2

∴C2=0

C1 = −1

2µdpdx

B

∴ v =1

2µdpdx

y2 −

12µ

dpdx

By

=1

2µ−

dpdx

By − y2

This equation gives velocity distribution over the cross sectional area of flow.

Velocity is maximum at y=B/2.

.

i.e. vmax =1

2µ−

dpdx

B2

y

=B2

8µ−

dpdx

Page 148: Fluid Mechanics and Hydraulic Machinery

E310/1 148 Mean Velocity:

To obtain mean velocity consider the elemental discharge through the volume as

dq = v dy

Integrating this

q =1

2µ−

dpdx

By − y2( )dy

0

B

=1

2µ−

dpdx

By2

2−

y3

3

0

B

=1

2µ−∂p∂x

B3

6

=

B3

12µ−∂p∂x

The mean velocityV =q

C.S.Area flow=

qB ×1

V =B2

12µ−∂p∂x

then V =23

Vmax

Loss of heat:

From mean velocity we have

−∂p∂x

=

12µVB2

−dp( )p1

p2

∫ =12µV

B20

L

∫ dx

p1 − p2( )=12µVL

B2

∴hf =p1 − p2( )

w=

12µV LwB2

which is proportional toV .

Shear Stress at boundary:

We know the shear stress at y is given by

τ=µdvdy

Page 149: Fluid Mechanics and Hydraulic Machinery

E310/1 149

= µ ddy

12µ

−∂p∂x

By − y2( )

τ =12

−dpdx

B − 2y[ ]

τ = τ0 at y = 0, τ = τ0 at y=B and τ = 0 at y = B/2.

∴τ 0 =12

−dpdx

B at y = 0

=12

−dpdx

−B( ) at y = B

=12

−∂p∂x

0( ) = 0 at y =B2

This shows that shear stress is maximum at boundary and zero at the centre.

Further then −dpdx

=

12µVB2

∴τ 0 = ±12

12µVB2

B =

6µVB

Identify the following as true or false:

SAQ(1). ∂p∂x

=∂p∂y

is difficult for steady and uniform flow.

SAQ(2). Mean velocity in case of laminar flow, between parallel plates which are at rest is

2/3 Vmax

SAQ(3). The pressure difference over a length L for laminar flow through parallel plates at

rest which are ‘B’ apart, is 12µV L

B2

SAQ(4). The shear stress at boundary for laminar flow through parallel plates at rest is

±6µV

B

SAQ(5). In case of steady laminar flow through parallel plates at 10 cm apart the pressure

gradient in the direction of flow is -40 KN /m2 /m. Find the maximum shear stress.

WE(1) Between two parallel plates at 0.1 m apart laminar flow of nil of viscosity 2.453

NS / m2 is taking place with 58.872 KN / m2 pressure drop over 20m long plates. Calculate

the discharge per m width, the shear stress at boundary , mean and maximum velocity and

velocity at 0.02 m from the boundary.

Page 150: Fluid Mechanics and Hydraulic Machinery

E310/1 150

Solution:

−∂p∂x

=p1 − p2

L=

58.872 ×100020

= 2943.6N / m 2 / M

Discharge / m width = q = B 2

12µ−

∂p∂x

=(0.1)3

12 × 2.45358.872 ×1000

20

= 0.1 m3/p

Mean velocity V =q

B ×1=

0.10.1

= 1m / p

=B 2

12µ−

∂p∂x

=(0.1)2

12 × 2.45358.872

20

= 1 m / A

Max velocity Vmax = 3 / 2 Vmea = 1.5 ×1 = 1.5 m / D

Velocity at 0.2 m = 1

2µ−∂p∂x

By − y2( )

=1

2 × 2.453(2943.6) 0.1 × 0.02 − (0.02)2

= 0.96 m / D

Shear stress at boundary = τ0 = −∂p∂x

B2

= 2943.6 ×0.1 / 2

= 147.18 N2 / m2

8.3. Laminar flow through parallel plate one plate stationary and other plate at rest:

Consider steady and uniform flow through parallel plates at ‘B’ m apart, bottom

plate being stationary and tap plate moving with velocity V m/s. Let the width of flow be

1 unit length. Consider an elemental volume of ‘dy’ thick at ‘y’ from the bottom plate of

length ‘dx’.

Page 151: Fluid Mechanics and Hydraulic Machinery

E310/1 151

Fig.

This type of flow is called as couette flow.

The forces that are acting over the elemental volume are shown in Figure.

Now as the flow is steady and uniform the total force acting over it are zero as the

acceleration is zero.

∴SF = 0

i.e as in the previous case, here also we have

∂p∂x

=∂p∂y

∂p∂x

=ddy

µ ∂v∂y

= µ ∂2v

∂y2

Integrating this twice using the B.C

v =1µ

∂p∂x

y2

2+ C1y + C2

using the B.C v = 0 at y = 0

v = V at y = B

C2 = 0

C2 =1µ

−∂p∂x

B2

+VB

∴ v = −1

2µ∂p∂x

(By − y2) +

VB

y

Thus the velocity distribution in couette flow depends on both pressure gradient in the

direction of flow and velocity of plate.

Here the pressure gradient may be either +ve or -ve.

Page 152: Fluid Mechanics and Hydraulic Machinery

E310/1 152

Case(1) When dpdx

= 0

v= vB

y -Simple or plain couette flow in which velocity distribution is linear with y.

of V=0 , then velocity distribution is same as that of laminar flow through stationary

parallel plates. This shows that parallel plates. This shows that couette flow is a

superposition of simple couette flow and laminar flow through parallel plates at rest.

The non dimensional form of velocity distribution for general couette flow is

uV

=yB

+1

2µ−∂p∂x

By2 − y2

V

=yB

+B2

2µv−∂p∂x

1 −

yB

yB

Let B2

2µv−∂p∂x

= P

Then uV

=yB

+ P 1 −yB

yB

Now the velocity distribution for different values of P are shown in Figure.

Fig.

Case 2: ∂p∂x

=+ve i.e., P=-ve, i.e., increase in pressure in the direction of flow in this case

for some layers the velocity is -ve(opposite to V)

Page 153: Fluid Mechanics and Hydraulic Machinery

E310/1 153

Case 3: When ∂p∂x

=-ve , i.e., P = +ve there will be pressure drop in the direction of flow

and there will be no backward flow.

Shear distribution:

We know shear stress at any depth y is

τ = µ dudy

= µ ∂∂y

VB

y −1

2µ∂p∂x

By − y2( )

= µ VB

−1

2µ∂p∂x

B − 2y( )

= µ VB

+ − ∂p∂x

B2

− y

Which shows that P varies linearly with y.

At y = 0 τ01=µVB

+ −∂p∂x

B2

At y=B/2 τ =µVB

At y = B τ02=µVB

+ −∂p∂x

B2

− B

=µVB

− −∂p∂x

B2

τ = 0 at µVB

=∂p∂x

B2

− y

y =B2

+

µVB

−∂p∂x

Page 154: Fluid Mechanics and Hydraulic Machinery

E310/1 154

Fig.

At this y, the velocity will be maximum.

Identify the following whether true or false:

SAQ(6). In case of plain couette flow velocity distribution is linear with y i.e.,

v = V y/B.

SAQ(7). At B / 2 , in couette flow where B is the distance between two parallel plates, the

shear stress is µ V / B, V is velocity of Tap plate .

SAQ(8). Backward flow occurs in same layers of couette flow if ∂p∂x

= +ve.

SAQ(9). Couette flow is a superposition of simple couette and laminar flow through fixed

parallel plates.

SAQ(10). In couette flow V = 10u / s, the plates are at 10 cm apart, determine the distance

of maximum velocity from fixed plate if ∂p∂x

= -20 Kn/m2/m and viscosity µ = 2 µ-S /

m2.

WE(2). In the previous problem if the tap plate moves at 10 m ./ s, determine the

maximum velocity, its distance from the flow plate and the shear stress over plates and

velocity at 0.02m from fixed plate.

if −∂p∂x

= 20 Kn / m2 / m.

Page 155: Fluid Mechanics and Hydraulic Machinery

E310/1 155 Solution:

y =B2

+µVB

1

−∂p∂x

; −∂p∂x

= 20,000N / m2 / m

y =0.12

+2.453 ×10

0.11

20,000

= 0.05+0.0122 = 0.0622 m

Vmax = Vs

y +1

2µ−∂p∂x

By − y2( ) at y= 0.00622 m.

Vmax = 10 × 0.0622

0.1+

12 × 2.453

× 20,000 0.1× 0.0622 − 0.0622( )2[ ]= 6.22 + 9.588 = 15.8 m/s.

Vol at 0.02 m from fixed plate

v at y = 0.02 = VB

× 0.02 +1

2µ−∂p∂x

By − y2( )

=100.1

× 0.02 +1

2 × 2.543× 20,000 0.1 × 0.02 − 0.02( )2( )

= 2+6.29 = 8.29 m/s.

τ at y=0 =µVs

+ −∂p∂x

B2

=2.543 ×10

0.1+ 20000 ×

0.12

=254.3+1000 = 1254.3 N / m2

τat y=0.1 , =µVB

− −∂p∂x

B2

= 254.3 - 1000 = -745.7 N / m2

8.4 Steady Laminar flow through horizontal pipe Hagen- Porsenille law:

Consider a concentric cylinder of ‘r’ radius and ‘dx’ long as shown in figure in a

laminar flow through a horizontal pipe of diameter D.

Page 156: Fluid Mechanics and Hydraulic Machinery

E310/1 156

Fig.

By applying Bernoulli’s equation between (1) and (2) we have,

p1w

+ z1 + v12

2g= p1

w+ z2 + v1

2

2g+ hf

i.e hf =p1 − p2

w

Qv1=v2 , z1=z2

The forces acting on the elemental volume are shown in figure. Then for steady flow, ∑F

= 0 , Qacceleration is zero, we have

pΠr2 − p +∂p∂x

dx

Πr2 = τ2Πrdx

τ = −∂p∂x

r2

But for laminar flow we know that

τ = µ dvdy

where y = R-r is the distance form boundary.

So dy = - dv

i.e., τ = −µ dvdr

= −∂p∂u

r2

i.e., dv =12u

∂p∂x

r dr

Integrating and substituting the B.C v = 0 at r = R, we have

Page 157: Fluid Mechanics and Hydraulic Machinery

E310/1 157

v =

14µ

∂p∂x

r2( )+ C

and C = −1

4µ−∂p∂x

R2( )

∴ v =1

4µ−∂p∂x

R2 − r2( ) is the velocity distribution over the cross

section of flow, which is parabolic. Velocity is maximum at r = 0

Fig.

i.e., vmax =1

4µ−∂p∂x

R2

=1

16µ−∂p∂x

D2

Then V = vmax 1−rR

2

Mean Velocity:

To get the mean velocity, discharge is to be calculated based on velocity

distribution. So consider at radius ‘r’ a ring of thickness ‘dr’ as shown in Figure.

Fig.

The discharge passing through this ring area is

dQ = dav = 2Πrdr ×1

4µ−∂p∂x

R2 − r2( )

So by integrating we have

Page 158: Fluid Mechanics and Hydraulic Machinery

E310/1 158

Q = dQ = 2Πrdr 14µ

− ∂p∂x

R2 − r2( )

0

R

∫∫

Q = 2Π1

4µ−∂p∂x

R2 r2

2−

r4

4

0

R

8µ−∂p∂x

R4

128µ−∂p∂x

D4

So the mean velocity is v

v = Qa

= Π8µ

− ∂p∂x

R4 × 1ΠR2

=1

8µ−∂p∂x

R2

=1

32µ−∂p∂x

D2

So v =12

vmax

Fig.

Further v = v at a radius r given by

vmax2

= vmax 1 − rR

2

i.e.,rR

=12

∴ r = 0.707 R

Page 159: Fluid Mechanics and Hydraulic Machinery

E310/1 159 Loss of head:

We know that the mean velocity is given by

v =1

32µ−∂p∂x

D2

− ∂p∂x

= 32µvD2

−∂p( )p1

p2

∫ = 32µvD2

0

L

∫ dx

p1 − p2( )=32µvL

D2

orp1 − p2( )

w= hf =

32µvLwD2 is known as Hagen Poisemille equation.

But hf =fLv2

2gD=

32µvLwD2

Then f =64µρvD

=64Rn

where Rn =Raynolds no = ρvD

µ

So for laminar flow, the friction factor is 64Rn

Shear stress:

The shear stress at any radius ‘r’ is given by

τ = −∂p∂x

r2

So at boundary τ0 = −∂p∂x

R2

( )=

=

p pL

D

whL

Df

1 2

4

4

=wL

fLvgD

D2

2 4

Page 160: Fluid Mechanics and Hydraulic Machinery

E310/1 160

τρ

0

2

8=

f v

Then

τ0ρ

= vA = shear velocity

vA = vf8

Power required to maintain the laminar flow is

P = F × v

= τ × A × v

=whf

2D4

× πDL × v

= w πD2

4v

hf

= wQhf N-m / sec or Kg-m / sec

Also P = Force ×v

=(p1-p2)A ×v

=(p1-p2) Q N-m / sec or Kg-m / sec

SAQ(11). Determine the shear stress at wall in case of a pipe of 10 cm diameter in which

laminar flow is taking place, given the -ve pressure gradient in the direction of motion as

20 KN / m2 / m.

SAQ(12). Determine the maximum velocity in a pipe of 10 cm diameter in which laminar

flow is taking place . Given -ve pressure gradient in the direction of motion as 2 KN / m2 /

m and µ=2.5 NS / m2.

SAQ(13).A laminar flow is taking place at 2 lps in pipe of 0.01 m2 C.S. Determine the

maximum velocity.

SAQ(14). Find the velocity in a pipe of 10 cm diameter at a radius of 2 cm from centre, in

which diameter flow is taking place. Given max velocity as 0.5 m / s.

SAQ(15). Find the radius at which local velocity is equal to mean velocity.

SAQ(16).If Reynolds no =1280 , find the friction for laminar flow through pipe.

SAQ(17).Pressure drop over a length in a pipe in which the flow is laminar, is

2 KN / m2 .Find the power lost, if the ratio of flow is 2 lps.

Page 161: Fluid Mechanics and Hydraulic Machinery

E310/1 161 SAQ(18). Given the pressure drop over 20m of pipe is 200 KN / m2 in a pipe of 10 cm

diameter in which the flow is laminar. Find the shear stress at wall.

WE(3). Under oil of dynamic viscosity 1.5 poise and specific gravity 0.9 flaws through a

20 mm diameter vertical pipe .Two pressure gauges have been fixed at 20m apart. The

pressure gauges fixes at higher level reads 200 KN / m2 and that at lower level reads 600

KN / m2 .Find the direction of flow and rate of flow. Verify whether the flow is laminar.

Then find the power lost in overcoming the friction.

Solution Taking datum through A

HA = Total energy at A

= pAw

+ zA + vA2

2g

=600 ×10000.9 × 9810

+ 0 +vA

2

2g

= 67.95 +vA

2

2g

Fig.

HB = Total energy at B

= pBw

+ zB + vB2

2g

=200 ×10000.9 × 9810

+ 20 +vB

2

2g

= 42.65 +vB

2

2g

Page 162: Fluid Mechanics and Hydraulic Machinery

E310/1 162 Q VA = VB ,HA > HB

∴ flow is from A to B (upward).

Loss of head is hf = HA - HB

= 67.95 +VA

2

2g

− 42.65 +

VB2

2g

= 25.30 m.

Assuming laminar flow

hf = 25.30 = 32µLvwD2 =

32 × 1.510

× 20 × v

0.9 × 9810 × 0.02( )2

= 27.18 v.

∴ v =25.3027.18

= 0.93 m / s.

Q =π4

0.02( )2 × 0.93 = 2.92 × 10−4m3 / s

RN =ρVD

µ=

0.9 × 98109.810

×0.93 × 0.02

0.15

= 111.6 < 2000

So the flow is laminar .

Power lost = WQ Hf =0.9 × 9810 × 2.92 ×10−4 × 25.3

= 65.3 Watts.

8.5. Turbulent flow- Smooth and rough pipes

We know that for flow through pipes when Reynolds no is > 4,000 the flow is

turbulent. In this case analytical treatment is very different due to fluctuations in the

velocity in the turbulent flow and so in the deviation of velocity distribution over the C.S

of flow, experimental values are to be considered. Here due to mixing the velocity

distribution is almost uniform over the C.S of flow as shown in Figure.

Page 163: Fluid Mechanics and Hydraulic Machinery

E310/1 163

Fig.

Due to mixing there will be some change in momentum which offers shear

resistance to flow. In turbulent flow there will be fluctuations in velocity. Let in 2.D flow

the fluctuations in velocity be Vx and Vy in x ans y direction respectively. Now if a fluid

mass ρAvy while moving over C.S of flow ‘A’ in ‘y’ direction fluctuates with velocity Vx

in x direction. Therefore the momentum change is ρAVyVx which is equal to shear stress

over C.S area A.

∴Shear stress τ =Forcearea

=ρAvx vy

A= ρvxvy

The transverse distance in which the fluctuating velocity of a lamp is equal

to mean velocity is called as mixing length ‘l’ by Prandtle in 1925. This fluctuating

velocities are related to mixing length by Prandtle as follows.

vx = ldvdy

= vy

Where v is the mean velocity.

Here vy is of the same order as vx.

∴Shear stress in turbulent flow is ρvxvy

τ = ρl2 dvdy

2

Further Prandtle assumes the mixing length proportional to ‘y’ transverse distance

from boundary i.e.,

l ∝ y

or l = k y

Where k is called Karmen’s constant ‘Roppa’

So τ = ρk2y2 dvdy

2and for small values of y , τ = τ 0

Page 164: Fluid Mechanics and Hydraulic Machinery

E310/1 164

∴τ 0 = ρk2y2 dvdy

2

τ0p

= k2y2 dvdy

2

dvdy

= 1ky

τ oρ

= v0ky

where vo =Shear velocity = τ0ρ

Now by integrating we have

v =v0k

loge y + C -------------------A

which is a logarithmic velocity distribution in turbulent flow.

Smooth and rough pipes

In fluid flows over a boundary, Prandtle recognises that there would be a small

layer adjacent to the boundary in which velocity varies gradually from zero at boundary to

free stream velocity (undisturbed velocity), as shown in Figure.

Fig.

This layer is known as boundary layer. The thickness of this layer normal to the flow , at

which the velocity variation is with 1% of free stream velocity, is called as boundary layer

thickness ‘S’. When the flow in the boundary layer is of laminar nature , then it is called as

laminar boundary layer and if turbulent nature , it is called as turbulent boundary layer .

But very near to the boundary even in turbulent flow , in a very thin layer fluid can not

maintain normal velocity component and so the flow will be always laminar. Then this

thin layer is turbulent boundary layer, adjacent to the boundary is called as laminar

sublayer whose thickness is ‘s’.

Page 165: Fluid Mechanics and Hydraulic Machinery

E310/1 165

Fig.

But as shown in fig any boundary surface will have certain irregularities rendering the

surface a roughness with average height ‘k’ where these roughness projections ‘k’ are

complete by submerged by laminar sublayer then the flow is not affected by the surface

roughness. Such pipe is called a hydraulically smooth pipe . If the roughness projections

project into the flow by penetrating the laminar sublayer , then such pipe is called as

hydraulically roughness pipe as the flow is disturbed by the roughness projections.

Velocity distribution in smooth and rough pipes:

We have already obtained an expression for the velocity distribution over a cross

section of flow over a boundary , in turbulent flow is given by v =v∗k

loge y + c

But at y = 0 , v is equal to ‘-α‘ indirectly that the velocity must be zero at certain distance

y’ normal to the boundary as shown in fig.

Fig

i.e., at y = y’ v = 0

∴ = −

C

vk

yelog '

So the velocity distribution is given by

Page 166: Fluid Mechanics and Hydraulic Machinery

E310/1 166

v =v∗k

logeyy'

Of y’ is known for both smooth and rough pipes, then the velocity distribution is

known for turbulent flow through pipes.

Before arriving at this distance y’ , we have to remember that there is a laminar

sublayer in the turbulent flow , adjacent to the boundary , in which the flow is laminar

type. In this layer then the velocity distribution is parabolic . But as the layer is very thin it

can be considered as linear with y.

Fig.

As shown in fig there will be a transition layer normal to the surface in between

laminar flow , in which the velocity distribution varies gradually from logarithmic to

parabolic. But in the absence of clear cut demarcation between each zone, the

intersection point of these two velocity distributions can be assumed to be the laminar

sublayer itself.

Velocity distribution laminar sublayer:

We know that the shear stress at boundary in laminar flow is given by

τ0 = µ vy

Page 167: Fluid Mechanics and Hydraulic Machinery

E310/1 167 ie., v = τ0

= ρτ0ρ

where τ 0ρ

= v∗

vv∗

= ρv∗yµ

=v∗y2

Velocity distribution in smooth pipes

By Nikaradse’s(student of Parandtle) experiments on flow through sand coated

pipes , the parameter v yvs = 116. at y =δ’

i.e.,v∗δ '

v= 11.6

or δ ' =11.6v

v∗− − − − − − − (1)

and at y = y' ,v∗y'

v= 0.108

i.e., y' =0.108v

v∗− − − − − −(2)

But we have from equation (1)

vv∗

=δ'

11.6

substituting y = g this in equ (2) We have y’ =0.108 ×δ’ / 11.6=δ’ / 107

Substituting this value of y’ in the velocity distribution

v =v∗k

logeyy1

, we have using k = 0.4

v =2.30.4

v* log10v∗y

0.108v

vv∗

= 5.75 log10v∗yv

+ 5.75log10(9.254)

= 5.75log10v∗y

v

+ 5.5

This equation is known as Karman -Prandtl equation for velocity distribution for

hydrautically smooth pipes. The velocity distribution for smooth pipe may also be given

Page 168: Fluid Mechanics and Hydraulic Machinery

E310/1 168 by an exponential equation empirically for v*y / v between 70 and 700 as

vv∗

= 8.74v∗yv

17

Velocity distribution in rough pipes

Nikuradse and others found by experiments that the roughness height k ∝y| and

y|= k / 30.

Now substituting k = 0.4 and y| = k / 30 in v =v∗k

logey′y

we have

vv∗

=2.30.4

log1030y

k

= 5.75log10yk

+ 5.75log10(30)

vv∗

= 5.75log10yk

+ 8.5

which is known as Karman-Prandtl equation for velocity distribution in rough pipes.

Criteria for smooth and rough pipes

This criteria depends on relative magnitude of laminar sub layer and roughness

hight, i.e., k′δ

=k

11.6vv∗

=v∗kv

×1

11.6

i.e.,k′δα

v∗kv

By Nikuradses experiments it was found that when v∗kv

≤ 3 or k′δ≤ 0.25 the pipe is

hydrautically smooth, when 3 <v∗kv

< 70 or 0.25 ≤k′δ

< 60 then the flow is transition

and when v∗kv

≥ 70 or k′δ≥ 6.0 the pipe is called as hydraulically rough pipe.

Mean velocity for smooth and rough pipes:

To get the mean velocity discharge is to be calculated base on the velocity

distribution as explained below.

Page 169: Fluid Mechanics and Hydraulic Machinery

E310/1 169

Fig.

Consider an elemental ring of ‘dr’ thickness at radius ‘r’ in a pipe of radius R.

The discharging following in this elemental area is dQ = 2Πr dr v.

Q = 2πr v∗ 5.75log10v∗yv

+ 5.5

0

R

is the discharge through smooth pipe.

But y = R - r.

∴Q = 2πv∗ 5.75log10v∗(R − r)

vrdr + 5.5rdr

0

R

So mean v =Q

πR2 = v∗ 5.75log10v∗Rv

+1.75

i.e., v

v∗= 5.75log10

v∗Rv

+1.75

is mean velocity for smooth pipe.

Similarly using equation for velocity distribution for rough pipes , the mean velocity in

rough pipes is given by

v = 1πR2 2πrv∗ 5.75log10

(R− r )k

+ 8.5

0

R

dv

vv∗

= 5.75log10Rk

+ 4.75

Subtracting these mean velocity distribution in the respective local velocity distribution we

have

v − vv∗

= 5.75log10yR

+ 3.75 is identical to both smooth and rough pipes.

At y = R , v = vmax

Then for both smooth and rough pipes we have

Page 170: Fluid Mechanics and Hydraulic Machinery

E310/1 170

vmax − vv∗

= 3.75

v − vmaxv∗

= 5.75log10yR

Friction factor for smooth and rough pipes:

By dimensional analysis we have fraction factor in turbulent flow is function of RN

and relative magnitude roughness height to diameter of pipe , i.e.,

f = φvDv

k

D

(a) For laminar flow f = 64 / RN for RN ≤2000

(b) For turbulent flow:

(i)Blasius developed

f =0.316

RN( )1/ 4 for RN =4000-105

(ii) For RN > 105 ‘f’ is developed as below. The velocity distribution for smooth pipes is

vv∗

= 5.75log10v∗R

v

+1.75

Substituting v∗ = v f8

in above

v

v f8

= 5.75log10v f

8 R

V

+1.75

1f

= 2.03log10 RN f( )− 0.91

But by Nikuradse experiments , it is corrected as

1f

= 2 log10 RN f( )− 0.8 for RN = 5 ×104 − 4 ×107

This is known as Kurmon-Prandtl resistance equation for smooth pipes, which is to be solved

by Trial and error .

Further Nikuradse gave another equation for the same as

f = 0.0032 +0.221

RN( )0.237

(c) For rough pipes friction factor is obtained as below.

Page 171: Fluid Mechanics and Hydraulic Machinery

E310/1 171 The velocity distribution for rough pipes in turbulent flow is

vv∗

= 5.75log10Rk

+ 4.75

Substituting v∗ = v f8

, in above we have

v

v fRk

8

5 75 4 7510=

+. log .

( )12 03 0 9110f

R fN= −. log .

which is corrected by Nikuradse’s data as

1f

= 2 log10Rk

+1.74

which is known as Kurman-Prandtl resistance equation .

Thus in smooth pipes ‘f’ depends on Reynold’s no. only where as in rough pipes it is

independent of RN but depends on (R / k) only. So criterion for smooth and rough pipes is

RN f

Rk

< 17 smooth pipes

> 400 Rough pipes

=17-400 Transition pipes.

For any commercial pipe , the friction factor ‘f’ can be obtained by L.F Moody, diagram

shown below on log-log scale , knowing RN and relative magnitude of radius of pipe ‘R’

or roughness height ‘k’ i.e., (R / k). The roughness height of commercial pipes can be

obtained by equating the loss of energy of both sand coated pipes and commercial pipes.

Page 172: Fluid Mechanics and Hydraulic Machinery

E310/1 172

Fig.

Identify the following as true or false:

SAQ(19) In turbulent flow the velocity distribution is logarithmic distribution.

SAQ(20). If laminar sublayer submerges the roughness , then it is called as smooth pipe, if it

projects out of the sublayer, it is called as rough pipe.

SAQ(21). In turbulent flow, the velocity is zero at certain depth from boundary for

logarithmic velocity distribution.

SAQ(22). The velocity distribution is zero at y| = S| / 107 where S| is the laminar sublayer

thickness, in smooth pipes.

SAQ(23). For rough pipes roughness value k = 3. y ‘ where y| is the depth of flow from

boundary at which velocity is zero.

SAQ(24). The criterion to distinguish between smooth and rough pipes, depends on relative

magnitude of laminar sublayers and roughness height.

SAQ(25).The difference of local velocity and mean velocity in turbulent flow is identical for

both smooth and rough pipes.

SAQ(26). Difference of Maximum and mean velocities in turbulent flow through pipes is

3.75×v* .

SAQ(27). Friction factor in turbulent flow is function of RN and R / k, relative magnitude of

radius and roughness height.

Page 173: Fluid Mechanics and Hydraulic Machinery

E310/1 173 SAQ(28). For rough pipes ‘f’ is independent of RN.

SAQ(29). For smooth pipes ‘f’ is dependent on RN but not on roughness height of pipe.

SAQ(30). Same pipe will behave smooth and rough if v is varied.

SAQ(31). Given v* =0.5 m / s in a turbulent flow v = 0.01 stokes, roughness height k=1.0 mm

determine whether the flow is smooth or rough.

WE(4) For turbulent flow in pipes , show that vmax

v= 1.33 f +1

Sol:

The velocity distribution for turbulent flow in pipes is

v − vv∗

= 5.75log10yR

+ 3.75 for both smooth and rough pipes.

Now at y = R , v = v max

sovmax − v

v∗= 3.75

substituting v∗ = v f8

vmax − v

v f8

= 3.75

vmaxv

= 3.75 f8 +1

vmaxv

= 1.33 f +1

But by Nikuradse experiments

vmaxv

= 1.43 f +1

WE(5) A turbulent flow of water is flowing in a pipe of 10 cm diameter with roughness height

0.5 mm with a mean velocity of 10 m / s. Given v = 0.01 stokes. Find whether the flow is

smooth or rough. If it is rough for what velocity it will behave like smooth pipe. f = 0.02 .

Find the decmeter of pipe.

Solution:

Criterion for smooth and rough pipe is v∗kv

Page 174: Fluid Mechanics and Hydraulic Machinery

E310/1 174

v∗ = vf8

= 10 ×0.02

8= 0.5m / s

So v∗kv

=0.5 ×100 × 0.5

0.01= 250 > 70

∴The pipe is rough pipes.

So for smooth pipe v∗kv

≤ 3

or v∗ =3 × v

k=

3 × 0.010.05

= 0.6 = 0.05 × v cm / s

v =0.6

0.05= 12cm /s.

Further for smooth pipes

RN f

Rk= 17

i.e., RN f = 17 × Rk

But for smooth pipe

1f

= 2.0 log RN f − 0.8

= 2.0 log17 × R

k

− 0.8

1

0.02= 2.0 log

17 × R0.05

− 0.8

Where R is in cm.

7.07 = 5.06 - 0.8 + 20 log (R)

= 4.26 + 2 log ( R )

log R = 2.8

R = 25 cm.

WE(6)

The velocities in a 30 cm pipe carrying oil are 4.5 m / s and 4.2 m / s on the central line

and at a radius of 5 cm from the axis. Calculate the discharge and shear stress at boundary.

Solution:

For smooth and rough pipes

v − vv∗

= 5.75logRy

Page 175: Fluid Mechanics and Hydraulic Machinery

E310/1 175

4.5 − 4.2( )

v∗= 5.75log

0.150.15 − 0.5

=1.01252.

v* =0.296 m / s = v f8

But mean velocity is related to vmax as vmax − v

v∗= 3.75 for both the smooth and rough pipes.

Now at y = R , v = vmax

So vmax − v

v∗= 3.75

Substituting v∗ = vf8

vmax − v

v f8

= 3.75

4.5 − v0.2963

= 3.75

∴v = 3.389 m / s.

Q =πD2v

4

=π4

0.3( )2 × 3.389

= 0.2395 m3 / s.

Further v* =v f8

0.2963 = 3.389 f8

f = 0.0612

Shear stress at boundary τ0:

v∗ = τ0ρ = 0.2963

τ0 = (0.2963)2 ×ρ

= (0.2963)2×1000 N / m2

= 87.79 N / m2

Page 176: Fluid Mechanics and Hydraulic Machinery

E310/1 176 WE (7). A 1000m long pipe line of diameter 0.3 m carries oil at the rate of 540 lps. of the

specific gravity of oil is 0.8 and Kinematic viscosity is 0.023 stokes, determine the loss of

head, max velocity and at 10cm from axis and the maximum roughness height uph which

it behaves like smooth pipe.

Solution:

v =QA

=540 ×10154

0.3( )2= 7.64 m / s.

RN =vDv

=7.64 × 0.3

0.023 ×10−4 = 9.97 ×105

For smooth pipe

1f

= 2.0 log10 RN f( )− 0.8

=2.0 log10 9.97 ×105 f( )− 0.8

=2.0 log10 f +11.2

Solving by Trial and error

f = 0.0116

For smooth pipe

RN f

Rk≤ 17

Rk

>RN f

17=

9.97 ×105 × 0.011617

= 6317.65

k =R

6317.65=

0.15 ×10006317.65

= 0.0237mm

Loss of head hf =fLv2

2gd=

0.0116 ×1000 × 7.64( )2

2 × 9.81 × 0.3

= 115.0 m

Shear velocity v* = v f8

= 7.64 0.0116

8= 0.29 m / s.

Maximum velocity for smooth pipe:

Page 177: Fluid Mechanics and Hydraulic Machinery

E310/1 177

vv∗

= 5.75log10v∗y2

+ 5.5

at y = 0.15 v = vmax

vmaxv∗

= 5.75log100.29 × 0.15

0.023 ×10−4

+ 5.5

= 5.75×4.278+5.5 = 30.0

vmax = v* ×30 = 8.7 m / s.

Velocity at r = 10 cm ,

y = R - r

=15 - 10 = 5

vv∗

= 5.75log100.29 × 0.05

0.023 ×10−4

+5.5

= 21.84 + 5.5 = 27.348

v = 7.9 m / s.

WE(8):

In problem (7) what should be roughness height . If the pipe behaves like rough

pipe. Then determine the maximum velocity and loss of head.

Solution:

RN fR

k

≥ 400

f ≥400 R k( )

9.97 ×105 ≥ 40.12 ×10−5 Rk

For rough pipes

1f

= 2.0 log10R

k( )+1.74

Page 178: Fluid Mechanics and Hydraulic Machinery

E310/1 178

140.12 ×10−5 R

k( )= 2.0log10R

k( )+1.74

Solving by trial, R / k = 363.3

k =0.15363.3

= 4.13 ×10−4 = 0.413mm

∴ f = 40.12 ×10−5 0.154.13× 10−4

= 0.1457

f = 0.0212

hL =fLv2

2gd=

0.0212 ×1000 × 7.642

2 × 0.3 × 9.81= 210.4m

Max velocity :

We know velocity distribution in rough pipe is

vv∗

=v

v f8

=v

9.64 0.02128

=v

0.393= 5.75log10

yk

+ 8.5

v = vmax at y = 0.15.

vmax

v∗= 5.75log10

0.154.13× 10−4

+ 8.5

= 14.72 + 8.5 = 23.2

∴vmax= 23.2 ×0.393 = 9.12 m / s

Summary:

(1). For laminar flow the velocity distribution is v =1

2µ−∂p∂x

By − y2( )for parallel plates at

rest.

The mean velocity =B2

12µ−∂p∂x

Max velocity = B2

8µ−∂p∂x

∴ v =23

vmax

Loss of head hf =12µvLωB2

Page 179: Fluid Mechanics and Hydraulic Machinery

E310/1 179 Shear stress at boundary

τ0 =12

−∂p∂x

B

(2) The velocity distribution in couette flow is

v =vyB

−1

2µ∂p∂x

(By − y2 )

Which is a super position of simple couette flow on laminar flow through stationary plates.

The non dimensional form of velocity distribution for general couette flow is

vV

=yB

+ P 1 −yB

yB

where P =B2

2µ2 −∂p∂x

where V is the velocity of top plate.

(3). For laminar flow through pipes the velocity distribution is

v =1

4µ−∂p∂x

R2 − r2( ) which is a parabolic distribution.

vmax =1

4µ−∂p∂x

R2

v =vmax 1 −rR

2

vav = 1

32µ−∂p∂x

D2

So vav = 1 / 2 vmax

Loss of head hL =32µvLωD2

and f = 64 / RN

Shear stress τ = −∂p∂x

r2

(4) Laminar sub layer thickness S’’ =11.6v

v∗

where v* = shear velocity = vf8

(5) Criterion for smooth and rough pipe is

(1) k / S| ≤0.25 smooth pipe

Page 180: Fluid Mechanics and Hydraulic Machinery

E310/1 180 k / S| ≥ 6.0 rough pipe

(2) v∗kv

≤ 3 smooth pipe

≥ 70 rough pipe

(6)

(a) Velocity distribution for smooth pipe v / v* =5.75 log10 (v∗yv

)+5.5

rough pipe = v / v* = 5.75 log10(y / k)+8.5 known as Kurman -

Prandtl equations.

(b) Mean velocity for

Smooth pipe v

v∗= 5.7log10

v∗Rv

+1.75

Rough pipe v

v∗= 5.75log10

Rk

+ 4.75

v − vmaxv∗

= 5.75log10yR

for both Smooth and rough pipes

and v − vmax

v∗= 5.75log10

yR

+ 3.75 for both smooth and rough pipe.

(7) Friction factor ‘f’

(a) smooth pipe

1f

= 2 log10(RN f ) − 0.8

(b) For rough pipe

1f

= 2 log10Rk

+1.74

(8)RN f

Rk

≤ 17 smooth pipe

≥ 400 rough pipe

Answers for S.A.Q.

1) - 4) True 5) 2.0 K N / m2 6) - 9) True

10) y = 0.051 11)500 n / m 2 12) 0.5 m / s

13) 0.4 m / s 14)0.42 m / s 15)r = 7.07 cm

Page 181: Fluid Mechanics and Hydraulic Machinery

E310/1 181 16) f = 0.05 17) 4 N m / s 18) τ0 =250 N/m2

19)- 30) True 31) Rough.

Exercise

8.1) An oil of specific gravity 0.92 and dynamic viscosity of 0.082 pose flows in an 80 mm

diameter pipe. In a distance of 20m the flow has a heat loss of 2m. Calculate (1) The mean

velocity , discharge velocity and shear stress at a radial distance of 38 mm from the pipe

axis and boundary shear stress.

[v =2.197 m / s , Q = 11.04 l/ s

v = 0.4284 m / s , τ = 17.114 Pa τ0 = 18.02 Pa]

8.2) What power will be required for kilometre length of a pipeline to overcome viscous

resistance to the flow of an oil of viscosity 2.0 poises through a horizontal 10 cm diameter

pipe at the rate of 200 l/ min? Find the Reynolds number of the flow if the relative density

of the oil is 0.92.

[ RN = 194.8 , P = 0.905 KW]

8.3) A flow of 60 L / s per meter width of glycerine of specific gravity 1.25 and dynamic

viscosity 1.5 poises takes place between two parallel plates having a gap of 25 mm

between them. Calculate the (1)maximum velocity (2) Boundary shear stress and (3)

Energy gradient

[ Vmax = 3.6 m / s , τ0 =864 Pa hf / L = 5.648]

8.4) Two parallel plates are placed horizontally 10 mm apart. The bottom plate is fixed and

the top plate is moved at a uniform speed of 0.25 m / s. The fluid between them has a

dynamic viscosity 1.472 N S / m2 . Determine the pressure gradient which corresponds to

the condition of zero discharge between the plates and the shearing stress at each plate.

[22.08 K N / m2 / m , -73.6 N / m2 , 147.2 N / m2]

8.5) A smooth pipe line 0.1 m in diameter and 1000m long carries water at the rate of 7.5 lps

of the kinematic viscosity of water is 0.02 stokes. Calculate the head loss , wall shear

stress , centreline velocity ,shear stress and velocity at 40 mm from the centreline and the

thickness of laminar sublayer.

[9.95m ,2.44 N/m2 1.15 m /s 1.95 N /m2

0.95 m /s ,0.47 mm]

8.6). A pipe of diameter 0.3m is to convey water at 40 oC at the rate of 200 lps of the power

required to maintain the flow in 100m length is 61.8 KW, Calculate the value of k, vmax .τ 0

and v*. Take v = 0.0075 stokes (at 40 oC for water)

Page 182: Fluid Mechanics and Hydraulic Machinery

E310/1 182 [0.57 mm ,3.4m / s ,23.18 N / m2 , 0.152 m /s]

8.7). A 300 mm diameter pipeline carries water at 20 oC with a mean velocity of 7.5 m / s .

The pipeline is new with no surface irregularities at the beginning , but it was found that

the surface irregularities grow at the rate of 0.075 mm per year. Find the number of years

after which the surface irregularities will affect the flow. Take v = 0.01 stokes.

[2.67 years]

8.8). Field tests on a 30 cm cast pipe carrying 0.25 m3/s of water [v = 1×10-6 m2/s] indicate

that the height of roughness projections has incurred to 1.5 mm after many years of

service. What increase in flow can be expected if the pipe is replaced by a new pipe with k

= 0.26 mm of the same diameter .

[40 lps].

*****

Unit 9

Open Channel flow Aims: The aims of this unit are to define open channel flow, types of flow, to explain velocity distribution over the cross section of flow, energy and momentum correction factors, to review velocity equations, in uniform flow and to analyse economical sections. Objectives :

1. To define open channel flow and compare it with pipe flow. 2. To explain types of flow in open channel. 3. To discuss about velocity distribution over cross section of open channel flow. 4. To calculate energy and momentum of fluid flowing in open channels and then to obtain their correction factors. 5. To review uniform flow and to obtain expressions for velocity of flow by chezy, manning and Bazio. 6. To define economical section and to obtain conditions for economical sections of rectangular, trapezoidal, triangular and circular open channels. 7. To apply the above to solve practical problems. 9.1 Introduction:

Like pipe flow, open channel flow is another important branch of Hydraulics, which is very useful to civil engineers. The flow of rain water in streams, rivers comes under open channel flow. In Irrigation the application of theory of open channel flow is very much necessary. So study of open channel flow is very important to civil Engineer without which he can not be a successful civil Engineer.

Page 183: Fluid Mechanics and Hydraulic Machinery

E310/1 183 9.2. Definition of Open Channel flow Open channel flow is defined as that flow which takes place in a passage with a free surface subjected to atmospheric pressure. Here the water surface is exposed to atmosphere.

Figure

When this flow is compared to the flow through a pipe it is seen as shown in figure. Here the bottom of open channel refers to centre line of the pipe, the free surface is similar to hydraulic gradient, and the total energy is same in both the cases over the datum. In open channels the bed will be slopped down towards the direction of flow, to make the gravity force (component of weight of water) to overcome resistance and to cause the flow of water. Channels are broadly classified into natural and artificial channels. Rivers streams etc are examples of natural channels. Artificial channels are made artificially to carry water with different cross sections like rectangular, trapezoidal, triangular, parabolic and circular sections. Closed conduits flowing partially, with free surface exposed to atmospheric pressure are also called as open channels. Under ground drains are the examples of closed conduit open channels. A channel which has the same shape of cross sections along its length is called as prismatic channel, otherwise it is called as non prismatic channel.

Identify the following as True or False SAQ1 When water surface in a passage exposed to atmospheric pressure it is called as open channel flow. SAQ2 In open channels the flow is gravity flow SAQ3 If the closed conduit running helpful it is called as pipe flow 9.3 Classification of Open Channel flow i. Steady and Unsteady flow If the flow characteristics like velocity depth do not vary with time at any given cross section of the channel, then it is called as steady flow

Page 184: Fluid Mechanics and Hydraulic Machinery

E310/1 184 ie

dvdt

= 0,dydt

= 0

In prismatic channels since cross section is constant along the length, then the flow is

steady if dydt

= 0

If velocity and depth vary with time at any cross section of channel then it is called as unsteady flow is

ie dvdt

≠ 0 and dydt

≠ 0

Flow in a river during rainy season is unsteady. ii. Uniform and non uniform flow (varied flow) When depth slope, cross section and velocity do not change along a given length, then the flow is called as uniform flow.

ie dyds

= 0, dϑds

= 0, dAds

= 0, dSds

= 0

So uniform flow occurs only in prismatic channels. If the above flow characteristics vary along the length of channel then it is called as non uniform (varied) flow.

ie dy/ds ≠ 0 etc Further varied flow is classified as gradually varied flow and rapidly varied flow. If the depth of flow varies abruptly over a short length of channel, then it is called as rapidly varied flow (R.V.F). Hydraulic jump is the example of R.V.F. iii. Laminar flow and Turbulent flow Flow in open channel can also be classified as laminar, transition and turbulent flows, like in pipe flow, based on Reynolds number which is given by

RN =ρVR

µWhere R = Hydraulic Mean radius

V = Velocity of flow By experiments it is shown that when RN = 500-600 , the flow is laminar in open channels and RN > 2000, the flow is turbulent flow and between 500-2000 the flow in transitional flow. iv. Subcritical, critical and super critical flow

Based on relative magnitudes of gravity force and inertia forces, the flow in channel can be classified as subcritical, critical and super critical flow. The relative magnitude of gravity force and inertia force is given by Froude Number which is square root of ratio of Inertia force to gravity force ie

Page 185: Fluid Mechanics and Hydraulic Machinery

E310/1 185

ie FN =Inertiaforcegravityforce

(ρL3 VT ) (ρL3 g)[ ]

12

= (ρL2 LT v

(ρL3g)

12

= ρL2 v2

ρL3g

12

=vLg

where L is taken is depth of flow = y

ie FN =vgy

If v< gy , FN <1, then the flow is called as subcritical or tranquil or streaming flow. If v= gy , FN =1, then the flow is called as critical flow.

and of V> gy FN >1, then the flow is called as supercritical, rapid or shooting or torrential flow. Hydraulic jump occurs when the flow changes from super critical to sub critical flow. Identify the following as True or False. SAQ4 During floods, the flow in a river is unsteady. SAQ5 In a prismatic channel ifdischarge is constant, then the flow is uniform flow SAQ6 When mean velocity is equal to gy then the flow is called as critical flow. Geometrical Properties of Channel section

y = Vertical Depth of flow T = Top width of flow A = Cross sectional area of flow P = Wetted Perimeter which is in contact with water R = Hydraulic mean radius or depth = A/P D = Hydraulic depth = A/T

Z = Section factor = A D = A AT = A3

T( )12 for critical flow

Z` = AR2/3 for uniform flow 9.4 Velocity distribution in open channel

Due to free surface and frictional resistance of the boundary surface, the velocity distribution over the cross section of flow is non-uniform. The velocity distributions as measured by pitot tube on various cross sections are shown in figure which are function of shape of section, roughness of channel and bends in the channel.

Page 186: Fluid Mechanics and Hydraulic Machinery

E310/1 186

The maximum velocity will occur at a depth equal to 0.05 to 0.15 y from free surface. The mean velocity can be computed from the velocity distribution and it is equal to local velocity at a depth o.6y from the free surface. A better approximation for the mean velocity is equal to average of velocities at 0.2 depth and 0.8 of depth from free surface.

ie Vmean = 12

(vel ab 0.2y+vel at 0.8y)

Due to non-uniform velocity distribution over the cross section, the computation of kinetic energy and momentum of flow based on mean velocity are to be corrected by multiplying those by ‘α’ kinetic energy correction factor and ‘β’ momentum correction factor respectively as shown below. α - Kinetic energy correction factor (coriolis coefficient) Kinetic energy based on velocity distribution =

------ do----- based on mean velocity = α12ρAv

v 2 = α

12ρAv3

∴α =ϑ 3

A∫ dA

Av 3 > 1

for turbulent flow α = 1.03 - 1.36β - Momentum Correction factor (Boussining co efficient)

β =ρϑdAϑ

A∫ρVAV

=ϑ 2 dA

A∫

AV 2

β = 1.01 - 1.12 for turbulent flow But generally for turbulent flow , these are taken as unity.

Identify the following as True or False SAQ7 Maximum velocity occur at a depth of 0.05 - 0.15 of depth from free surface. SAQ8 Mean velocity is equal to local velocity at a depth of 0.6 depth from free surface SAQ9 Kinetic and momentum correction factors are taken as unity in turbulent flow.

Page 187: Fluid Mechanics and Hydraulic Machinery

E310/1 187 9.5 Uniform flow in open channel

Review : We know already that the depth of flow wetted area, velocity and discharge are constant along the prismatic channel in case of uniform flow. So the water surface , the bed of channel are parallel to total energy line. Consider a steady and uniforms flow in a prismatic channel of length L is shown in figure

Figure

The forces acting are 1. Gravity force which ;is equal to the weight component in the direction of flow = w sinθ2. Frictional resistance given by τox PL where τo - shear resistance per unit surface area. 3. Hydrostatic forces p1 & p2 which are equal and opposite as depths y1 = y2 in uniform

flow Now as the flow is steady ∑F = m × a = 0 --- acceleration = 0 in steady flow ∴w sinθ - τ .PL + p1-p2 = 0w sinθ = τ.PL This shows that the gravity force is equal and opposite to frictional resistance in uniform flow w=wΑL

∴τ o =WALsinθ

PL= W

Ap

sinθ = ωRsinθ

But sinθ ≈(z1 − z2 )

l= s. bed slope = tan θ

for small angle ‘θ’∴ τ o = w R So

But τ0 = f8 ρv 2 by pipe flow analysis

ie τ 0 = wRSo = f 8ρv2

Page 188: Fluid Mechanics and Hydraulic Machinery

E310/1 188

v = 8wpf

Rs0 . = C Rs0

where C = 8gf

is called as chezy’s coefficient

and V= C Rs o is called chezy’s equation. for mean velocity in uniform flow in open channels. Here C is inversely proportional to ‘f’ Darcy weisback coefficient of friction.

The dimensions of C = L12 T −1 and so C is not constant and varies for each system of unit.

It is simple but its determination is difficult. Further by applying Bernoulli’s equation between (1) and (2) , we have Z1+ y1 + V1

2 /2g = Z2 + +y2 + V22 / 2g + hf

But y1 = y2 V1=V2 in uniform flow ∴ hf = Z2 -Z1

slope of TEL = z2 − z1

L=

hf

L= s f = so = sw

Thus all the slopes are parallel to each other in uniform flow. The depth of flow in uniform flow is called as normal depth ‘yn’

To determine C Empirical formulae have been developed to find C as given below. a. The Ganguillet - Kutter formula

Based on flow measurement, in open channels above two Swiss engineers proposed in 1869 an empirical formula to find C in M.K.S as

C = 23 +

0.00155s

+1n

1 + (23 + 0.00155s

) nR

where n- Kutter’s roughness coefficient = Manning roughness Coefficient n - depends on channel surface and its condition. The typical values of n for different surfaces are given below channel surface value of n 1. Very smooth concrete, planed wood 0.012 2. Ordinary concrete lining 0.013 3. B.W lined with CM

0.015

4. C.C finish. 0.015 5. Unfinished c.c 0.017 6. Neatly excavated rock 0.02

Page 189: Fluid Mechanics and Hydraulic Machinery

E310/1 189 7. Unlined earth channels in good condition 0.02 8. Rubble masonry 0.02 9. Rivers and earth Channels in fair condition 0.025 10. Earth channels with gravel bottom 0.025 11. Earth channels with dense weed 0.035 12. Mountain stream with rock bed 0.045 b. Bazin formula He proposed in 1897 the following formula to find C in M.K.S as

C =157.6

1.81 + mR

where ‘m’ is proposed by Bazin as below

channel surface value of mVery smooth cement , planed wood 0.11 Concrete, brick or unplan ed wood 0.21 Ashlar, rubble masonry or poor B.W 0.83 Earth channels in very good condition 1.54 ” ordinary condition 2.36 ” rough condition 3.17

c. Manning’s formula In 1889 he proposed the following formula to find mean velocity in M.K.S as

v= 1n

R2/ 3 S1/2 which is very simple and given satisfactory results and is widely

used in practice

Here C = 12

R1 6

Identify the following as True or False

SAQ10 Water weight component in the direction of flow is equal to frictional resistance in uniform flow. SAQ11 Chezy’s Coefficient is in verbally proportional to square root of Darcy weisback Coefficient of friction ‘f’ SAQ12 In uniform flow bed slope, water surface slope and energy line slope are all parallel to each other. SAQ13. Chezy’s coefficient C is dependent on surface of channel and its condition SAQ14 Chezy’s coefficient is inversely proportional to Manning’s roughness coefficient

Worked Example (1) A rectangular channel conveys a discharge of 10 m3/s . If the width of channel is

6m, find the depth of flow if C = 54.62 and bed slope = 1

5000Sol:

Page 190: Fluid Mechanics and Hydraulic Machinery

E310/1 190 Q = AC RS

1.0 = b × yn × 54.626yn

6 + 2yn

1

50006yn

3

6 + 2yn

=10

6 × 54.62

2

× 5000

y3n = 4.655 + 1.552yn

yn = 1.95

Worked Example (2) A discharge of 100 l.p.s is flowing in a rectangular channel of 60 cm wide with a normal depth of 30 cm. Find the necessary slope if C = 56

A = 0.6×0.3 = 0.18 m2

velocity = Q A =0.1

0.18= 0.555 m s

p = b+2d = 0.6+2×0.3 = 1.2 m

Hydraulic mean depth R = A p =0.181.2

= 0.15m

But ϑ = c Rs0.555 = 56 × 0.15 × 3

s =1

1500

9.6 Economical Sections Economical section is that when maximum discharge can pass through it for a given cross section, roughness coefficient and bed slope. By this definition it is clear that discharge is maximum for a given cross sectional area when velocity is maximum since by continuity equation Q = AV. Velocity is maximum

when R is maximum because V = C RS or 1n

R2 3S 1 2 ie R = A/P , R is maximum

whenP is minimum So for given slope and roughness value V is maximum when P is minimum. Therefore section is economical when P is minimum for a given cross section, roughness coefficient and bed slope . By making use of this condition expressions can be obtained for different channels whose sections are to be economical. a. Rectangular channel.

Consider a rectangular channel of bottom width ‘B’ and depth of flow as y

Then P = B+ 2y, A = By

So p = Ay

+ 2y which is function of y only

Page 191: Fluid Mechanics and Hydraulic Machinery

E310/1 191 If p is to be miximum

dpdy

= 0

ie dpdy

= −Ay2 + 2 = 0

A = 2y2

By = 2y2

B = 2y or y = B 2is the condition for rectangular channel to be economical Further Hydraulic mean radius R = A/P

R =By

B + 2y=

2y × y4y

=y2

ie R =y2

is another condition for rectangular channel to be economical

b. Trapezoidal channel

Consider a Trapezoidal channel with bottom width B and side slope Z : 1 as shown in figure with depth of flow as y Here the cross sectional area is constant The parameters involved are B,y, Z . So there will be three cases as explained below to get conditions for economical sections case (1) for given cross section are A, Z is constant and y is variable.

figure

A = y(B+YZ) and P = B+ 2Y 1 + Z 2

So B = Ay

− yZ

and then p2 =Ay

− yZ + 2y 1 + Z2

which is function of ‘y’ only

for P to be minimum dρ dy = 0

ie dpdy

= −Ay2 − Z + 2 1+ Z

2= 0

Ay2 + Z = 2 1 + Z

2

But substituting for A = y(B+yZ), we have

y(B + yZ)

y 2 + Z = 2 1 + Z2

B+2yZ = 2y 1 + z 2

ie 12

(top width ) = side slope is the condition for Trapezoidal section to be economical

Page 192: Fluid Mechanics and Hydraulic Machinery

E310/1 192 Further R = A/p =

y(B + yZ)B + 2y 1+ Z

2

But 2y 1 + Z2

= B + 2yZ So substituting this in above equation , we have

R =y(B + yZ)2(B + yZ)

=y2

ie Hydraulic mean radius = 12

depth of flow is another condition for trapezoidal section to

be economical. Further draw a perpendicular line OA from centre of Top width t side of the section as shown in figure.

figure.

OC = 12

top width = (B + 2yZ)

2sin θ =

yy 1+ Z

2 =1

1 + Z2

OA = OC sin θ =(B + 2yZ)

11 + Z

2

But (B + 2yz)

2= y 1 + Z 2

ie OA = y 1+ Z1 + Z

2

2

= y

This means that Trapezoidal channel will be economical when a semi circle of radius ‘y’ with centre as mid point of top width will be tangential to both sides and bottom. So trapezoidal section will be economical for a given cross sectional area keeping B and Z as constant, when

1. 12

top width = side slope

2. Hydraulic mean radius R = y/2 3. Semicircle of radius y and mid point of top width as centre will be tangential to both sides and bottom. case 2. For given cross section B is constant here also we have

A = (B +zy)y p= B+ 2y 1 + Z 2

From (1) Z = A/y2 - B/y

Page 193: Fluid Mechanics and Hydraulic Machinery

E310/1 193 substituting this for p, we have

p = B+2y 1 +Ay2 − B y

2

dpdy

= 2 1 +Ay 2 −

By

2

+ 2y ×12

1

1 +Ay2 −

By

2 × 2Ay 2 −

By

−2Ay 3 +

By2

= 0

2 1 + (Ay2 −

By

)2

+ 2y

Ay2 −

By

2Ay3 +

By2

= 0

1 = ZAy 2

Z =y2

A=

y 2

(B + Zy)Y;

(B+Zy) = yZ

By

+ Z =1Z

,

By =

1Z

− Z =1 − Z 2

Z

ie By

=1 − Z 2

Z

are the conditions required for Trapezoidal section to be

&` A =y 2

Zeconomical when B is constant

case 3 Depth of flow y = constant here also we have A = (B+Zy)y A/Y-Zy=B p = B+2y 1 + Z 2

p =Ay

− Zy + 2y 1 + z 2

For p to be min dp/dz = 0

ie dpdz

= −y +2y

2 1 + z 22z = 0

2z = 1 + z2

1+z2 = 4z2

z= 13

or tanθ = 3

or θ = 600

So when depth is constant for a given cross sectional area, Trapezoidal section is economical when the inclination of slides with horizontal is 600.

c. Triangular channel

Page 194: Fluid Mechanics and Hydraulic Machinery

E310/1 194 Consider a triangular section with depth of flow as y and side slope Z=1. Let apex angle be 2θ.

figure

Then A =

12

2y tan θ y = y2 tan θ

p = 2y sec θ

but y = A

tanθ(fromA)

s0 p = 2A

tansecθ

for p to be minimum dp/dθ = 0

ie dpdθ

= 2 Asecθ tanθ

tanθ−

secθ2 tan 32 θ

sec2 θ

= 0

2 tan2 θ - sec 2 θ =0

sin θ =12

or θ = 450 ie Z = 1 So triangular section is economical when included angle is 900 i.e. side slope is 1:1 , i.e. half square on diagonal.

figure

Further R =Ap

=y2 tanθ2y secθ

=y sinθ

2

But sin θ=12

∴R =y

2 2

NOTE: Q = ACAp

s

Page 195: Fluid Mechanics and Hydraulic Machinery

E310/1 195 Q is maximum when A is ;maximum for a given p. Then the conditions will be same for triangular section to be economical as derived in the previous section , for a given p. d. Circular channel

In this case both cross sectional area and wetted Peri meter vary with depth of flow y. Hence in circular channels two conditions will be derived (1) for maximum dischargeand (2) for maximum mean velocity as arrived below. i. Condition for maximum discharge

Let in a circular channel of radius r, the depth of flow be ;y as shown in figure and ‘θ’ bethe angle subtended at centre by wetted perimeter. Then we have

A =Πr 2

2θΠ

− 2 ×12

2sinθ

22 cosθ

2

=r 2θ

2−

12

r 2 sinθ

=r 2

2(θ − sinθ )

p = r θ

Then Q = AC RS = CA Ap

s = c A3

ps

Then for given C & S, Q is maximum when A3/p is maximum

ie d A3 / p( )

dθ= 0

=p3A2 dA dθ − A3 dp dθ

p 2 = 0

but dA dθ =r 2

2(1− cosθ )

and dp dθ = rby substituting dA dθ , dp dθ in above equation we have

3p r 2

2(1− cosθ ) − Ar = 0

2θ-3θ cosθ + sinθ= 0ie = 3080

for maximum discharge depth of flow is y = r+r cos(180-θ/2) = r(1+cos 260)

= 1.8988 r ≈ 0.95D so the depth of flow is 0.95D for maximum Q and hydraulic mean radius is

R = A p =r 2

2(θ − sinθ) ×

12θ

Page 196: Fluid Mechanics and Hydraulic Machinery

E310/1 196 =

r(θ − sinθ )2θ

=r2

×1

308

=r2

×1

308180

× Π(Π

180308 − sin 308)

= 0.5733r 0.29D So for maximum discharge R = 0.29D The above conditions slightly vary if Manning’s equation is used for velocity i.e y = 0.938D Condition for maximum velocity

We have velocity = V = C RS= C A p s

So V is maximum when A/p is max

i.e d(A / p)

dθ=

pdA dθ − A dp dθp2 = 0

and substituting for dA/dθ as dp/dθ in above equation we have

rθ r 2

2(1 − cosθ ) −

r 2

2(θ − sinθ )r = 0

θ = tan θie θ = 257.50

So the depth of flow for maximum velocity is y = r+ r cos (180-θ/2) r[ 1+cos 51025’]

=1.626r = 0.81D So the depth of flow y = 0.81D for maximum velocity Further R = A/p

= r

2θ(θ − cosθ )

=r

2Π 257.5180

Π180

257.5 − sin 257.5

≈ 0.6086r = 0.3D is maximum velocity the hydraulic mean radius in 0.3D State whether the following are true or false.

SAQ15. When discharge passing through a given cross section is maximum, then it is called as economical. SAQ16. For a given cross section Q is maximum when p is maximum SAQ17 Width of a rectangular channel is 10 cm Find the depth of flow when it is to be economical. SAQ18. When for rectangular section hydraulic mean radius is equal to B/4, then it is economical.

Page 197: Fluid Mechanics and Hydraulic Machinery

E310/1 197 SAQ19. Half of Top width of Trapezoidal section is equal to side slope, then it is called as economical section.

SAQ20. When hydraulic mean depth is equal to 12

nthe depth of flow , then both

Trapezoidal and rectangular sections are economical SAQ21. The sides and bottom of a Trapezoidal section are tangential to a semi circle of radius equal to depth of flow and its centre being mid point of top width, then the section is economical . SAQ22. When depth of is kept constant, the trapezoidal section is economical if side slope is 600 with horizontal. SAQ23 When the slope of side with vertical is 45 0 then the triangular section is economical. SAQ24 In case of circular section analysis of economical section with cross section being constant is not possible. SAQ25 When depth of flow = 0.95 diameter, then circular channel is economical SAQ26. For maximum discharge to pass through circular channel the hydraulic mean depth is 0.29 diameter SAQ27 For circular channel Q is maximum when (A3/p) is maximum SAQ28 For circular channel velocity is maximum when (A/p) is maximum SAQ29 The mean velocity through a circular section is maximum when depth of flow = 0.81 diameter and hydraulic mean radius = 0.3 dia SAQ30 For a given cross section the economical section circular channel is semi circle. Worked Example (3) A lined rectangular channel with Manning’s n = 0.02, is 5m wide and the depth of flow is 2m with a bed slope of 1 in 1500 keeping the same rectangular shape of section wetted perimeter and slope, find the maximum extent increase in discharge. sol : A1 = 5×2 = 10 m2

p1 = 5+2×2 = 9 m

R1 = A1 /p1 =109

−1.11m

Q1 =12

A1 R1

23 so

12

=1

0.02×10 × (1.11)2 3 (

11500

)1 2

=13.8558 m3 /s For maximum Q, y = B/2 P = 2y+B = 2B 9 = 2B , B = 4.5 m, y = 2.25cm A = 4.5 ×2.25 = 10.125 m2

R =10.125

9= 1.125m

Q =1

0.210.125 × (1.125)2 3 1

1500

12

= 14.148m3 /s

∴ change in Q = 14.148 -13.8558 = 0.29242 m3 /s

Page 198: Fluid Mechanics and Hydraulic Machinery

E310/1 198 WE (4) Water is to flow in a channel at 12.5 m3 / s with a mean velocity of 1.25 m/s .Calculate the economical cross section of a). rectangular b). Triangular c). Trapezoidal and d). circular section. Which of these have least perimeter and maximum perimeter a). Economical rectangular channel

y = B/2, A = B× y = 2y × y = 2y2 ×

A =12.51.25

= 10 = 2y2 , ∴ y =102

= 5 = 2.236

So B = 2×y = 2 × 2.236 = 4.47 2m p = B+2y = 8.944 m b). Triangular section to be economical θ = 450

and A = 12

× 2y × y = y 2

10 = y2 y = 3.162m p = (2 × 2)y = 8.943m

c). Trapezoidal section to be economical side slopeθ =600 with horizontal

Z = 13

A= (B + 2zy) + B

2

y = (B + zy)y

(B + 2zy)2

= y 1 + z 2 = y 1+13

=2y

3

B + 2 ×13

y

2

=2y

3

B =23

y

So A = 23

y +13

y = 3y 2

10 = 3y2

y =10

3

12

= 2.4m

B =23

× 2.4 = 2.775m

p = 2y 1 + z2

+ B = 2y 1 +13

+23

y = 2 3y = 8.3m

d. Circular section is economical for a given cross section when Q = c A3 p s is maximum when p is minimum i.e

Page 199: Fluid Mechanics and Hydraulic Machinery

E310/1 199

P = dθ A= r 2

2(θ − sinθ ) =

d 2

8(θ − sinθ)

d =8A

θ − sinθ

∴ p =d2θ

=8A

(θ − sinθ )×θ

For P to be minimum, dp dθ = 0

θ − sinθ −θ

2 θ − sinθ(1− cosθ ) = 0

2(θ − sinθ ) − θ(1 − cosθ) = 0θ = Π

ie economical section is semi circle.

d=8A

θ − sinθ=

8 × 103.142 − 0

= 5.045m

p=rθ =5.045

2× 3.142 = 7.926m

∴The least perimeter is of circular one and highest perimeter is rectangular & triangular Worked Example (5) A power channel of Trapezoidal section has to be excavated through hard clay at the least cost. Determine the dimensions of the channel given Q = 14 m3 /s. Bed slope 1 in1500 and Manning’s n = 0.02

Side slope for best section is 600 with horizontal. So Z = 13

Trapezoidal channel to be economical we have

b + 2zd

2= y z 2 +1

b + 2 ×13

y

2= y y 3

ie b = 2∫3

y

and A = y(b+zy) = 3y2

Q = Aϑ = 14 = 3y 2 ×1

0.02y2

2 3 12500

1 2

∴y = 2.604m

b = 2∫3

y = 3.007m

Worked Example 6) A lined channel ( n = 0.014 ) is of trapezoidal section with one side vertical and the other on a side slope of 1.5 H to 1 v . If the channel is to deliver 9 m3 /s on a slope of

Page 200: Fluid Mechanics and Hydraulic Machinery

E310/1 200 0.0002 , find the efficient cross section which requires minimum lining. Find the corresponding mean velocity. Sol

figure

A = B +zy2

y

B = A y −zy2

and p = B + y + y Z 2 +1

=Ay

−zy2

+ y + y 1+ z2

=Ay

+ 1 −Z2

+ 1 + z 2

y

For p to be minimum, dpdy = 0

ie dp dy =−Ay2 + 1−

z2

+ 1+ z2

= 0

A = y2 (1− 1 −1.52

+ 1 +1.52

= 2.0528 y2

B =Ay

−zy2

=2.0528y2

y−

1.5 × y2

= 1.3028y

p = 1.3028 y +y+ 3.25y = 4.1056y

R = Ap =

2.05284.1056

y = 0.5y

Q =1n

AR2 3s 1 2

9 =1

0.0142.0528y 2 .(0.5y)2 3 (0.0002)

12

y = 2.062 m B = 2.687 m So A = 2.0528 y2 = 8.7282 m2

∴ Velocity v = QA

=9

8.7282= 1.031m / s

SUMMARY 1. Kinetic energy correction factor

α =ϑ 3dA

A∫

Av3 > 1, for turbulent flow

α = 1.03 - 1.36 and momentum correction factor

Page 201: Fluid Mechanics and Hydraulic Machinery

E310/1 201

β =ϑ 2

A∫ dA

Av 2 > 1

β for turbulent flow = 1.01 - 1.12 2. Chezy’s equation is v = C RS

C - Chezy’s coefficient R -Hydraulic mean radius S - bed slope for uniform flow bed slope = water surface slope = Energy slope

Manning’s formula v = 1n

R2 3S 1 2

n - Manning’s roughness coefficient and c = 1n

R1 6

4. Economical section is that when maximum discharge can pass through it for a given cross sectional area ie Q = Ac A pS is maximum when p is minimum for given A 5. For rectangular cross section it is economical when y = B/2, R = y/2

6. For Trapezoidal section it is economical when, for a given cross section A, with side slope constant

12

(Top width) = side slope

R = y/2 or when semicircle of radius y and mid point of top width as centre, will be tangential to sides and bottom. When bottom width constant , side slope variable, for given cross section, it is economical when

B y =1− z 2

zand A =

y2

zwhen depth y = constant,

it is economical when side slope θ = 600 with horizontal.

7. Triangular section is economical when the included angle is 900 , ie hole slope is 450 with vertical. 8. Circular section is economical when y = 0.95D for maximum Q R = 0.29D

y = 0.81D for maximum velocity R = 0.3D

and for given cross section A it is economical when it is semicircle Answers to SAQ

(1) - (16) -- True ; 17 - 5m

Page 202: Fluid Mechanics and Hydraulic Machinery

E310/1 202 (18) - (23) -- True ; 24 - False, (25) - (30) -- True Exercise

9.1 A trapezoidal channel base width 8 m and bed slope 1 in 400, carries water at 12 m3/s and side slopes are 1:1 , compute the normal depth. Take n = 0.025 [0.85 m ] 9.2 A trapezoidal channel has a bed width of 2.0 m side slope of 1.25 horizontal : 1 vertical and carries discharge of 9 m3 / s at a depth of ;2.0m . Calculate the average velocity and bed slope of the channel . Take n = 0.015 [ v = 1.0 m/s , s0 = 2.0554×10-4]9.3 A rectangular channel 3.0 m wide had a badly damaged living whose Manning’s ‘n’ was estimated as 0.025. The lining was repairedand it has now an n = 0.014. If the depth of flow remains the same as 1.3 m as before the repair, estimate the new discharge [Q = 4.894 m3 / s] 9.4 What diameter of a semi circular channel will have the same discharge as a rectangular channel of width 2.0m and depth 1.2 m ? Assume the bed slope & Manning’s n are the same for both the sections [ D = 2.396 m] 9.5 A triangular channel has a vertex angle of 75 0 and a longitudinal slope of 0.001 If mannigs n = 0.015 estimate the normal depth for a discharge of 250 lps in this channel [yn = 0.668m] 9.6 A rectangular channel [ n = 0.02] is 5.0 m wide and 0.9 m deep and has slope of 1 in 1600. If the channel had been designed to be of efficient rectangular section for the same wetted perimeter, what additional discharge could it carry [ ∆Q = 2.211

m3 / s] 9.7 A trapezoidal channel has one side vertical and the other side has a slope of 1.5 H ; 1V . This carries a discharge of 15 m3 /s with velocity of 1.5 m/s . Calculate the dimensions of an efficient section of this shape and also the bottom slope necessary to achieve this discharge [ n = 0.0130] [ y = 2.207n, B = 2.876m, s0 = 3.335 × 10-4 ] 9.8 Deter mine the dimensions of a concrete lined ( n= 0.014) trapezoidal channel of most efficient proportions to carry a discharge of 10.0m 3/s. The bed slope of the channel is 0.005 [ y = 1.25 m, B = 1.444m, side slope Z = 0.5773] 9.9 Determine the efficient section and bed slope of a trapezoidal channel ( n = 0.025) designed to carry 15 m3 /s of flow. To prevent scouring the velocity is to be 1.0 m/s and the side slope of channel are 1V : 2H [y = 2.463m, B = 1.163m, s0 = 4.735 × 10-4]

* * *

FLUID MECHANICS & HYDRAULIC MACHINERY

UNIT X

OPEN CHANNEL FLOW

Page 203: Fluid Mechanics and Hydraulic Machinery

E310/1 203 Aims:

The aims of this unit are to define specific energy, specific force and to derive conditions

for critical depth to compute critical depth and to apply this concept to channel

transistors.

Objectives:

1. To define specific energy and explain meaning of critical depth by means of specific energy curve.

2. To derive conditions for critical depth for (a). given discharge and (b) given

specific energy

3. To define momentum in open channel flow and obtain an expression for specific

force

4. To derive a condition for critical state of flow for (a) given discharge and (b)

given specific force.

5. To compute critical depth

6. To analyse critical flow in rectangular channels.

7. To apply the above to solve practical problems.

10.1 Introduction:

In the design of transitions in open channels, specific energy and critical depth concept is

very much necessary. Similarly the energy description below spillways, hydraulic jump is

utilised. So the conditions required for the foundation of hydraulic jump can be obtained

from specific force concept. In addition to it there are many practical applications of

hydraulic jump. So this unit is very useful for civil engineers in solving open channel

problems.

10.2 Specific energy critical depth

The concept of specific energy was first introduced by Bakhmeteff in 1912 It is defined

as total energy of flow per unit weight of water which is measured with respect to the

channel bed as datum. Thus datum head in Bernoulli’s total head is zero.

ie specific energy E = y +ϑ 2

2g, But V =

QA

so E= y +Q2

2gA2

Page 204: Fluid Mechanics and Hydraulic Machinery

E310/1 204 Thus for a given Q in a prismatic channel, specific energy is function of depth of flow

only. So the relationship between specific energy for a given Q, and cross sectional area

A, and depth of flow is as shown in the figure.

Since E = y + v 2

2g , the curve of specific energy is asymptotic to 450 line through

origin and x axis as shown in figure. In the figure it is seen that specific energy is

minimum at a depth of yc . This depth is called as critical depth. So critical depth ‘yc’ in

open channel is defined as that depth at which the specific energy is minimum for given Q

and A. Velocity at this depth is called as critical velocity.

When the depth of flow in the channel is increased velocity decreases for the same

discharge in same channel or vice versa. When depth of flow is decreased the velocity

increased. When depth of flow is more than the critical depth yc , the flow is called as

subcritical flow or tranquil flow and when the depth is less than critical depth the flow is

called as supercritical flow or rapid flow. Thus for any given specific energy E , there are

two possible depths y1, and y2 which are called as subcritical depth and y2 is called as

super critical depth. These depths are called as alternate depths.

For given Q the minimum specific energy can be obtained by differentiating E

with respect to y as follows

E = y +Q 2

2gA2

dEdy

= 1 +Q 2

2g(−2)

1A3

dAdy

= 0

But as shown in figure dA = Tdy

So dA/dy =T

substituting those in the above equation, we have

1=Q 2TgA3 or

Q 2

g=

A3

Tis the condition for critical flow.

Page 205: Fluid Mechanics and Hydraulic Machinery

E310/1 205 But v =

QA

hydraulic depth D = AT

Then we have Q2

A2

×

1g

=AT

v 2

g= D

v2

gD= 1 or

vgD

= 1

A at critical depth, Fr , Froude number = 1

So for sub critical flow Fr <1 Q D is more and v is less

for super critical flow Fr >1 D is less and v is more

Similarly for a given specific energy, we have

Q2

2gA2 = E − y (A)

Q = A 2g(E − y)

Thus for given E and A, Q is function of y only . So the relationship between Q and y is

obtained as shown in figure.

ie Q is maximum at a depth called as critical depth, which can be obtained by

differentiating Q w.r.to y as follows

dθdg

= 2g(E − y)dAdy

+12

A1

2g(E − y)2g(−1) = 0

2g(E-y)T-Ag = 0

2(E-y) = A/T

Page 206: Fluid Mechanics and Hydraulic Machinery

E310/1 206

But Q2

gA 2 = 2(E − y) from equation (A)

ieQ2

gA 2 =AT

or Q 2

g=

A3

Tis the condition for critical flow at which Q is maximum, which is same

as the previous equation, for critical depth.

The above curve can be obtained when water is controlled by a gate into a prismatic

channel. When the gate is closed discharge into channel is Q = 0 and depth of flow v/s of

the gate is maximum. As the gate is gradually opened, the depth of flow v/s of the gate

decreases and discharge is gradually increased, till it reaches a maximum value at which

the depth of flow v/s of gate is critical. Further opening of the gate will have no effect

over the flow and thus top portion of the curve is obtained ;and the bottom portion is

imaginary.

Similarly d/s of the gate Q = 0, y = 0 and depth of flow gradually increases till it

reaches a value at which the Q is maximum then this depth is called critical depth. After

words opening of gate will have no effect over the flow. Thus lower portion of the curve

is obtained and upper portion of the curve is imaginary.

From the curve it is seen for any discharge there are two possible depths, y1 and y2 . These

depths y1 is called as subcritical and y2 is called as super critical depth. These two depths

are called as alternate depths.

10.3 Critical flow in rectangular channel

consider a constant flow in a rectangular channel of B wide.

In this case specific energy E = y +v2

2g

Let q be the discharge per metre width

Then ϑ =qy

Page 207: Fluid Mechanics and Hydraulic Machinery

E310/1 207

So E = y +q2

2gy2

and for min E, dEdy = 0

ie dEdy

= 1 +q 2

2g(−2)y3 = 0

q2

gy 3 = 1

q2

g= yc

3

or yc = q g23 which is called as critical depth Thus at minimum specific energy the

flow is critical.

ie q2

(y2 )gy= 1

ϑ 2

gy= 1

ϑgy

= 1

ie Froude no Fr =1

suppose for E constant we have

q2

2gy2 = (E − y)

or q = y 2g(E − y)

for maximum discharge dq/dy = 0

ie dqdy

= 2g E − y +y

2 E − y(−1)

= 0

2(E-y) - y = 0

But from eqn (A) q2

gy 2 = 2(E − y)

∴q 2

gy2 = y

q2

g= yc

3 which is same as before

Page 208: Fluid Mechanics and Hydraulic Machinery

E310/1 208 So in rectangular channel , for constant discharge specific energy is minimum or for

constant specific energy discharge is maximum. For these two conditions the flow is

critical which is given by

q2

g= yc

3

Further 2(E-y) -y =0

(E-y) = y/2

E = y + y 2 =32

yc

ie critical depth yc =23

L5

At this depth velocity is critical velocity and bed slope is critical bed slope.

The above condition can be obtained by

Q 2

g=

A3

T

for rectangular channel T = B

∴Q 2

A2( )g =AB

= y

Q2

B2 y2( )g = y

q2

g= y3c which is derived as above

10.4 Momentum of flow in open channel-specific force

Consider a small reach of a channel as shown in figure

The forces that are acting on the fluid are

1. hydrostatic forces p1 and p2 where

P1 = wAz1 and P2 =wAz2

Page 209: Fluid Mechanics and Hydraulic Machinery

E310/1 209 z1 & z 2 = Centre of Gravity of area of cross section A1 & A2 from force surface

respectively

2. Water weight component W sin θ in the direction of flow where W = water weight in

the reach.

3. Frictional resistance due to surface in contact with water.

By Newton’s 2ns law, we have

∑f = rate of change of momentum per second

∴P1 -P2 +W sin θ -Ff =wQg

(v2 − v1 )

Here as the reach is small, we can neglect Ff =0 also as the slope is small, we can also

neglect the weight component in the direction of flow.

So p1 - p2 =wqg

(v2 − v1 )

wA1z1 − wAz2 =wqg

(v2 − v1 )

or qv1

g+ Az1 =

qv2

g+ Az2 = F is constant

ie qvg

+ Az is called as specific force which is constant . This specific force of the

water at any cross section is the force per unit specific weight of water.

In a prismatic channel for given discharge this specific force is function of depth ‘y’ . So

the relationship between specific force F and depth y is as shown in figure.

ie when the specific force is minimum the depth is called as critical depth. So for specific

force to be minimum we have dFdy

= 0

ie dFdy

= (−1)q2

gA2dAdy

+d Az( )

dy= 0

Page 210: Fluid Mechanics and Hydraulic Machinery

E310/1 210 where

dAdy

= T

and d(A z) u is the change in moment of Area A about force surface for a change in

depth dy which can be calculated as given below.

d(Az ) = A z + dy( ) + Tdy dy2

− Az = Ady +

T(dy)2

2

∴d(Az ) = Ady by neglecting small terms.

substituting these in above equation we have

q2

gA 2 T =Adydy

= A

q2

g=

A3

Tis the condition for critical flow at which F is minimum and the

depth of flow is critical depth yc

Further q2

A2 ×1g

=AT

= D

v2

gD= 1 or

vgD

= 1 = Fr

is at critical depth Froude number is 1 if Fr is <1, the flow is called as sub critical at which

depth of flow is more and velocity is less

If Fr is >1, the flow is called as super critical flow at which the depth of flow is less and

velocity is more.

Thus when flow passes from supercritical to sub critical through a critical depth, then

hydraulic jump force as shown in figure in which there is turbulent mixing of the fluid.

Due to this mixing lot of energy is dissipated.

So this concept is used for energy dissipation below spillways etc.

From the specific force diagram it is seen that for the same specific force these are two

possible depths y1 and y2.

Page 211: Fluid Mechanics and Hydraulic Machinery

E310/1 211 The depth y1 is called as super critical depth and the depth y2 is called as subcritical depth.

so these depths for which specific force is same is called as initial depth (y1) and

sequential depth (y2).

Now from specific force we have

qgA

F Az2

= −

or q = gA(F − Az )

so for a given specific force q is maximum when dqdy

= 0

ie ( )dqdy

gA F Az

F AzdAdy

Ad Azdy

=−

× − +−

=

12

1 0( )

( )

ie (F-A z)T = Ad(Az )

dy= A

Adydy

= A2

But (F-A z) =q2

gA

q2

gAT = A2

or q2

g=

A3

Tis the condition for critical flow at which depth is critical and q is

maximum

Thus for given specific energy or specific force, the discharge is maximum when the flow

is critical.

10.5 Computation of critical flow.

When the depth of flow is equal to critical depth, then the flow is called as critical flow.

This can be known by

q2

g=

A3

T

c

qg

= AAT

= zc

where zc is called as section factor for critical flow.

Thus for a given discharge zc is function of depth ‘y’ only.

So for different depths, the relationship between Z and y is shown in figure, where Z =

Page 212: Fluid Mechanics and Hydraulic Machinery

E310/1 212

AAT

, for different depths, Z can be called as A A T and plotted as shown in figure.

Now for a given discharging

Zc =qq

so from the graph, for this section factor Zc we have yc read. which is critical depth. Or

for a given depth of flow, the critical section factor Zc is known from the graph from

which

Zc =qc

g, the critical discharge ‘qc ’ can be calculated.

For a given discharge the slope of bed can be adjusted such that the uniform depth is equal

to critical depth yc . Or for a given depth of flow, the discharge and slope can be adjusted

such that the flow is critical . Then this slope is called as critical slope sc.

Thus when the flow is in critical state we have.

1. E = specific energy is minimum for a given discharge

2. F = specific force is minimum for a given discharge

3. Discharge is maximum for a given E or F

4. Froude number Fr = 1

5. For rectangular channels

yc =23

E or yc =q2

g

13

and ν c

2

2g=

12

yc or Ec = yc +ϑ c

2

2g= yc +

12

yc =32

yc

ie Ec = 32 yc

Identify the following as True or False

Page 213: Fluid Mechanics and Hydraulic Machinery

E310/1 213 SAQ 1. For critical flow at a given discharge Froude Number = 1 and E is minimum

SAQ 2. Discharge per unit width is 1m3/s. Find the critical depth in rectangular channel.

SAQ 3.. For rectangular channel critical depth is 1m. Find the maximum discharge

SAQ 4. For rectangular channel E is 1.5 m find the critical depth

SAQ 5 For a discharge 1m3 /s per metre width the critical depth is 0.465 m Find Froude

number.

SAQ 6. When qyc

= gyc , the flow is critical.

Worked Example (1) A rectangular channel with a bed slope of 1 in 200, carries a

discharge of 10 m3 / s , Find the critical depth if the width of channel is 2m.

q = 102

= 5m3 / s / m

yc =q2

g

13

=52

9.81

13

= 1.368 m

Worked Example (2) If the specific energy of flow in 5m wide rectangular

channel is 1.5 m, then calculate the maximum discharge

yc =23

E =23

×32

= 1m

but yc =q2

g

13

or qmax = gyc3( )

12 = 3.13m 3 / s / m

Worked Example (3) A trapezoidal channel with 5 m bottom width and 2H to 1v

side slope carries a discharge of 9.81 m3 / s. Find the critical depth.

A = (B+zy)y

T = (B+2zy)

Now q2

g=

A3

Tfor critical depth.

(9.81)2

9.81= 9.81 =

(b + 2yc )yc( )3

(b + 4yc )

9.81 13 (b + 4yc )

13 = (b + 2yc )yc

or yc = 0.59

Page 214: Fluid Mechanics and Hydraulic Machinery

E310/1 214 for different depths y = 0.1, 0.2, 0.3 , 0.4, 0.5, 0.6, 0.7. 0.8 find Z = A A T and plot a

curve between z & y as shown in figure

Then find zc =qg

= =9 819 81

9 81..

.

for this value from the graph find yc

10.6 Transitions

A transition is a small portion of a channel of varying cross section connecting two

channel sections. It may be sudden or gradual. It may be contracted or expanded.

It may be obtained either by

a. reducing and expanding bed width or

b. raising or lowering bed

c. by varying both bed width and bed elevation.

These transitions are necessary for measuring devices, to change the velocity to create

hydraulic jump to dissipate energy and to economise cost of hydraulic structures like

aquident etc.

The purpose of a transition is to minimise the energy loss by changing the hydraulic

conditions gradually .

10.6.1 Transitions with reduction in width

In this case the specific energy will be constant for a given discharge in the channel.

When the width is reduced the uniform depth y1 at (1) is reduce to y2 at (2) in subcritical

flow as shown in figure, as the velocity at (2) is increased. This reduction in depth at (2)

continues till it reaches a critical depth at which the discharge per unit width is maximum.

This critical depth yc 2 is less than y2..

If the width at (2) is further reduced beyond the critical flow, the depth at (1) is increased

changing the specific energy E1 at (1) to E11 such that the new depth yc2

1 at (2) will be

the critical depth for the new specific energy E11. The new critical depth yc2

1 is greater

than yc2.

Page 215: Fluid Mechanics and Hydraulic Machinery

E310/1 215

Similarly if the flow is supercritical flow the depth y2 at (2) is more than y1 at (1) for the

given E1 . As the width goes an reduced the depth at (2) ‘y2’ goes on increased, till it

reaches a critical condition at which the discharge qmax per unit width at (2) is maximum

and the depth yc2. If further the width is reduced beyond this critical flow, the head at (1)

is lowered to increase the specific energy E1 such that the new depth it (2) would be

critical depth yc2

1 for the new specific energy. E11 and the new critical depth Y1

c2 is more

than yc2.

Thus for a given discharge and specific energy, there is a limit for a reduction in width at

which the flow is critical and beyond this reduction in width causes a change in u/s depth

Here E1 = E2 = Ec =32

yc

and yc =q2

2

g

13

q2 = q/B2

10.6.2 Transition with raise in bed.

In this for the same discharge, the specific energy varies, when the bed is raised by ∆z as

shown in figure

Page 216: Fluid Mechanics and Hydraulic Machinery

E310/1 216

Here E1 = E2 + ∆z

Hence for a given q1 E reaches a minimum as bed gradually is raised. At this minimum

specific energy the raise in bed ie ∆Zmax is maximum possible. This can be obtained by

E1 = Ec + ∆z max

Ec =32

yc

and yc =q2

g

1 3

As in the previous section here also in case of subcritical flow the depth at (2) goes on

decreases till it reaches critical value yc2which is less than y2 which is less than y1, and

in

case of supercritical the depth ‘y2’ at (2) goes on increases till it reaches a critical value

yc2as shown in figure.

If further raise in bed causes the y, depth to raise or fall as the flow is subcritical

or supercritical.

Page 217: Fluid Mechanics and Hydraulic Machinery

E310/1 217 In case of both reduction in width and raise in bed is analysed by the same procedure.

Identify the following as True or False

SAQ 7 By changing hydraulic conditions the transition reduces loss of energy.

SAQ 8 For a given discharge in a channel the critical depth in the transition is same both

in subcritical and supercritical flows.

SAQ 9 The minimum width or maximum raise in bed of a channel corresponds to critical

flow.

SAQ10 If the specific energy in the channel is 10 m and the critical energy is 9.8 m, the

maximum raise in bed is 0.2 m

SAQ11 If the width is reduced or bed is raised beyond the critical flow, then the v/s depth

will be altered.

SAQ12 A rectangular channel 2.5 m wide, the specific energy is 10m . Then when the

width is reduced till critical flow is obtained the critical specific energy is 10m

SAQ13 The specific energy in a channel is 10.2 m. The maximum raise in bed is 0.2 m.

Then the critical depth is 0.67m.

SAQ 14 Critical velocity head is 12

critical depth

Worked Example (4) A 10 m wide is rectangular channel carries a discharge of 30 m3/s

with 1.66 m depth of flow. Calculate the maximum raise in bed and the corresponding

fall in water surface. If the raise is increased by 10% of the width at critical flow, find the

rise in water bed U/s.

q = 3010

= 3m 3 / s / m

ϑ 1 =3

1.66= 1.8m ϑ 1

2

2g = 0.166m

∴ E1 = y1 +ϑ1

2

2g= 1.66 + 0.166 = 1.826m

and yc = q 2

g

13

=32

9.81

13

= 0.97m

Ec =32

yc = 1.457m

∴ ∆zmax = E1 - Ec = 1.826 -1.457 = 0.36m

Drop in water level = y1 - (∆zmax + yc ) =vc

2

2g−

ϑ12

2g

Page 218: Fluid Mechanics and Hydraulic Machinery

E310/1 218 = 1.66-(0.36+0.97) = 0.32m

or0.97

2− 0.166 = 0.32m

raise in bed level =- 0.1 ×0.36 = 0.036

New raise in bed is 0.396 m

New specific energy U/s is E11 = Ec +∆z = 1.457+0.396 = 1.853m

E11 = 1.853 = y1

1 +32

y11( )2

× 2 × 9.81

y11 = 1.7 m

Worked Example (5) A rectangular channel 10m wide carries a discharge of

10m3/s with a depth of 0.8 m Find the minimum contraction in bed. If the bed width is

further reduced by 10%. What would be the rise in water level U/s.

v1 =qy1

=1

0.8= 1.25m / s

v12 / 2g = 0.0796 m

E1 = y1 + v12 /2g = 0.8 + 0.0796 = 0.8796 m

E1= E2 = Ec

∴ yc = 2 3 Ec =23

× 0.8796 = 0.5814 = q22 g( )

13

∴q2 = yc3g = 0.58643 × 9.81 = 1.4m3 / s / m

vc2 2g =

12

yc =0.5864

2= 0.2932m

vc = 2.398 m/s

10 =Bcyc Vc = Bc ×0.5864×2.398

∴ Bc = 7.11m

or q2 B2 = Q

∴B2 =Qq

m2

1014

714= =.

.

New width = 7.11 × 0.9 = 6.399m

q21 =

100.399

= y113

g = 1.562

∴ yc1 = 0.629m

Ec1 = E1

1 = 3 2 × yc1 = 0.9436m

Page 219: Fluid Mechanics and Hydraulic Machinery

E310/1 219

E11 = y1+v1

2/2g = 0.9436 = y 1 + q2

y12 2g

= y11 +

1y1

2 ×19.62

y11 = 0.86 m

Worked Example (6) A rectangular channel 5 m wide carries a discharge of 10 m3

/s at a depth of 1.5 m. If the bed width is reduced to 3m and a hump[ is constructed to

create critical flow determine the same.

ϑ 1 =qy1

=2

1.5= 1.53m / s

ϑ 12

2g = 0.0906m

E1 = y1 + ϑ 12 2g = 1.681m

q2 =10B2

=103

= 3.33m

yc =q2

2

g

13

=3.332

9.81

13

= 1.04m

E1 = ∆z+yc + vc2 / 2g

= ∆z+ yc +1/2 yc

1.681 = ∆z +1.5 × 1.04

∴∆z = 0.0545m

10.7 Summary

1. Specific energy E = y + ϑ 2

2gis the total energy with r.to bed as datum

2. The condition for critical flow is a prismatic channel is Q2 /g = A3/T where T =

top width of flow both for q constant or E is constant

At this critical flow , E is minimum when Q is constant and Q is maximum for

given E..

3. For rectangular channel critical depth = yc = q2 g( )13

where q = discharge / m width

and F r = ϑc2 / yc g = 1

Ec = 1.5 yc ; vc2 / 2g =

12

yc ; yc =23

Ec

4. Qvg

+ Az is called as specific force

Page 220: Fluid Mechanics and Hydraulic Machinery

E310/1 220 5. For given specific force Q is max at critical flow given by

Q 2

g = A3

T

6. The minimum width Bc of channel is given by critical flow at which

Ec = E1 = 1.5yc

and yc = qc2 g( )

13

where qc = discharge per unit width at (2) = Q /Bc

7. The maximum raise in bed is obtained by critical flow in

E1 = ∆zmax + Ec

Ec = 1.5 yc

yc =q2

g

13

8. Same analysis will be followed in case of both reduction in width and raise in bed.

Here to find the maximum raise in bed the critical flow condition is to be applied.

Answers to SAQ

1. T 2. 0.465 3. 3.13 m3 /s/m 4. 1.m

5. 0.465 6. to 9: T 10. 0.2m 11. and

12. T 13 0.67m 14 T

Exercise

10.1 A rectangular channel 7.5 m wide carries 12 m3 /s with a velocity of 1.5 m/s

compute the specific energy. Find the critical depth and critical velocity and the

minimum specific energy [ 1.18 J/N 0.639m, 2.503 m/s; 0.959 J/N ]

10.2 A trapezoidal channel having a bottom width 5m and side slopes 1:1 carries a

discharge of 12 m3 /s . compute the critical depth and critical velocity. If

Mannings n = 0.02 determine the bottom slope required to maintain the critical

depth [ 0.793 m, 2.61 m/s, 1 in 200 ]

10.3 A trapezoidal channel with a base width of 6 m and side slopes of 2h-1v conveys

water at 17 m3 /s with a depth of 1.5 m. Is the flow subcritical or super critical

[ Fr = 0.38, subcritical ]

10.4 A rectangular channel 2.0 m wide carries a discharge of 6 m3 /s. If the specific

energy is 1.8 m, calculate the alternate depths [ y1 = 0.625 m y2 = 1.626 m]

Page 221: Fluid Mechanics and Hydraulic Machinery

E310/1 221 10.5 corresponding to a discharge of 9 m3 /s, calculate the critical depth in triangular

channel of vertex angle 72 0 [ yc = 1.99m]

10.6 Calculate the discharge corresponding to a critical depth of 1.5 m in a

a. rectangular channel of 1.5 m wide

b. triangular channel of vertex angle of 1200

c. trapezoidal channel of bed width = 6.0 m and side slopes 1.5 h to 1 v

[ a. q = 8.63/m3/s b. 10.571 m3 /s c.. q = 42.08 m3 /s ]

10.7 A rectangular channel 2.4 m wide carries uniform flow of a 7 m3/s at a depth of

1.5m. If there is a local rise of 150 mm in the bed level, calculate the change in

water elevation. What can be the maximum rise in the bed elevation such that the

u /s depth is not effected. [73.3mm, 262 mm]

10.8 Uniform flow occurs at a depth of 1.5 m in a long rectangular channel 3m wide and

laid at a slope of 0.0009 . If Manning’s n = 0.015 calculate the width of

contraction which will produce critical depth without increasing the u/s depth of

flow. [ 2.08m]

10.9 A rectangular channel is reduced gradually from 2 m to 1.5 m and the floor is

raised by 0.25 m at a given section. When the approaching depth of flow is 1.5 m,

what rate of flow will be indicated if the flow at the contracted section is at

critical depth [3.95 m3 /s]

***

FLUID MECHANICS & HYDRAULIC MACHINERY

UNIT XI

DIMENSIONAL ANALYSIS

Aims:

The aims of this unit are to explain the use of dimensional analysis ,to define fundamental

dimensions and units, to explain Rayleigh’s method and Buckingham Theorem and their

applications.

Objectives:

The objectives of this unit are

1. To explain the use of dimensional analysis

Page 222: Fluid Mechanics and Hydraulic Machinery

E310/1 222 2. To define fundamental dimensions and units and to differentiate between unit and

dimension

3 . To explain Rayleigh’s method

4. Toexplain Buckingham Theorem

5. To apply the above in solving practical problems.

11.1 Introduction

For civil Engineers Dimensional analysis is very useful in arriving at an equation

governing a complex phenomenon where mathematical solution is not possible. In this

analysis it is possible to verify the homogeneity of an equation and to derive the non

dimensional numbers which govern the phenomenon

This is very useful in experimental investigation using model study. So let us understand

this clearly.

11.2 Dimensional analysis

In any phenomenon there are number of parameters which govern the phenomenon. In

some simple phenomenon it is possible to arrive at an equation governing phenomenon,

connecting the parameters by mathematical analysis. But it is not always possible in some

complex phenomenae. In this case dimensional analysis is a powerful tool to arrive at an

equation governing the complex phenomenon by connecting the parameters involved in it.

In this analysis it is also possible to know about the non dimensional numbers which

govern the phenomenon and how the constants in the equation will very with the non-

dimensional numbers. Using this analysis we can verify the homogeneity of an equation

and to convert an equation in one system of unit to another system of unit .

11.3 Dimensions and units

Any physical quantity can be expressed in terms of primary or fundamental quantities

Mass (M), length (L) Time (T) and Temperature (θ). Temperature is used in

compressible fluid flow.

The other quantities which are expressed in terms of the primary quantities are called as

secondary quantities viz. area (L2), Volume (L3), Velocity (L/T), Acceleration (L /T2),

some people use force (F) , instead of M, which is called as F.L.T. system instead of

M.L.T. system.

Unit is the standard measurement of the above primary quantities . There are absolute and

gravitational units. In absolute system absolute acceleration viz. 1cm /s 2 , 1m/s2 is used

Page 223: Fluid Mechanics and Hydraulic Machinery

E310/1 223 where as in gravitational units also called as Engineering units, acceleration due to

gravity is used. Thus is absolute units , say in C.G.S units, Mass = 1g(m), Length =

1cm, Time = 1sec, Force = Mass × acceleration = 1g(m)× 1cm / s2 = 1 dyne and in S.I

units , Mass = 1kg(m) , Length = 1m, Time = 1 second, Force = 1kg (m) × 1m / s2 =

Newton. In gravitational units Man = 1 Metric slug, L = Metre , T = second , Force

= 1 kg (m) ×9.81 m / s2 or 1 M.slug ×1m / s2

So Force = 1 kg (f) = 9.81 N

The dimensions and units of important quantities are given below

Quantity symbol Dimension

MLT

FLT

SI

Unit

MKS C.G.S

A) Geometric

Length L L L m m cm

Area A L2 L2 m2 m2 cm2

Volume V L3 L3 m3 m3 cm3

Curvature C 1 / L 1/L m-1 m-1 cm-1

Slope s1i M0L0T0 P0L0T0 m/m m/m cm/cm

Angle α 1θ M0L0T0 F0L0T0 radian or degrees

radian or degrees

radian or degrees

B).Kinematic

Time T T T S S S

Velocity

(linear)

V L/ T L/T m/s m/s cm/.s

angular velocity ω 1/T 1/T` rad/s` rad/s rad/s

Frequency n 1/T 1/T 1/s 1/s 1/s

Acceleration (linear) a L/T2 L/T2 m/s2 m/s2 cm/s2

Angular acceleration α 1/T2 1/T2 rad /s2 rad/s2 rad/s2

Gravitational

acceleration

g L/T2 L/T2 m/s2 m/s2 cm/s2

Discharge Q L3/T L3/T m3/s m3/s cm3/s

Kinematic viscosity v L2/T L2 /T m2/s m2 /s cm2/s

(stake)

Page 224: Fluid Mechanics and Hydraulic Machinery

E310/1 224 circulation J L2/T L2/T m2 /s m2 /s cm2/s

C). Dynamic

Mass M M FT2 /L kg m.slug g(m)

Force F ML/T2 F N kg(f) Dyne

Weight W` ML/T2 F N kg(f) Dyne

Mass density ρ M/L3 FT2 /L4 kg/m3 M.slug/

m3

g(m)/c

m3

sp weight w,r M/L2T2 F/L3 N/m3 kg(f)/m3

Dy/cm3

Sp gravity s M0 L0 T0 F0L0T0 ------- --------- -------

Pr intensity p M/LT2 F/L2 N/m2

(pascal)

kg(f)/m2

Dy/cm2

Shear stress τ M/LT2 F/L2 N/m2 kg(f)

/m2

Dy/cm2

Dynamic viscosity µ M./LT FT/L2 N.S /m2 kg(f).s/

m2

Dy.s/c

m2

(poise)

Surface Tension σ M/T2 F/L N/m kg(f)/m Dy/cm

Modules of elasticity E M/LT2 F/L2 N/m2 kg(f)/m2

Dy/cm2

Compressibility 1/E LT2 /M L2 /F m2 /N m2/kg(f

)

cm2

/Dy

Impulse, Momentum I,M ML/T FT N.S Kg(f).S Dy.s

Work, Energy W,E ML 2 /T2 FL Nm

Joule(J)

Kg(f),m Dy.cm

Torque Γ ML2 /T2 FL N.M KG(f).

m

DY.Cm

Power P ML:2 /T3 FL/T N.m/s

(Wallw)

Kg(f)-

m/s

Dy.cm/

s

Identify the following as True or False

SAQ 1 Complex phenomenon can be solved by Dimensional analysis

Page 225: Fluid Mechanics and Hydraulic Machinery

E310/1 225 SAQ 2 The non dimensional parameters governing any phenomenon can be known by

dimensional analysis

SAQ 3 The homogeneity of an equation can be verified by dimensional analysis

SAQ 4 Any physical quantity in one system of unit can be converted to other system of

unit by Dimensional analysis

SAQ 5 The fundamental quantities to express any physical quantity are called as

dimensions

SAQ 6 The standard measurement of any physical quantity is known as unit.

SAQ 7 The dimension of power is FL/ T

SAQ 8 The unit of pressure in S.I system is Pascal

SAQ 9 The unit of work is Joule in SI system

SAQ 10 The unit of power is SI system is Watt

11.4 Homogenous equations

Fourier’s principle of dimensional homogeneity states that an equation which expresses a

phenomenon of fluid flow must be algebraically correct and dimensionally homogeneous.

An equation is said to be dimensionally homogenous if the dimensions of the both sides of

an equation are the same. This can be explained as follows.

consider v = cϑ 2gH

LHs dimensions = v = L / T

RHs dimensions = 1 × L T 2 × L( )12 = L T

LHs dimensions = RHs Dimensions

So the above equation is homogenous equation

Similarly Q = 2 3 L 2gH 3 2 , hf = fLv2

2gd, hf =

32uvLwD2

are all homogenous equations.

But V = C Rs, Q = 1.84LH3/2

are not homogenous equations

Homogenous equations are independent of units and so these equations are the same in

any system of units where as non homogenous equations are to be modified from system

of unit to another system of units

Page 226: Fluid Mechanics and Hydraulic Machinery

E310/1 226 A dimensionally homogenous equation can be reduced to non dimensional form as

explained below

consider Q = cd 23

L 2gH 3 2 .

QL gH 3 2 = cd

23

2 = constant = M0 L0 T0

Two dimensionally homogenous equations can be multiplied or divided with out effecting

the homogeneity of the resultant equation. But it can not be so if those are added or

subtracted as the resulting equation will not be dimensionally homogenous.

In any dimensionally homogenous equation the number of dimensionless ‘Π’ terms is

equal to number of variables ‘n’ unions the fundamental quantities ‘m’ ie

No of Π terms = (n-m)

Identify the following as True or False

SAQ 11 Dimensionally homogenous equations are the same in any system of units/

SAQ 12 Non homogenous equations are to be modified from one system of unit to

another system of unit.

SAQ 13 The dimensions of both sides of an equation are the same in dimensionally

homogenous equation.

11.5 Rayleigh Method

This method was proposed by Lord Rayleigh in 1899. In this method, if there are ‘n’

variables like x1 , x2 , x3 -------- xn in any complex phenomenon, then dimensionally

homogeneous exponential form of relationship between any one of the variable s and the

remaining (n-1) independent variables can be obtained as shown below.

x1 = c [ x2a x3

b x4c --------------xn

n]

where a,b,c ----n are the exponentials of the independent variables and

C is a dimensionless constant .

Then the dimensions of each variable are substituted in the above equation. Let the

fundamental dimensions be m. Then by equating the exponent of each dimension on both

sides of the equation we get ‘m’ simultaneous equations connecting the exponentials of

the variables. Among the ‘n’ variable there will be (n-m) independent variables and m

repeated variables. The repeated variables are identified by experience and the exponents

of those variables are expressed in terms of the exponentials of independent variables.

Page 227: Fluid Mechanics and Hydraulic Machinery

E310/1 227 substituting these in the above equation the functional relationship can be obtained by

grouping the variables of the same exponential.

11.6 Buckingham Theorem

This theorem states that if there are ‘n’ variables involved in any phenomenon , which can

be described by ‘m’ fundamental quantities and these variables are related by a

dimensionally homogenous equation then the relationship among ‘n’ variables can be

expressed by (n-m) dimensional quantities and each quantity consists of (m+1) variables

where ‘m’ is the repeated variables and other one is independent variable in the (n-m)

variables. The repeated variables should contain all the ‘m’ fundamental quantities and

at the same time they should not form a non dimensional quantity. This can explained as

below.

Let (x1 x2 x3 - - - - - xn ) be ‘n’ variables in any phenomenon . then

x1 = f (x2 x3 - - - - - -xn )

or f1 ( x1 x2 - - - - - -xn ) = c

where ‘c’ is a dimensionless constant. Then the above phenomenon can be expressed by

(n-m) - non dimensional ‘Π’ terms as follows

f2 = ( Π 1 Π 2 - - - - - - - - - Π (n-m) ) = c1

Where each Π term consists of (m+1) variables, where ‘m’ being repeated variables

raised to some unknown exponentials which are to be determined and one more

independent variable which will be selected from the remaining (n-m) variables. as shown

below

Π1 = x1a1 x2

b2 x3c1 − − − − − − − − − xm

m1 xm+1

Π2 = x1a2 x2

b2 x3c2 − − − − − − − − − −xm

m2 xm+ 2

Πn-m = x1an− m x2

bn−m x3cn− m − − − − − xm

mm−m xn

By substituting the dimensions of each variables in the above Π terms , and equating the

dimensions of R.H.S the Π term to zero, ‘m’ simultaneous equations are obtained in terms

of exponentials. Then solving the above equations the un known ‘m’ exponentials are

obtained and then by substituting those in the aboveΠ term eachΠ term can be obtained.

Thus the governing equation of the phenomenon can be obtained by expressing any one of

Π terms as a function of the remaining Π terms as

Page 228: Fluid Mechanics and Hydraulic Machinery

E310/1 228 Π1 = f1 (Π2 Π3 - - - - - - - - - - - - - -Πn-m)

or Π2 = f2 (Π1 Π3 - - - - - - - - - - - - --Πn-m) etc

Identify the following as True or False

SAQ 14 When large no of variables are involved in a phenomenon , Bucking ham Π

Theorem is followed

SAQ 15 In Rayleigh method all variables which are treated as function of another

variable are raised to some exponentials.

SAQ 16 In Rayleigh method ‘m’ simultaneous equations are to be solved by determining

the exponentials of repeated variables in terms of exponentials of other variables.

SAQ 17 Rayleigh method is tedious one when large no of variables are involved.

SAQ 18 In Buckingham Π theorem each Π terms consists of m variables raised to

some unknown exponentials and one more variable out of remaining (n-m) variables.

SAQ 19 In Buckingham Π Theorem any phenomenon involving ‘n’ variables can be

represented ;by (n-m) Π terms where ‘m’ are fundamental dimensions.

SAQ 20 In Buckingham Π theorem a relationship between the variable involved in any

phenomenon can be evolved by expressing one of the Π terms as function of remaining

Πterms.

SAQ 21 Grouping of like exponentials is followed in Rayleigh method

SAQ 22 If the variables are wrongly selected the resulting functional relationship is

erroneous

SAQ 23 In dimensional analysis relation of variables depends on vast experience of

investigator.

Worked Example (1) Check the homogeneity of the following equations

Q = Cd a 2gH

Q = 1.84 LH 3/2

Sol: Lhs of Q = Cd a 2gH , dimension is L3 / T

RHs dimension = 1L2 LT 2 L

12

= L3 / T

∴Q = cd a 2gH dimensionally homogenous is dimension of LHs≠RHs

Now Dimension of LHs of Q = 1.84 LH3/2 is L3 / T

and the dimension of RHs = LL 3/2 = L5/2

Page 229: Fluid Mechanics and Hydraulic Machinery

E310/1 229 ∴ Q = 1.84 LM3/2 is non homogeneous as dimension of LHs ≠ RHs

Worked Example (2) Convert the equation Q = 1.84 LH 3/2 into FPS system

Q = CL H 3/2

C =L3

1LL3 2 =

L12

T

∴Conversion factor from MKS to FPS is

C = (3.28)

12

1= 1.81

∴1.81 ×1.84 LH 3/2 = 3.33 LH3/2 is the equation in FPS system

Worked Example (3) Obtain drag force on sphere of diameter D in a fluid of density P,

and viscosity µ.

Let the velocity of sphere be ‘V’

FD =φ (D,V,ρ,M)

Then FD = C[ Da Vbρcµd]

substituting the dimensions of each variable we have

MLT -2 = M0 L0T 0 La L T( )b ML3

c MLT

d

Equating the exponential of each dimension of both sides, we have

-M- 1 = c+d

-L- 1 = a+b-3c-d

-T- -2 = -b-d

Let D, V, ρ, be repeated variables and os expressing a, b c in terms of d we have

c = 1-d

b = 2-d

a = 1-b+3c+d

a = 1-2+d+3-3d +d = 2-d

∴FD = c[D2-d V2-d ρ1-d µd]

c D 2V 2ρµ

ρvD

d

=c D 2v 2ρρvD

µ

− d

Page 230: Fluid Mechanics and Hydraulic Machinery

E310/1 230 ∴FD = cρv2 D2 f RN( ) = cDAρv 2 / 2

where A =ΠD2 / 4

∴FD = cDΠD2

8ρv2 , where c =

Π8

Worked Example (4) Solve the WE(3) by Buckingham method

Sol:

Let FD = f(Dv ρµ) or c = f1 (FD, D,V, ρµ)

Here n = 5

m = 3(M,L,T)

No of Π terms = n-m = 5-3 = 2 no

Let repeating variables be D,V, ρ

ie Π 1 = f1 (Da1Vb1 ρc1 FD )

and Π2 = f2 (Da2 Vb2 ρc2 µ)

Then substituting the dimensions of each variable in the Π term we have

M0 L0T0 = Π1 = f1 (La1 (l/T))b1 (M/L3)c1 MLT-2)

Equating the dimensions of RHS to 0 we have

-M- 0 =C 1 +1

-L- 0 = a1 +b1 -3c1 +1

-T- 0 = -b1 -2

∴c1 = -1

b1 = -2

a1 = -b1 +3c1 -1

= +2-3-1 = -2

∴Π1 = f1 (D-2 V-2 ρ-1 FD)

= (FD / ρV2 D2)

Similarly Π2 = M0L0T0 = f2 (Da2 vb2 ρc2 µ)

M0L0T0 = f2 La2 (L T )b2 ML3

c 2 MLT

Again equating the dimensions of RHS to 0 we have

-M- 0 = c2 +1

-L- 0 =a2 +b2 -3c2 -1

-T- 0 = -b2 -1

Page 231: Fluid Mechanics and Hydraulic Machinery

E310/1 231 c2 = -1

b2 = -1

a2= -b2 + 3c2 +1

=+1-3+1 = -1

∴Π2 = (D-1 V-1 ρ−1 µ )

=M

ρVD

= RN

−1

So the phenomenon can be represented by 2 Πterms viz.

C1 = f1 (Π1 Π2 )

c1 = f1( FDρv 2 D2 ,

1RN

)

f1 (RN) = FD / FD \ ρv2 D 2

or FD = ρv2D2 f (RN) = CDAρv 2

2

where A = ΠD 2

4

where CD is fn (RN)

11.7 Limitations of Dimensional analysis

1. It will not provide any clue to select correct variables involved in any phenomenon.

Experiments are necessary to verify the variables.

2. If will not give complete equation constants are to be determined by experiments.

Summary

1. Dimensional analysis is very powerful tool to solve complex phenomenon.

2. By dimensional analysis we can verify the homogeneity of equation.

3. By Dimensional analysis empirical equation from one system of unit to another

system of unit.

4. Fundamental Quantities are M.L.T system or F.L.T system which will express any

physical quantity.

5. Units are standard measurements of the dimensions.

6. There are absolute system of units viz. C.G.S system and S.I system and

gravitational system viz. M.K.S system.

7. In Rayleigh method homogenous exponential relationship[ between the variable can

be obtained by equating one of the variables to a function of other remaining

Page 232: Fluid Mechanics and Hydraulic Machinery

E310/1 232 variables raised to unknown exponentials . This method is ttedious when variables

are many

8. In Buckingham Π theorem any phenomenon can be represented by no of Π terms

equal to (n-m) where n = no of variables and m = Fundamental dimensions. The

relationship is obtained by equating one of the Π terms to a function of other Π

terms. Each Π term consists of (m+1) variables where ‘m’ are repeated variables

raised to some unknown exponential and the other independent variables will one of

the remaining (n-m) variables without exponential.

Answers to Short answer questions

(1) - (23) ---True

EXERCISE

11.1 The following is a formula for calculating the maximum flood discharge Q from a catchment of area A

Q = 7000A

A + 4Where Q is in Cu.Ft /s

A is in Sq miles

Convert this to M.K.S system in which Q is in m3 /s and A is in Sq. Km

[Q =123.1AA +10.4

]

11.2 The capillary rise h of a fluid of density ρ and surface tension σ in a tube of

diameter D depends upon the contact angle and gravity g. Obtain an expression for h

by Rayleigh’s method

h D = fσ

lgD2 ,θ

11.3 The critical depth yc in a trapezoidal channel depends upon the discharge Q, the

side slope of the channel, the bottom width B and the gravity g . Obtain expression

for yc by Rayleigh;’s method

Page 233: Fluid Mechanics and Hydraulic Machinery

E310/1 233

yc

B= f m,

QB2 gB

11.4 The stagnation pressure ps in an air flow depends upon the static pressure po , the

velocity V of the free stream and density ρ of the air. Derive a dimensionless

expression for Ps

pspo

= fp0

pv2

11.5 (a) The discharge Q over a v-shaped notch is known to depend on the angle of the

notch, the head of the water surface, the approach velocity v0 , and gravity g .

Determine the dimensionless discharge

(b) Q for any shape neglecting the velocity of approach, connecting viscosity and

surface tension

(c) Q for rectangular notch

(a) Q

H 2 gH= f θ,

v0

gH

(b) Q = H 5 2g12φ

υ

H3

2g1

2,

σH 2ρg

(c) Q = CdLH 3/2 ]

11.6 Obtain expressions for the resistance to motion of a motor boat, assuming it to be

(a) entirely due to viscosity (b) entirely due to waves

(c) due to viscosity & waves and (d) due to viscosity and surface tension.

(a) RρL2 v2 = φ

ρLvµ

, (b)

RρL2 v2 = φ(v2 gL)

(c) R

ρL2 v2 = φρLvµ

,v2

gL

(d)

RρL2 v2 = φ

ρLVµ

lv 2 L

11.7 By dimensional analysis show that ;the torque T on a shift of diameter d,

revolving at a speed N in a fluid of viscosity µ and mass density ρ is given by

T = ρd5 N2 φν

d 2 N

using Rayleigh’s method. Then show that power

p = (ρd5N3) φν

d2 N

Page 234: Fluid Mechanics and Hydraulic Machinery

E310/1 234 11.8 The resistance to the motion of a supersonic air craft of Length L, moving with a

velocity v in air of density ρ depends on viscosity µ & bulk modules of elasticity

K of air. Obtain using Buckingham ‘sΠ theorem, the expression for resistance R

R = ρL2v 2φµ

ρLvKρv 2

11.9 The torque T exerted on the shaft of a turbine is found to be a function of

diameter D, width B and speed N of the runner, densityρ and viscosity µ of the water

flowing in the turbine and difference of pressure P at in let and outlet of turbine. If p

= wH, where w = Sp Wt of water H is the head over turbine , show that torque T may

be expressed by

T = ρN 2 D5φ BD ,

µρD2 N

,gH

D2 N 2

11.10 Obtain an expression for Q through a pump or turbine or compressor which

depends o shaft work gH, Power P, speed of rotation N, diameter of impeller or

sunner D, mass densityρ, and viscosity µ by Bucking ham’s method.

[ QND3 = φ

pρD5 N 3 ,

gHN 2 D 2 ,

µρND 2

]

11.11 For laminar flow in a pipe the drop in pressure ∆p is a function of pipe length L, its

diameter d, mean velocity of flow V and viscosity of fluid µ. By Rayleigh’s

method obtain an expression for ∆p.

[∆p =µvd

φLd

]

11.12 The head loss hL dueto friction in a pipe is function of diameter D, lengthL, rough

ness magnitude ∑ of the pipe, velocity of flow V, the gravity g and fluid densityρ

and viscosity µ then find an expression for hL

hL D = fLD

;∑D

,gDV 2 ,

µρvD

11.13 The shear stress τ0 at the bed of a rough channel depends upon the depth of flow y,

velocity of fluid v, roughness height ∑ of the bed, fluid density ρ and viscosity µ

. Find an expression for τ0

Page 235: Fluid Mechanics and Hydraulic Machinery

E310/1 235

τ 0 ρv2 = fµ

ρvy,∑y

11.14 The terminal velocity of descent v of a hemispherical parachute is found to depend

on its diameter D, weight w acceleration due to gravity g, density of air ρa and

viscosity of air µ . Obtain an expression for V

[vgD

= fw

ρD3 g

,

µρD gD

]

11.15 The lift force F on an airfoil is a function of the angle of attack α velocity

of flow V, chord length C , span L , density ρ, viscosity µ and bulk modulus of

elasticity E obtain an expression for F

F

ρv 2c2 = fρvcµ

,v p

E,Lc

***

FLUID MECHANICS & HYDRAULIC MACHINERY

UNIT XII

HYDRAULIC SIMILITUDE

Aims: The aims of this unit are to define similitude to explain geometrical kinematic and

Dynamic Similarities , to obtain Froude’s , Reynold’s , Mach’s, Wahers’ and Euler’s

numbers, to obtain model laws to explain types of models and to explain scale effects in

models.

Objectives : The objectives of this unit are

1. To define hydraulic similitude

2. To explain geometrical, Kinematic and Dynamic similarity

3..To obtain Froude’s Reynolds, Mack’s, Waher’s and Euler’s numbers and their

use in model study.

4. To obtain different model laws useful in model study.

5. To explain types of models and obtain model scales.

6. To explain scale effects in model study and to how to modify the results of

model in transferring to proto type , using scale effects

7. To apply the above to practical problems.

Page 236: Fluid Mechanics and Hydraulic Machinery

E310/1 236 12.1. Introduction

In complex phenomenon , experimental investigation is necessary to arrive at correct

selection of variables involved in the phenomenon and to arrive at the constants involved

in the equation.

In verification of designs , results on prototype is not possible as it is uneconomical and so

models are to be constructed and verify the designs by model results.

For this purpose model study is quite useful. So a thorough knowledge of hydraulic

similitude is necessary for proper design construction and operation of models.

12.2 Hydraulic similitude

Similitude is defined as the similarity between model and prototype. Then the model will

yield quantitative results about the characteristics of prototype . Compute similitude

between model and pro to type can be obtained if both are geometrically, Kinematically

and dynamically similar. These similitudes are explained as below.

12.2.1 Geometrical Similarity

If the ratios of corresponding linear dimensions like length, width, thickness etc., of the

model and prototype are constant, then geometrical similarity exists between model and

prototype. This means the model and prototype are identical in shape but differ in size.

Let Lr be the linear scale ratio then

Lr =Lm

L p

=bm

bp

=t m

t p

etc.

The Area ratio = A r =Lr

Lp

×bm

br

= Lr2

Volume ratio V r =Lm

L p

×bm

bp

×tm

t p

= Lr3

where subscript ‘m’ refers to model and

‘p’ refers to prototype.

12.2.2 Kinematic similarity

It indicates the similarity of flow pattern(flownets), between model and prototype ,. This

means Kinematic similarity between model and prototype exists when the flow paths of

homologous particles are those particles at similar points around model and prototype as

shown in figure.

Page 237: Fluid Mechanics and Hydraulic Machinery

E310/1 237

This can be obtained when the ratios of velocities and acceleration , of the homologous

particles at the corresponding points are same between model and prototype. For this

similarity, the model and prototype must be geometrically similar. So for Kinematic

similarity we have at similar points

Time scale ratio = (T1 )m

(T1 ) p

=(T2 )m

(T2 )p

= Tr

Velocity scale ratio Vr =(V1 )m

(V2 ) p

=(V2 )m

(V2 )p

=Lm

TmLp Tp

=Lr

Tr

acceleration Scale ar =(a1 )m

(a1 )p

=Lm

Tm2

Lp T 2p

=Lr

Tr2 =

vr

Tr

discharge scale ratio Q r =AmVm

ApVp

= Lr2 Lr

Tr

= Lr3 Tr

or Qr = Lm3 Tm( ) Lp

3 Tp = Lr3

Tr

12.2.3 Dynamic Similarity

Dynamic similarity means the similarity of forces at similar points of model and

prototype. So far dynamic similarity the ratio of all similar forces at homologus points

acting on fluid particles of model and prototype which are geometrically and

kinematically similar, are to be constant or equal. The forces that are acting on fluid

particle at homologus points of model and prototype may be one or combination of

Inertia force Fi

Viscous force Fϑ

Gravity force F gPressure force Fp

Elastic force Fe

Surface tension force Fs

Page 238: Fluid Mechanics and Hydraulic Machinery

E310/1 238 By Newton’s 2nd law we have The resultant force SF = m ×a

(∑ F)m

(∑ F)p

= ((rFv +

rFs + Fp +

rFc +

rFs )m

(rFv +

rFs +

rFp +

rFc +

rFs )p

=(m × a)m

(m × a)p

=(Fi )m

(Fi )p

as inertia force is equal to mass & acceleration.

Also we have

(Fi )m

(Fi ) p

=(Fv )m

(Fv ) p

=(Fg )m

(Fg ) p

=(Fp )m

(Fp ) p

=(Fe )M

(Fe ) p

=(Fs )m

(Fs ) p

or

(Fi )Fv

m

=(Fi )Fv

p

(Fi )Fg

m

=Fi

Fg

p

(Fi )(Fp )

m

=Fi

Fp

p

Fi

Fe

m

=Fi

Fe

m

Fi

Fs

m

=Fi

Fs

p

Thus complete similitude exists between model and prototype when they are geometrically

kinematically and dynamically similar. So in complete similitude the dimensionless

quantities are constant for both model and prototype .

12.3 Force ratios :

When a fluid mass is in motion inertia force exists . So in fluid flows some non

dimensional parameters can be developed which are going to be same for both model and

prototype as ratio of Inertia force to another predominant force in the fluid. viz,

Reynold’s number RN =InertiaforceViscousforce

12.3.1 Reynold’s number

In fluid flows when viscous force is predominant, this number is to be considered, which

is the ratio of Inertia force to viscous force

Inertia force is given by = mass × acceleration

= ρL3 × L T 2

Page 239: Fluid Mechanics and Hydraulic Machinery

E310/1 239 = ρL2 × L2 T 2 = ρL2v 2

and viscous force = shear stress × area of surface

= τ × A

= µ vL

× L2 = µVL

∴ Reynold’s no R N =InertiaforceViscousforce

=ρL2 v2

µvL=

ρLvµ

=vLυ

where υ = µ ρ

∴RN =ρvLµ

=vLυ

For flow through pipes, this number is very important.

12.3.2 Froude number ‘FN’

When gravity force is predominant this number is very useful as in the case of open

channel flows.

Froude number is defined as the square root of ratio of Inertia force to gravity force.

So gravity force = mass × acceleration due to gravity

= ρL3 × g

Froude no : ‘FN’ =ρL2 v 2

ρL3 g=

v2

gL=

vgL

This number is useful in the design of hydraulic structures, ships and in the study of

hydraulic jump etc. where gravity force is predominant

12.3.3 Euler’s number ‘EN’

When pressure gradients exists ie fluid flows i.e where pressure force is predominant then

this number is useful which is the square root of ratio of Inertia force to pressure force.

i.e. Euler’s Number ‘EN’ =ρL2 v2

p × L2 =vp ρ

12.3.4. Mach number ‘MN’

In the case of compressible fluid flows at high velocities like high speed missiles,

projectiles in air, etc, the elastic force is predominant and in this case. Mach number is

useful, which is depend as the square root of cauchy number which in turns is the ratio of

Inertia force to elastic force.

Page 240: Fluid Mechanics and Hydraulic Machinery

E310/1 240

So canchy’s no CN =P2 L2V 2

KL2 =V 2

K ρ

where K is bulk modules of fluid

and C = K ρ which the velocity of sound in that fluid

i.e. MN = CN =Vk ρ

=vc

Here when C = V, MN = 1 which is called as sonic flow

Similarly when V>C, M>1, which is called as supersonic flow and when V<C , M<1,

and so this flow is called as subsonic flow.

When M<0.4, the effect of compressibility of fluid can be neglected.

12.3.5. Weber number ‘WN’

In fluid flows when surface tension forces are predominant then this number is useful

which is defined as square root of ratio of Inertia force to surface tension force

WN =ρL2 v 2

σL=

Vσ ρL

where σ is called as surface tension force per unit length.

This number is useful in case of motion of small jets, droplets , formation of waves and

flow over weir under small heads etc.

For complete similitude the above numbers are to be same for both model and prototype.

Based on this fact the model can be planned, and the test results of the model can be

transferred to prototype.

In planning the models, only the important predominant forces are to be considered and

then less important forces are to be neglected.

12.4 Model laws or similarity laws.

In planning of models, complete similitude is to be considered. But it may not always be

possible because some less important forces in the phenomenon are neglected to make the

problem simple. Also in many practical cases only one force in addition to inertia force is

predominant. So taking this fact into consideration the following model laws are

developed.

12.4.1 Reynold’s Model law

Page 241: Fluid Mechanics and Hydraulic Machinery

E310/1 241 In fluid flows where viscous force predominant Reynolds number is same for both

model and prototype. So

(RN )Model = (RN)Prototype

∴ρm vn Ln

µm

=ρ p vp Lp

µp

i.e. ρr vr Lr

µ r

= 1

or vr Lr

υr

= 1

i.e. vr =υ r

Lr

=µr

ρr Lr

Based on this fact the other scale ratio can be developed as arrived below.

Let Length scale ratio = Lr

Area scale ratio = A r = Lr2

Volume scale ratio = L3r

Velocity scale ratio = vr =υ r

Lr

=µr

ρr Lr

Time scale ratio = T r =Lr

vr

=ρr Lr

2

µr

Acceleration scale ratio = ar =vr

Tr

=µ r

2

ρr2 Lr

3

Discharge scale ratio = Qr =Lr

3

Tr

=Lr µr

ρr

Force scale ratio = mass × acceleration

Fr = µr2 ρr

Work, Energy, Torque scale ratio = Force × distance moved

=µr

2 Lr

ρr

Pressure intensity scale ratio = µ r

2

ρr Lr2 =

ForceArea

power scale ratio = W.D / sec

=µ r

3

ρr2 Lr

Page 242: Fluid Mechanics and Hydraulic Machinery

E310/1 242 Mass scale ratio = ρr L3

r

Mass density scale ratio = ρr

specific weight scale ratio = Mass × g

L3 =µ r

2

Lr3 × ρr

Momentum scale ratio = Mass × velocity = µr L2r

12.4.2 Froude Model law

When gravity force is predominant Froude number is useful in planning the model

and so Froude Model laws are developed based on Froude number which is same for both

model and prototype where gravity force is predominant as explained below.

so (FN)m = (FN)p

i.e. vm

gm Lm

=v p

gp L p

or vr

gr Lr

= 1

i.e. Vr = gr Lr or when gr = 1 vr = Lr

Let Linear scale ratio = Lr

Area scale ratio = L2r

volume scale ratio = Lr3

velocity scale ratio = vr = gr Lr =Lr

Tr

Time scale ratio = Tr =Lr

vr

=Lr

gr Lr

=Lr

gr

Acceleration scale ratio = vr

Tr

= gr

Discharge = Lr

3

Tr

= Lr5

2 gr

Mass scale ratio = ρLr3

Force scale ratio = Mass × acceleration = ρr Lr3 gr = Lr

3υ r

Mass density scale ratio = ρr

Specific weigh scale ratio = ρr gr = υ r

pressure scale ratio = υr gr=ρrLrgr

Momentum scale ratio = ρ r Lr7/2 gr

1/2

Page 243: Fluid Mechanics and Hydraulic Machinery

E310/1 243 Impulse = ρr

1 2 Lr7 2υ r

1 2

Work , energy, scale ratio = L g Lr r r r`4 4ρ υ=

Power scale ratio = Lr7 2ρr gr

3 2 =Lr

7 2υr3 2

ρr1 2

Here is general gr = 1, This law is useful for flows over spill ways, flow through sluices

flow of jet from a nozzle generation of surface waves etc.

12.4.3 Euler’s Model law

When pressure forces control the flow this law is useful in which Euler’s number is same

for both model and prototype.

i.e. (EN)m = (EN)p

i.e. vm

pm ρm

=vp

pp ρ p

or vr

pr ρr

= 1

or Vr = prρr

12

This is useful for flows through pipes where the flow is turbulent in which the viscous

forces and gravity forces are completely absent.

In this case also using Vr , the other scale ratios can be developed is explained in

the previous articles.

12.4.4 Mach Model law

In a compressible fluid flows like flow of air in pipes, or motion of projectiles in air, this

law is very useful in which Mach number is same for both model and prototype. Thus this

law is useful in aero dynamic test, water hammer analysis etc.,

i.e. Vm

Kmρ r

.= or υr = Kr ρ r Based on this scale ratio other scale ratios can be

developed as explained already.

12.4.5 Weber Model law

When surface tension is the predominant force, then this law is useful in which Weber

number is same for both model and prototype.

i.e. vm

σ mρm Lm

=vp

σ pρ p L p

Page 244: Fluid Mechanics and Hydraulic Machinery

E310/1 244

or vr =

σ r

ρr Lr

Making use of this the other scale ratios can be developed from this. For flows over weirs

under low heads, very thin sheet of liquid flowing over a surface this law is useful.

Identify the following whether True or False

SAQ 1 To find constants involved in the equation and correct selection of variables

involved in a complex phenomenon , model study is necessary

SAQ2 To check the designs economically Model study is necessary

SAQ3. Complete similitude exists between model and prototype if both are

geometrically Kinematically and Dynamically similar

SAQ4 When the ratios of corresponding linear dimensions of Model and prototype are

the same then they are geometrically similar

SAQ5 At similar points the ratios of velocity acceleration of homologus particles are

the same, the model and prototype are Kinematically similar

SAQ6 For Kinematic similarity the geometrical similarity is a pre requisite.

SAQ7 For Dynamic similarity the ratio of all similar forces at homologus points acting

over a fluid particle of model and prototype are the same.

SAQ8 For Dynamic similarity the model and prototype are to be Geometrically and

Kinematically similar

SAQ9 In complete similitude the dimensionless quantities are constant for both model

and prototype

SAQ10 In Reynolds model law velocity scale ratio is Vr = µrρr Lr

SAQ11 In Froude Model law velocity scale ratio is

Vr = Lr if gr = 1

SAQ12 In Euler Model law velocity scale ratio vr = pr rρ

SAQ13 If Mach no = 1, the flow is called as sonic flow

SAQ14 If velocity scale ratio vr = σ r ρr Lr then this law is called as Weber

Model Law.

SAQ15 Linear dimension scale ratio in Froude Model is 1/4 , then find Velocity

of flow in model if velocity in prototype is 2m/s

Worked Example(1)

Page 245: Fluid Mechanics and Hydraulic Machinery

E310/1 245 A spillway 7.2 m high and 150 m long discharges 2150 m3 / s under a head of 4m. If a

1:16 model of the spillway is to be constructed, find the model dimensions, head over the

model and discharge in model.

Solution:

Lr = 1/16Tvr =1

16=

14

Qgr = 1

Then Lm

L p

=116

∴Lm =15016

= 9.375m

Qr =Lr5/2 =

116

5 2

= 9.765 ×10−4

∴Qm = Qr × Qp =9.765 × 10 -4 × 2150 = 2.0996 m3/s

Height of model = 7.216

= 0.45m

Head over model = 4

16= 0.25m

Worked Example(2)

Estimate the resistance of boat 100m long when moving in water with a velocity of 8m/s.

The resistance experienced by a model constructed to a scale of 1

100when moving in the

same water is 0.2N. Find velocity and length of model.

Solution

: Fr =ρr Lr

3

gr

Qgr =1, ρr = 1

Fr = Lr3

Fp =Fm

Lr3 = 0.2 ×1003 = 200KN

Vr = Lr =1100

=110

∴VM =1

10× vp =

810

= 0.8m / s

Lr =1

100∴ Length of model =

Lp

100=

100100

= 1m

Worked Example (3)

Page 246: Fluid Mechanics and Hydraulic Machinery

E310/1 246 A model of an open channel is built to a scale of

1100

. If the model has a Mannings

coefficient n = 0.013, find its value in prototype.

Sol: Vr = Lr (1)

But velocity v = 1n

R2

3 s1

2 and Rr =Lr

∴Vr =Rr

2 3

nr

=Lr

23

nr

(2)

by (1) and (2)

Lr =Lr

2 3

nr

∴ nr =Lr

23

Lr

= Lr1 6

∴nm =1

100

16

× np

ie np =nm

1 100( )1 6 = 0.013 ×1001

6 = 0.028

Worked Example (4)

A 1:6 scale model of an aircraft is tested in a wind tunnel. The drag force experienced and

power required by the air-craft to overcome the drag force in air are respectively 6420N

and 642 KW when moving at a velocity of 100 m/s. Find the drag force and power

required to overcome drag and velocity of model. Properties of air are same for both

model and prototype.

Sol: Lr =16

velocity scale ratio Vr =υ r

Lr

=1Lr

= 6

Qυr = 1

∴ vm = 6×up= 6× 100 = 600 m/s

Fr =µr

2

ρ r

= 1 Qρr = 1 & µ r = 1

∴ Drag force on model = Fr × Fp = 6420N

Page 247: Fluid Mechanics and Hydraulic Machinery

E310/1 247

Power required by model = µ r

3

Lr ρr2

×Power of prototype

=1Lr

× 642KW = 6 × 642KW

=3852KW

Worked Example(5)

Caster Oil ( s = 0.96, µ = 9.8s poise ) flows through a pipe of a 1 m diameter at the rate

of 5 m3 /s. If the head lost in a geometrically similar pipe of 5 cm diameter carry water ( µ

= 0.01 poise) is 0.035 cm of water/m length, calculate the head lost.

Solution:

Lr =5

100=

120

For dynamic similarity ρ

µr r r

r

v L= 1

ρr =1000960

µr =0.019.8

Vm

Vp

= Vr =µr

ρ r Lr

=0.019.8

×9601000

×201

=0.01959

Velocity in prototype Vp =Qp

ΠQ p2 / 4

=5

Π4

(1)2= 6.37m / s

Vm = Velocity in model = Vr × Vp

= 6.37 × 0.01957 = 0.1248 m/s

Discharge scale ratio = Lr µ r

ρ r

=1

20×

0.019.8

×960

1000= 4.898× 10-5

∴ Qm = Discharge in model = Qr ×Qp

= 4.8978 × 10 -5× 5= 2.45 × 10-4 m3/s

This is also = Vm × Qm

= 0.1248×Π4

0.05( )2

Page 248: Fluid Mechanics and Hydraulic Machinery

E310/1 248 = 2.45 × 10-4 m3 /s

pressure drop ratio = µ r

2

Lr2ρr

orρ r vr2

=(0.01)2

9.82 ×20( )2

12 ×960

1000= 0.01959( )2

=0.3998 × 10 -3

Pressure drop in model = 0.035100

× 103 = 0.35 Kg / m2 /m

∴ pressure drop in prototype = 0.35

Pr dropscaleratio

=0.35

0.3998 × 10−3 = 875.37kg / m 2 / 20m

=875.37

960= 0.912m of oil in 20m

∴ Pressure drop in on e metre = 0.912

20= 0.04559m

12.5 Types of Models

In general Hydraulic Models can be classified as undistorted and distorted models. A

model is said to be prefect or undistorted if it is similar to prototype i.e all the linear scale

ratios are same. The transfer of results from model to prototype is direct.

A model is said to be distorted when one or more characteristics of the model are not

identical to corresponding characteristic of the prototype. Distortion may be geometric

distortion, material distortion, distortion of hydraulic quantities or combinations of the

above. These models are used in river models and open channel models.

Distorted models are necessary for the following reasons.

1. To maintain accuracy in vertical measurement, otherwise if undistorted model is

designed, the depth of flow, as in the case of a river model, is very small which can not

be measured accurately. Similarly the slope of model channel is different from that of

prototype . So river models, open channel models etc., are going to be distorted models.

2. To maintain turbulent flow

3. To obtain suitable bed material and its adequate measurement

4. To obtain suitable roughness condition

5. To accommodate the available space, money, water supply and time.

Advantages of distorted models.

Page 249: Fluid Mechanics and Hydraulic Machinery

E310/1 249 1. Vertical exaggeration leads in river models steeper slopes, more depth of water,

magnification of wave heights . This leads to accurate measurement.

2. Since slopes are exaggerated turbulent flow is maintained in the model also otherwise,

it would be laminar flow.

3. On Distorted model sufficient tractive force can be developed which cause the bed

material in the model to move

4. Small models which are economical can be planned by distortion.

Disadvantages

1. Because of vertical exaggeration the velocity distribution and so kinematic energy can

not be reproduced.

2. Flow regime is upset because of steep slopes of the distorted model.

3. A model wave may differ from that of prototype in action.

4. Difficult to prepare river bends, earth cuts etc.

5. It causes an unfavourable psychological effect on the observer

12.6 Specific models

1. Submerged objects

In case of testing of aircrafts and submarines, the drag force is given by F = ρL2 v2φ(RN )

and for dynamic similarity

(RN) m = (RN)p

(MN)m = (MN)p

i.e. ρr vr Lr

µ r

= 1 and Vr =kr

ρ r

But on account of range of ρ,µ, k available for range of fluids, it is not possible to satisfy

both conditions. So as one force is taken as secondary force which resulted in scale

effects. Then scale effects are to be accounted in presenting the prototype performance.

Q(RN)m = (RN)p

we have Fm

Fp

= Fr = ρ r Lr2vr

2

But ϑ r =µ r

ρr Lr

Page 250: Fluid Mechanics and Hydraulic Machinery

E310/1 250

∴Fr = ρr Lr2 µ r

2

ρ r2 Lr

2 =µ r

2

ρr

which is same as force scale ratio obtained

by Reynolds model law

∴Fm =µr

2

ρ r

Fp

2. Ship models

It is a partially submerged body . The resistance experienced by it is sum of skin friction

and surface waves friction and pressure drag due to wave formation. But pressure drag is

negligible in case of stream lined body like ship.

∴ Total resistance of ship = R

R = Rw + Rf

where Rw = Resistance due to waves

and Rf = Skin friction

So total resistance = ρL2 v2φ(RN , FN )

For perfect similitude, we have

(RN)m = (RN)p

and (FN)m = (FN)p

i.e. ρr Lr vr

µ r

= 1 and vr

gr Lr

= 1

Vr =µ r

ρr Lr

which also equals to vr = gr Lr

if gr = 1

Vr = Lr

So we have

Vr =µ r

ρr Lr

= Lr

i.e. υ r

Lr

= Lr

or υr = Lr3/2

If the fluid is same for both model and prototype , υr =1 i.e. Lr =1 i.e. Full Scale model is

to be tested which is meaningless. So the usual practice is to consider only Froude Model

law, by towing the model in water ∴ ρr =1

Page 251: Fluid Mechanics and Hydraulic Machinery

E310/1 251 Then the resistance scale ratio Rr = ρr Lr vr

2 Q(RN) & (FN) are same for

both model & prototype

= Lr2 vr

2 Qρr =1

But vr = Lr

∴Rr = Lr2 Lr = Lr

3

Also skin friction (Rf) = fL2v2

where ‘f’ is the frictional resistance per unit surface area per unit velocity which is same

for both model and prototype.

∴ (Rf)r = Lr2 vr

2

but Vr = Lr

∴ (Rf)r = Lr3

Then Rw = (R-Rf) = ρ L2 v2φ(FN)

i.e. (Rw)r = ρr Lr2vr

2

and vr = Lr

( )∴ =R Lw r r rρ 3

Thus (Rw), Rf, and so R = Rw+Rf can be determined.

3. Pressure conduit model

In this case the pressure drop is given ∆p = ρv2φ(RN , ∑ /D )

where ∑ /D indicates the roughness ht of surface, which is neglisible in case of laminar

flow and is to be considered only in turbulent flow.

∴ Pressure drop scale ratio = (∆p)r = ρrvr2 and for dynamic similarity (RN)m = (RN)p

i.e. ρr vr Lr

µ r

= 1

4. River Model

a. Rigid river Model is governed by Froude Model Law which is usually distorted model

to increase the depth of flow in the model , so that it can be measured accurately.

In this case the velocity scale ratio is

Vr =1

Nr

Rr( )2 3Sr( )1 2

And for dynamic similarity,

(FN)m = (FN)p

Page 252: Fluid Mechanics and Hydraulic Machinery

E310/1 252

i.e. vr

gr Lr

= 1 and if gr =1

Then vr = Lr = Dr

vr =1

Nr

Rr( )2 3Sr( )1 2

= Dr

i.e. 1

Nr

Dr2 3 Dr

1 2

Lr1 2 = D r

1Nr

Dr7 6

Lr1 2 = D r

and slope scale ratio Sr =Dr

Lr

It is also equal to Sr =Dr Nr

2

(Rr )4 3

i.e. Dr

Lr

=Dr Nr

2

(Rr )4 3

But for wide river R = D

∴Nr =(Dr )2 3

Lr

b. Movable bed models

In this case also a distorted model is used to get required tractive force in model.

∴ Tractive force τ = γ RS

and the attractive force scale ratio τr =

τr = γr Rr sr

Further this tractive force can be reduced by using light material like coal dust, saw dust,

pumice stone powder etc. Then extreem distortion is not necessary.

Since flow in river is mostly turbulent which can be ensured if Reynolds no is high.

i.e. when V × R >0.007 for turbulent flow

<0.002 for laminar flow

where v is in ‘m/s’

and R = hydraulic mean radius in ‘m’ Another parameter to find turbulent

flow in water is Karman number

i.e. RK =vkυ

=k grs

υ

Page 253: Fluid Mechanics and Hydraulic Machinery

E310/1 253 where V = shear velocity = gRs

and Rk > 50 or 60 for turbulent flow for sand

> 100 for any other roughness

12.7 Scale effect

Complete similitude can be obtained if all the model laws are satisfied. But in many cases

this is not possible. For example if both Froude model law and Reynolds model law are to

be satisfied as in the case of ship model, we get Lr3 2 = υ r . Thus if some fluid is selected,

υr = 1 and so Lr =1 , that means, full scale model is to be followed which is meaningless.

And also it is not possible to have the liquid of desired viscosity. So under these

conditions the secondary important variables are neglected which leads to some

discrepancy in the prediction of behaviour of Prototype. This discrepancy is called as

Scale effects. So the model results are to be corrected when they are to be transferred to

prototype. Similarly in models surface tension also influence the phenomenon which is

not so in case of prototype. This also leads to scale effect as surface tension is not

considered in model design.

In order to arrive at these scale effects, several models with different scales are tried and

the results are compared to judge the resulting scale effects. In this way an empirical

relationship between scale effects and model size may be arrived which can be used to

correct the model results.

Identify the following as True or False

SAQ 17 A model is said to be distorted where one or more no of characteristics of

model will not be identical to the corresponding to characteristics of prototype.

SAQ 18 Distortion is necessary to have meaningful dimensions of model and to

develop required tractive force in model and to obtain turbulent flow.

SAQ19 In distorted model velocity distribution and kinetic energy are not going to be

reproduced

SAQ20 Secondary forces are not considered in the model design. This leads to scale

effects

SAQ 21 By considering both Reynolds model law and Froude model law we get

υr =Lr3/2 which leads to impracticable model.

SAQ 22 In the above problem if some fluid is used in both model and prototype the

scale of model is Full size

Page 254: Fluid Mechanics and Hydraulic Machinery

E310/1 254 SAQ 23 The resistance of ship is equal to Lr

3 if same fluid is used for both model and

prototype.

SAQ 24 In river model Vr = Dr

SAQ 25 In model to maintain turbulent VR > 0.007 or VKυ

> 60 for sand

SAQ 26 If some model laws are neglected leads to scale effect

Worked Example (6)

150m long ship with a wetted area of 3000 m2 . A 1:30 scale ship model is towed in

water at 2m / s to produce a resistance of 40N. Given the ratio of skin frictional

resistance per unit area per unit velocity of model and prototype as 1/3 and for prototype it

is 6.0. Calculate the speed of ship and power required in sea water

(γ =10300N/m3 ) Take propeller efficiency as 75%.

Solution:

Vr = Lr =130

=1

5.477

Vp =vm

Lr

=2Lr

= 2 × 5.477 = 10.94m / s

Surface area of model = 3000

30 × 30=

103

m 2

fp = 6.0 fm =2.0

Rf of Prototype = fpAp vp2

= 6 × 3000 × (10.94)2

= 2154304.8 N = 2154.3KN

Rf for model = 2 ×103

× 22 = 26.7N

Rw for model = 40 -26.7 = 13.33N

(Rw)r = ρ Lr3

=10001030

×1

303

∴ (Rw) of prototype = (Rw)model ×10001030

×1

303

=13.33 ×10301000

× 303 = 370707.3N=370.7KN

Page 255: Fluid Mechanics and Hydraulic Machinery

E310/1 255 ∴ R = Rw+Rf = 370.7+2154.3 = 2525KN

power = R × Vp

0.75

2525 × 10.94 = 36831.3 K W.

Worked Example (7)

A river model with rigid bed has horizontal scale ratio of 1:1000 and vertical scale ratio of

1:100. The discharge in river is 5000 m3/s at roughness coefficient n = 0.03 Find these in

model. Also if the fluid particle takes 1 hr to travel 100 m in the model Determine the

same in river.

Solution:

Vr = Dr =1100

=1

10

Qr = Ar Vr =LrDr Vr =1

100 ×1000×

110

=1

106

∴Qm = Qr × Qp =1

106 × 5000 =1

200m 3 / s

Nr =Dr

2 3

Lr1 2 =

( 1100 )

23

( 11000) 12

= 1.47

Nm = Np × 1.47 = 0.03 × 1.47 = 0.0441

Lr

Vr

=Lr

Dr

=1

1000× 100 =

1100

Tp =Tm

Tr

=1.0hr1/100

= 100hr

Worked Example (8)

A river model with movable bed has horizontal scale of 1 in 500 and vertical scale of 1 in

125. Tractive force in river is 3N/m2 at a slope of 1in 500 and in the model is 0.4N/m2.

calculate the slope of model river if river slope is 1

10000

Sol: τ r =τ m

τ p

=0.43

=1

7.5

Rr = Dr

τ r = γ r Rr sr =γr DrSr

17.5

= 1 ×1

125× Sr

Page 256: Fluid Mechanics and Hydraulic Machinery

E310/1 256 Sr =

1257.5

= 16.65

Model river slope = Sm = sp × sr

=1

10000×16.65 = 1 in 600

Summary

1. Similitude is defined as the similarity between model and prototype. Complete

similitude between model and prototype can be obtained if both are geometrically

kinematically and dynamically similar.

2. If the ratios of corresponding linear dimensions of model and prototype are constant

then both model and prototype are geometrically similar.

Lr =Lm

Lp

=bm

bp

=dm

dp

3. When the ratio of velocity and acceleration of the homologus points at the

corresponding points are the same between model and prototype, then they are

kinematically similar. This means the flow nets are similar.

vr =vm

vp

=Lr

Tr

, Qr =Qm

Qp

, Qr =Qm

Qp

=Lr

3

Tr

4. The ratio of all similar forces acting on fluid particles at homologus points of model

and prototype which geometrically and kinematically similar are the same then they

are dynamically similar. The dimensionless quantities are constant in both model

and prototype.

( Fi /Fr)n = ( Fi / Fr )P

( Fi /Fg)m = ( Fi / Fg )p

( Fi /Fp)m = ( Fi / Fp)p

( Fi /Fe)m = ( Fi /Fe )p

( Fi /Fs)m = ( Fi / Fr )p

5. Forces ratios (Fi /Fr) = Reynolds no = ρvLµ

and Fi

Fv

= Froude no =vgL

Mach no = Fi Fe =vk c

Page 257: Fluid Mechanics and Hydraulic Machinery

E310/1 257 Eulers no = Fi Fp =

vp e

Weber no = WN = Fi Fs =v

σ ρL

6. Model laws

In any model law, the governing nondimensional quantity is same for both model

and prototype and from this velocity scale ratio Vr is obtained and from this other scale

ratios are developed viz

Froude Model law

vr

gr Lr

= 1 ; Vr = Lr if gr = 1

Qr = Lr3 / Tr Qr =

Vr

Tr

=Lr

Tr

Force Scale Ratio = F r =ρr Lr

7 2

Tr

etc.

Similarly for other model scales, Vr is obtained and from this other scale ratios can

be developed.

7. Models broadly can be classified as undistorted and distorted models . If one or

many characteristics of model are not identical with the corresponding

characteristics of prototype, then these are called as distorted.

Distorted Models are necessary to get meaningful measurements in model.

8. If both Reynolds Model and FroudeModel laws are considered we get

υr = Lr3/2 But it is not possible to have fluids of required viscosity or if same

fluid is taken, then the model is of full size which is impracticable. So the

secondary forces are neglected leading to scale effects. These scale effects are

going to be determined by considering different models and by comparing

those test results.

9. For ship resistance

Rr = ρr Lr2vr

2 = Lr3

if Froude Model law is considered in the same fluid ie ρr =1

and Vr = Lr

Page 258: Fluid Mechanics and Hydraulic Machinery

E310/1 258 10. For river model resistance co efficient

Nr =Dr

2 3

Lr

where Vr = Dr

and Sr =Dr Nr

2

(Rr )4 3 =Dr

Lr

called Rr = D r

11. In moving bed river model s

τ r = γr RrSr to maintain turbulent flow VR > 0.007

and V.K

V> 60 for sand

Answers for SAQ

(1) to (15) --- True (16) Velocity model = 1m /s

(17) to (26) --- True

EXERCISE

12.1 Aspillway model is to be built to a geometrically similar scale of 1

40cross a flume ,

of 50 cm width. The prototype is 20 m high and maximum head on it is expected to

be 2m (1) what height of model and what head on the model should be used? (ii) If

the flow over the model at a particular head is 10Lps, what flow per m length of

prototype is expected ?

(iii) If the -ve pressure in the model is 15 cm, what is the -ve pressure in the

prototype ? Is it practicable?

[(I) 0.5 m, 0.05m (ii) 5059.64 lps (iii) 6m , yes]

12. 2 A pipe of diameter 1.8 m is required to transport an oil of sp. gr 0.8 and

viscosity 0.04 poise at the rate of 4 m3/s. Tests were conducted on a 20 cm

diameter pipe using water at 200 c. Find the velocity and rate of flow in the

model . Viscosity of water at 200 c = 0.01 poise

[ 2.829 m/s, 888lps ]

12.3 A model of a submarine of scale 1 /40 is tested in a wind tunnel. Find the speed

of air in wind tunnel if the speed of sub-marine in sea-water is 15 m/s. Also find

the ratio of the resistance between the model and its prototype. Take the values of

Kinematic viscosities for sea-water and air as 0.012 stokes and 0.016 stokes

respectively. The weight density of sea-water and air are given as 1030 kg/m3

and 1.24 kg (f) / m3 respectively.

Page 259: Fluid Mechanics and Hydraulic Machinery

E310/1 259 [800 m/s. Fm / Fp = 0.00214 ]

12.4 A 1:20 model of a flying boat is towed through water. The prototype is moving

in sea-water of sp weight 1024 kg(f) / m3 at a velocity of 15 m/s. Find the

corresponding speed of the model . Also determine the resistance due to waves

on model if the resistance due to waves of prototype is 500 kg(f) .

[3.354 m/s , 0.061 kg (f) ]

12.5 A ship 250 m long moves in sea-water whose specific weight is 1030 kg(f) /m3 .A

1:125 model of this ship is to be tested in a wind tunnel. The velocity of air in

the wind tunnel around the model is 20 m/s and the resistance of the model is 50

kg(f). Determine the velocity of ship in sea-water and also the resistance of the

ship in sea water. The sp. weight of air is given as 1.24 kg(f) / m3 . Take the

kinematic viscosity of sea water and air as 0.012 stokes and 0.018 stokes

respectively

[0.106 m/s, 18228.7 kg(f) ]

12.6 In a 1:30 model of a stilling basin the height of the hydraulic jump in the model

is found to be 0.25 m. What is the height of the hydraulic jump in the prototype?

If the energy dissipated in the model is 0.002 h.p, Find the corresponding h.p

lost in prototype?

[ 7.5, 296 hp]

12.7 A 1:6 scale model of a canal is made to study wave motion . If in the

prototype canal a waves describes a certain distance in 20 sec, what would be the

time taken by the wave to travel a corresponding distance in the model

[ 8.16 sec]

12.8 A 1:25 model of a navel ship having a submerged area of 4 m2 and length 6 m

has a total drag of 1.50 kg(f) when towed through water at a velocity of 1.15

m/s . Find the total drag on the prototype when moving at the corresponding

speed. Use the relation Ff =12

cf ρAV2 for calculating the skin friction

resistance. The value of cf is given by

cf = 0.0735Re( )1 5

Take Kinematic viscosity for sea water as 0.01 stokes and sp weight for sea

water as 1000kg(f) /m3 .

Page 260: Fluid Mechanics and Hydraulic Machinery

E310/1 260 [15008 kg (f) ]

12.9 A 1: 25 scale model was tested to estimate the resistance of the prototype ship.

The model has a wetted surface area of 2 m2 . The prototype velocity is 3 m/s

in sea water of density 1.04 g (f) /cm3 . When the model was towed at the

corresponding speed, the total resistance was found to be 2.65 kg(f). The skin

resistance of the model in fresh water is 0.0094 ϑ 1.9 kg(f) / m2 and that of

the ship in sea water is 0.009 ϑ 1.95 kg(f) /m2 .

Calculate (i) The corresponding speed of model and

(ii) The total resistance of ship

[ 0.6 m/s, 42990 kg(f) ]

12.10 A concrete open channel has Manning’s n = 0.014. A 1:64 model of this channel

is needed. Find the value of ‘n’ for model

[nn =0.007 ]

12.11 The mean velocities in the river and its model are respectively 3m/s & 1m./s .

If the slopes in the river and model are 1in 2500 and 1 in 200 respectively,

calculate the length ratio

[ 1in 112.5]

12.12 A river model constructed with a horizontal scale of 1in 4900 and a vertical scale

of 1 in 225 has a discharge ratio of 1 in 1.8 × 107 . Obtain the scale ratios for

velocity and rugosity coefficient [ 0.0612, 2.06]

12.13. A rectangular channel 50 cm wide and 60 cm deep with water supply of 50 lps is

available in the Laboratory for making a model of spillway 50 cm crest length

with a height of 17 m discharging 1000 on crimes under a head of 1.10 m .

Design a suitable model. Will it be distorted.

[ Lr =1

100, Dr =

1342

, yes ]

***